You are on page 1of 217

TableofContents

TableofContents
Introduction
TheMagooshTeam
WhatisMagoosh?
FeaturedIn
WhyOurStudentsLoveUs
MeettheAuthors
FromtheAuthorofthiseBook
TheSATischanging
1)Youvecometotherightplace
2)Getthetestoutoftheway
3)Bethe rsttotakearetake
MeettheRedesignedSAT
OhtheSATisa-changin
TheFormatoftheOldSATvstheNewSAT
GeneralSATTipsandStrategies
HowLongShouldYouStudyfortheSAT?
The70-150PointPlan=OneMonth
The150-250PointIncrease=OneSummer
The250-350PointClub
The1350+Plan
HowtoMakeYourSATPrepStick
Enlistafriendasastudybuddy
ReallypickapartpracticeSATquestions
Locateyourweakareasandfocusonthem
HowtoTakeaPracticeSAT(TheRightWay)
Buildafoundation rst
Timeyourself
Dontgetinterrupted
Figureoutyourwronganswers


sat.magoosh.com 1

SATScoring
ScoringOnTheRedesignedSAT
AvoidScoreDrama
Dontspreadscoresaround
SATWritingTest
IntrotoSATWriting
Whatyouneedtoknow
BasicTipsforSATWriting
1.Toreadornottoread
2.Knowyourrules(akaStandardEnglishConventions)
3.Knowthequestiontypes
4.Importanceofcontext
StandardEnglishConventions
SentenceStructure
1.SentenceBoundaries
2.Subordinationandcoordination
Mini-quiz:
Answersandexplanation:
Aquicknoteonsubordination
3.Parallelstructure
4.Modi erPlacement
5.InappropriateShiftsinVerbTense,Mood,andVoice
6.InappropriateShiftsinPronounPersonandNumber
ConventionsofUsage
1.PronounClarity
2.Possessivedeterminers
3.Agreement
4.Frequentlyconfusedwords
5.Logicalcomparison
6.Conventionalexpression
ConventionsofPunctuation
1.End-of-sentencepunctuation
2.Within-sentencepunctuation


sat.magoosh.com 2

3.Possessivenounsandpronouns
4.Itemsinaseries
5.Nonrestrictiveandparentheticalelements
6.Unnecessarypunctuation
ExpressionofIdeas:TheArtofWriting
Intro
Development
Organization
E ectiveLanguageUse
PuttingitallTogether:PracticePassage
PracticePassage
SATReadingTest
IntroductiontotheSATReadingTest
TheSATReadingTest:Strategies
ActiveReading
Pacing
ANoteonSATReadingPassages
PassageTypes
TheLiteraturePassage
TheNon ctionPassages
ThePairedPassage
PassageComplexity
SATReadingQuestionTypes
CommandofEvidence
WordsinContext
AnalysisQuestions
DirectReference/LineReferenceQuestions
InferenceQuestions
HowtoImproveonNewSATReading
WrongAnswersonSATReading
ReadingSATPassagesFaster
HowtoStayFocusedonSATReadingPassages
WhySATReadingcanbeboring


sat.magoosh.com 3

Howtostaysharp
TakingNotesonSATReadingPassages
MentalSnapshotsatWork
WrittenNotes
ATipifYouareStillFindingitImpossibletoFocus
PracticePassage
SATMathTest
IntrotoSATMath
WhattoKnowabouttheChangesinNewSATMath
SATMathConceptandSectionBreakdown
HeartofAlgebra
PracticeQuestions
PassporttoAdvancedMath
CoordinateGeometryandPassporttoAdvancedMath
ProblemSolvingandDataAnalysis
Ratio,Proportion,Units,andPercentage
Proportions
Graphs,Tables,andScatterplots
Scatterplots
ABestFitLine
DataandStatistics
Histograms
Themedian
Themean
Medianvs.Mean
SubjectsandTreatments
Conclusion:
AdditionalTopicsinMath
Geometry
GeometryFormulaCheatSheet
CoordinateGeometry
Trigonometry
TipsandStrategiesforPreparingforNewSATMath


sat.magoosh.com 4

SATStudySchedules
MakingtheMostofYourSATStudySchedule
HowtoUseanSATStudyGuide
WhichStrategyWillWorkBestforMe?
HowDoIKnowifMyStudyPlanWorks?
FindingandUsingGreatSATResources
SATResources
TestDayTips
BookReviews
TheGood(theBestSATBooksof2016)
TheBad
The(notso)Ugly
2016BestSATBookReviews
TheO cialSATStudyGuide2016
BarronsStrategiesandPracticefortheNewPSAT/NMSQT
KaplanNewSAT2016Strategies,PracticeandReviewwith3PracticeTests
PrincetonReview500+PracticeQuestionsfortheNewSAT
AdditionalTestPrepResources
MagooshSATYouTubeChannel
ACTPrepResources
YourCollegeSearch
Step1:TakeanInventoryofYourself
Step2:EstablishYourMust-Haves
Step3:Research,Explore,andVisit
Step4:Re neYourList
Step5:ExpressYourInterest
FreeCollegeSearchResources
IntrotoFinancialAid
WhydoIneed nancialaid?
Deadlines
CSS
Separate nancialaiddocuments
1)Institution-speci c


sat.magoosh.com 5

2)Calgrants
Other
MeritAid
Privatescholarships
Takeaway


sat.magoosh.com 6

Introduction

WelcometotheMagoosheBook!Wehopetotakewhatmanyseeasagruelingpre-collegeritual--preppingfor
theSAT--andturningitintoanexperiencethatisbothfunandinstructive.ThiseBookismeanttoserveasa
comprehensiveoverviewoftheSAT,combiningcrucialinformationonteststructureandquestiontypeswhile
providingessentialstrategiesandtipsfordoingyourbesttestday.

TheinformationinthiseBookisasynthesisofsomeofthebestcontentontheMagooshHighSchoolBlog.So
nomatterwhereyouareinyourstudies,ifyourepreparingfortheSAT,thiseBookisforyou!

IfyourereadingthiseBookasaPDFonacomputerortablet,youcanclickonspeci csectionsintheTableof
Contentsifyouwanttoskiparound.

Ifyourealreadyfamiliarwiththeexamandarelookingformorestudymaterial,headovertotheResources
section!


sat.magoosh.com 7

TheMagooshTeam

WereateamofpassionateeducatorsinBerkeley,California.Welikewordgames,videogames,andhelping
studentsdoreallywellonstandardizedexamssothattheycanachievetheireducationaldreams!:)

YoucanlearnmoreaboutusandwhatwedoonourTeampage.Ifyouhaveanyquestions,feelfreetocontact
usathelp@magoosh.com!


sat.magoosh.com 8

WhatisMagoosh?

MagooshisanonlineSATprepcoursethato ers:
140+uniquelessonsonallSATsubjects
400+Reading,Math,andWritingpracticequestions,withvideoexplanationsaftereveryquestion
Materialcreatedbyexperttutors,whohavein-depthknowledgeoftheSAT
Accessanytime,anywherefromanyinternet-connecteddevice
EmailsupportfromexperiencedSATtutors
Customizablepracticesessionsandquizzes
Full-lengthtimedpracticetest
Personalizedstatisticsbasedonyourperformance


sat.magoosh.com 9

FeaturedIn

WhyOurStudentsLoveUs


IthinkMagooshde nitelyhelpedme!!!IuseditonlyforacoupleofweeksinadvanceofthetestandI
reallywishIhaduseditmonthsinadvance.AllyourvideolessonsareincrediblyhelpfulandIlovethe
amountofpracticequestionsyouprovide.Loveditsomuchandithelpedmyscorebymorethan
100points...andthatwasjuststudyingtheweekbefore!

HelpedmereviewthefundamentalsofmathandwritingthatIhaveforgotten.Alsoprovidedmewith
usefultipstouseonthetest.Thedashboardreallyhelpedmeunderstandmystudying
performancesandhabits.

MagooshplacedmeinasettingwhereIcanberightinthecomfortofmyhomewatchingclearand
coherentinstructionalvideostolearnabouttopicsIwasn'tsureaboutorsawthatIwasweakinfrom
thediagnostic.Thesta alsoalwayssentoutthemostfruitfulremindersandinformationinemails
andIwasabletoincreasemySATscoreby200pointsinamatterofamonth.


sat.magoosh.com 10

MeettheAuthors

Herearetheawesomeinstructors(andstudents)whowrotethecontentforthiseBook:

ChrisLele
Forthelasttenyears,ChrishasbeenhelpingstudentsexcelontheSAT,GREandGMAT.In
thistime,hescoached5studentstoaperfectSATscore.SomeofhisGREstudentshave
raisedtheirscoresbynearly400points.HehastakenmanyGMATstudentsfromthe
doldrumsofthe600stothecovetedlandofthe700+.Rumorhasithedoesasecrethappy
dancewhenhisstudentsgetaperfectscore.

KristinFracchia
KristinistheACTExpertwhocreatesawesomelyfunlessonsandpracticematerialsfor
students.WithaPhDfromUCIrvineanddegreesinEducationandEnglish,shesbeen
workingineducationsince2004andhashelpedstudentsprepareforstandardizedtests,as
wellascollege/graduateschooladmissions,since2007.Sheenjoysmarathonrunning,
backpacking,hotyoga,andesotericknowledge.

MikeMcGarry
MikecreatesexpertlessonsandpracticequestionstoguideGMAT(andother)studentsto
success.HehasaBSinPhysicsandanMAinReligion,bothfromHarvard,andover20
yearsofteachingexperiencespecializinginmath,science,andstandardizedexams.Mike
likessmashingfoosballsintoorbit,anddespitehavingnoobviouscranialde ciency,he
insistsonrootingfortheNYMets.

RitaKreig
Ritahelpshighschoolers ndMagoosh,improvetheirSAT/ACTscores,andgetintotheir
dreamschools.SheearnedbothherBAandMasterofPaci cInternationalA airsfromUC
SanDiego,whereshealsostudiedSpanish,French,andPortuguese.Ritaloveseducation
andcommunitydevelopment,justasmuchasshelovesvinyasayogaandbakingcookies.


sat.magoosh.com 11

LucasFink
LucasistheteacherbehindMagooshTOEFL.HesbeenteachingTOEFLpreparationand
moregeneralEnglishsince2009,andtheSATsince2008.BetweenhistimeatBard
Collegeandteachingabroad,hehasstudiedJapanese,Czech,andKorean.Noneofthem
comeinhandy,nowadays.


MaddiLee
MaddistartedwritingforMagooshasahighschooljunior.Sheisanavidfreelancewriterand
hasbeenfeaturedinmultipleliterarypublicationsandanthologies.Whensheisntwriting,
shelovestraveling,doodling,andmostofall,sleeping.Throughherownexperienceand
passion,shehopestohelpguidefellowstudentsthroughtherollercoasterthatisSATand
collegeadmissions.MaddicurrentlyattendsYale.


sat.magoosh.com 12

FromtheAuthorofthiseBook

Hi,mynameisChrisandIlovetestprep.Okay,Iknowthatslikeanuberdorkythingtosay.Atthesametime,
beingaroundthesetestsformanyyearshasgivenmeasenseofthewaythatthetestwritersask
questionsandcreatetrapanswers!andIlookforwardtoimpartingthisknowledgetoyou.Myhopeisthat
afteryougothroughthiseBook,thoughyoumightnotnecessarilyhearttestprep,butyouwillgetadeeper
understandingofthenewSATandusethattoyouradvantage.

Imalsoheretotellyouthatbecominggoodattestprepislikeanyotherskill:youhavetopractice.Taking
standardizedtestsisnotaninbornskill.Manyoftenhavemoreexposurethanwedo,yetwepersistatthinking,
oh,theyrejustgoodattests.Sowhilemanyoftheconceptsinthisbookmightbenewtoyou(thoughIm
guessingquiteafewwillbefamiliar),justremember:themoreyoulearnandthemoreyoupracticeSATtest
questions,themorelikelyyouaretoimprove.

Ofcourse,itsnotthateasy.Youllrealizethatyouarebetteratcertainpartsthanothers.Reading,forexample,
mightbeeasierforyousinceyouenjoyreadinginyoursparetime,whereascomplicatedgraphsdealingwith
threedi erentvariableswillmakeyourheadspin.Remember,though,thatmosthavethesesimilarissues.What
willhelpyougetthroughthetroublespotsarepersistence,andapositiveattitude.Youwillthinkofquestions
youanswerincorrectlyasopportunitiesforlearning.

Tohelpyoubetterunderstandthepartofthetestyouneedtoworkon,Ialongwithmyteamofcontent
expertshavebrokenthetestupintowriting,reading,andmath.Asyouworkyourwaythroughthisbook,I
recommendhighlythatyoudoactualSATpracticequestions.Nothingcanhelpyouimprovemorethandoing
actualquestions.Ofcourse,thosepracticequestionsarebasedonfundamentalsandconcepts,andthats
wherethiseBookcomesin.OnewayofusingitisbydoingCollegeBoardpracticequestions, guringoutyour
areasofweakness,andthenreadinguponthoseareasinthisbook.Oryoucanjustreadthroughtheentire
eBookbeforetacklingquestions,andthencomebacktospeci cpartsoftheeBook.

Finally,ifyouhitaplateauorawall,orwhatevermetaphoryouwanttoemployfornotimproving,stop.Andtake
abreak.Ourbrainsoftenlearnevenwhilewearenotlearninginfrontofabook.Takingacoupleofdayso
fromparabolaswilllikelymakeiteasierforyoutounderstandthemthesecondtimearound.So,fromallofus


sat.magoosh.com 13

hereatMagoosh,goodluckonyourSATjourney.Andwhoknows,youmightactuallyendupenjoyingtestprep
morethanyouthought.

TheSATischanging

WhenmanyhighschoolfreshmanfoundoutacoupleofyearsbackthattheSATwasundergoingitsmost
dramaticchangesever,theyreactedinatotallypredictablefashion:theyfreakedout.

Afterall,theywouldbeguineapigsnotjustforanytestbutforatestthatmorethananyothercoulddetermine
theirfutures.Sincethen,thewritersofthetestTheCollegeBoardhavereleasedreamsofnewmaterialand
practicetests.Yet,thereissomuchlingeringdreadtothenewtestthatmanyareoptingtotaketheACT,the
onceperceivedunderperformingsiblingtothemightySAT,butatestthathassincegainedmorerespectability
amongstcollegeadmissionsboards.

Tomakemattersworse,atleastasfarasthenewSATwasconcerned,noscoringdatahadbeenreleased.So,
youcouldtakeanyoneofthefourpracticetestsavailableyethavenoideahowyoufaredcomparedtoother
students--norknowwhatscoreontheoldtestyournewSATscorecorrespondedto.
Buthereyouareanyway,andIamgoingtotellyouthatyoushouldntworrytoomuch,thatyoumightactually
usethissituationtoyouradvantage.How?

1)Youvecometotherightplace

BycheckingouttheMagoosheBookandallthatito ers,youalreadyhaveaninsiderstakeonthetest.

2)Getthetestoutoftheway

Aslongasyouprepandareseriousaboutthetest,youshouldtaketheopportunitytotakethetestinthe rst
fewmonths.Thatwayyoucanfocusonotheraspectsofyourcollegeapplications.

3)Bethe rsttotakearetake


sat.magoosh.com 14

Byprepping rst,youcanalsobeoneofthe rsttretakethetest.Nowbeforeyoubalk,youshouldrealizethata


retakeisnotseenasabadthing.Infact,admissionsboardsexpectyoutotakethetestseveraltimes.Whatthey
arelookingforisscoreimprovementbetweentests.Whoknows,admissiono cesmightevengiveyouextra
pointsforbeingoneofthepluckysoulstotakethe rstfewtests.

Takingthetestthe rsttimewillalsogiveyouagoodsenseofexactlyofhowyoudointheactualtesting
environment.Thiswillgiveyouvaluablefeedbackonhowtoimprovethesecondtimearound,andwhattostudy
beforetheretake.

ThiseBookwillhelpyoualongyourtestprepjourney!Andifyouhaveanyquestions,wererealpeople(not
robots)readytohelp!Youcancontactusathelp@magoosh.com.


sat.magoosh.com 15

MeettheRedesignedSAT


sat.magoosh.com 16

OhtheSATisa-changin

TheSATexamhastakenonmanydi erentformssinceitdebutedinthe1920s,buttherecentredesignisbyfar
thelargestchangeinthelast30years.AsofMarch2016,theCollegeBoardreleasedanewSATexamthataims
tobetterrepresentwhatstudentslearninhighschool,andbeafairerindicatoroffuturecollegeperformance.
Theactuale ectsthatthenewSATformatwillhaveoncollegeadmissionsremaintobeseen.Inthemeantime,
letsfocusonwhatyoucanexpectfromthisnewlyredesigned,refurbished,andrestructuredexam!

TheFormatoftheOldSATvstheNewSAT

Letsstartwithaside-by-sidecomparison:

Category OldSAT(Pre-March2016) NewSAT(March2016-Present)

ExamLength 3hoursand45minutes 3hours(+50minutesfortheoptionalessay)

Sections 1. CriticalReading 1. Evidence-BasedReadingandWriting


2. Writing a. ReadingTest
3. Math b. Writing&LanguageTest
4. Essay 2. Math
a. No-calculatorsection
b. Calculatorsection
3. Essay(optional)

Essay Required(25minutes) Optional(50minutes)

Score 600-2400 400-1600

Subscoring None Subscoresgivenforeverysection

GuessingPenalty -pointforeverywronganswer None


sat.magoosh.com 17

Pleasenotethatthereisnolongerapenaltyforgettingananswerwrong.IfyounevertooktheoldSAT,then
youmightwonderifthisisabigdeal.Letmetellyouitis.

Okay,onemorechartbeforewemoveon.LetslookatwhatyoucanexpectineachsectionofthenewSAT
exam:

Section WhattoExpect

Reading 65minutes
52multiple-choicequestions
Readpassagesfromliterature,historicaldocuments,socialsciences,andnatural
sciences.Answerquestionsaboutinformationandideasinthetext,theauthorsuse
oflanguage,andhowpassagesrelate.

Writing& 35minutes
Language 4passages,44multiple-choicequestions
Readpassagesandanswerquestionsrelatingtogrammar,vocabularyincontext,
andediting.Thefourpassagesrelateto:careers,socialstudies,humanities,and
science.

Math 80minutes
No-calculatorsection:25minutes
Calculatorsection:55minutes
58multiple-choicequestions
No-calculatorsection:20questions
Calculatorsection:38questions
1setofextended-thinkinggrid-inquestions
Answerquestionsrelatingto:
HeartofAlgebra:AlgebraI,includinglinearequationsandinequalities
ProblemSolving&DataAnalysis:Interpretingdata,analyzingrelationships
PassporttoAdvancedMath:AlgebraII(includingquadraticandhigh-order
equations
AdditionalTopicsinMath:geometry,trigonometry,complexnumbers


sat.magoosh.com 18

Essay 50minutes
(Optional) 1essay
Readthepassage,analyzetheauthorsargument,anddemonstrateyourabilityto
writeane ectiveessay.Thepromptbarelychangesfromtesttotest,butthe
passageisdi erenteachtime.

Alright,thataboutcoversthebasics.Letsmoveontosomegeneraltipsandtest-takingstrategies.GeneralSAT
TipsaStrategies


sat.magoosh.com 19

GeneralSATTipsandStrategies


sat.magoosh.com 20

HowLongShouldYouStudyfortheSAT?

Somewouldsayaweekend;somewouldsayalifetime.Clearly,thetruthissomewhereinbetween,andthe
answerreallydependsonyou.Areyouthebusytype,buthaveloftygoals?Thenstudyinganentiresummeris
probablynecessary(andmore,ifyoudontdoaswellasplanned).NotlookingtorocktheIvyLeague,but
hopingforadecentin-stateschool?Maybeamonthisallyoullneed.

Butdontspendjustaweekend.Whereasamonthcanmakeabigdi erenceinyourscore,dependingonthe
materialsandwhetheryouhaveateacher,aweekendisntgoingtomakemuchofadi erenceonyourscore
(andall-nightersareneverfun).

Onthe ipside,dontfritteryourlifeaway,attendingSATschoolseveryweekend,yearafteryear(collegesdo
wantwell-roundedstudents).Ofcourse,moststudentsdonotfallintothiscategory,butifyouarea
super-achieverkeepthisadviceismind.

Ultimately,howlongyoustudyfortheSAThasalottodowithyourcurrentscoreandtheaveragescoreofthe
schoolyouhopetogoto.Figurethatout rst,andthenyoullhaveabettersenseofjusthowmuchtimeyoull
need.

The70-150PointPlan=OneMonth

Lifeisbusy,andyoujustdonthavetimetodedicateitalltoonething.ButaslongasyoucangiveSATprepa
monthofyourtime,youcanwithhardworkandagoodteacherexpecttoimproveabout30to70pointsin
bothMathandReading/Writing,oruptoaround150pointsoverall.

The150-250PointIncrease=OneSummer

Withtherightmaterialsandagoodteacher,asummerisampletimeforyoutoraiseyourscorebyasmuchas
250points(assumingyouarenotstartingo withascoreofover1350).Youwillneedtotakepracticetests
everyweekandgothroughthemwithareallygoodteacher/tutor.Thatpersonwillalsohelpdetermineyour
weakpoints.


sat.magoosh.com 21

Ifyouareunabletogetateacher/tutor,youcanstillseesuchapointincrease.Itlljustbealittletougher;youll
havetobemoredisciplinedandattunedtotheerrorsyoutendtomake.

The250-350PointClub

Firsto ,Imheretotellyouthatsuchascoreincreaseispossible.Secondly,thehigheryourbaselinescore(the
scoreyoustartoutat),thelesslikelyitistoseesuchanincrease.Soyeah,ifyouarealreadyscoring1400,then
itisimpossibletoincreaseby300points.

Butforstudentsscoringinthe900-1200range,a300-pointincreaseispossible,givenlotsofhardworkand,of
course,time.Youwillneedtobecomeanavidreader,amathmachine,andagrammarnazi(oratleastthinklike
one).Butthesearealllearnableskills,withtherightguidanceandmaterials.Youwillprobablyneedmorethana
summer;infact,youmight ndyourselfworkingback-to-backsummers.Butthatsokay,ifyouareinbetween
yoursophomoreandjunioryears.Withthedetermination,youcandoit.

The1350+Plan

Letssayyouarealreadystartingatthe1350-or1400-level.Increasing100ormorepointsisgoingtobetough.
Butyouareprobablyalreadyastrongstudent.Youmightonlyneedabout6weeks,butthatwillhavetobean
intense6weeks(youwillbedreaminginfourth-degreepolynomialsandReadingpassagequotes).Going
throughmultiplepracticetestsaweekwithanSATtutormaybenecessary.Withanentiresummer,andtheright
instruction,youshouldbeabletobreakthe1500-pointthreshold.


sat.magoosh.com 22

HowtoMakeYourSATPrepStick

Whenstudyinganything,SATtopicsincluded,therestheconstant,underlyingdangerthatyoumayforgetwhat
youvelearned.RememberthatSpanishquizinseventhgrade?Theoneyoustudiedforthenightbefore,but
cameintoclassfeelingtotallyunpreparedfor?Ormaybeitwasntseventhgrade;maybeitwaslastweek.Most
ofusknowthefeelingprettywell.

SowhileyourepreppingforyourSAT,howcanyoumakesurethatthethingsyougraspdontjustslipaway?

Enlistafriendasastudybuddy

Unlikeyourothertests,youdonthavetochoosefromthepeopleactuallyinyourclassestostudyfortheSAT
with.Sincemostkidsaregoingtotakeitanyway,youcanpickandchoosejustaboutanybodyinyouryearto
practicewith.Doingitwithafriendmightreallymakethewholethingalotmoreenjoyable,whichisimportant
foravoidingSATprepapathy.

Butthemostimportantthingaboutworkingwithsomebodyelseisthatitmakesyouactuallyengagewiththe
questions.Ifyoureworkingtogether,thentherewillconstantlybeonepersonexplainingtheirprocesstothe
otherandthatsgoodforbothsides.Thepersonexplaininghastoclarifytheirthoughtsratherthanjust
answeringthequestionandmovingon,andthathelpsthemtoremembertheexperience.Likewise,theperson
listeninghastotrytofollowthethinkingoftheirfriend,whichgivesthemsomethingmorepersonaltointeract
withthanthecoldnessofanSATprepbookalone.Goingbackandforthactingasteacherandstudentisa
greatwaytoapproachthissoneitherpersonisleftjustbeingsilent.

ReallypickapartpracticeSATquestions

Aftereverypracticequestion,yourgoalshouldbetoknowexactlyhowthatquestionwascreated.Youshould
strivetobeabletowritethetestyourself,evenifyourenotplanningtoapplyforworkattheCollegeBoardand
writeSATsanytimesoon.

Donotjustgettheanswerandmoveon.Itsnotabouttheanswer;itsabouttheprocess.Youwanttobesure
thatyouunderstandinyourboneswhyyouarrivedattheanswerandhowitcouldhavegonewrong.Youre


sat.magoosh.com 23

probablygoingtoseeaquestionthattestsasimilarlogicalprocessonyouractualSAT,whetheritsinreading,
math,orwriting,somakesurethatyoutakeagoodhardlookatit.Afterthat,trytowriteasimilarquestion
yourselfusingslightlydi erentdetails.

Locateyourweakareasandfocusonthem

Ifyouvegottenafewfunctionquestionswrong,forexample,spendawholelotoftimejustrepeatingthatone
typeofquestion.Inordertodothis,youregoingtoneedtherightkindofmaterials,though.Unfortunately,the
O cialSATStudyGuidedoesntcutit,here,sinceitjustdoesnthavesectionsdevotedtospeci ctypesof
questions(likemathfunctions)thatyoucanfocuson.MagooshsonlineSATprepo eringsarebettersuitedfor
it.

Butevenifyoudonthaveawholelotofquestionsthatworkonthesameskills,youcanstillusethisfocusjust
repeatthosesamequestions.Iknow,Iknow,itsoundsdull.Butthelogicalprocessthatyougothroughtoget
theansweristhefocus.Byrepeatingtheorquestion(s)fromweektoweek,youcanhelpdrivehomethe
strategyyouneed,andthatstrategymayproveprettyhelpfulonyourSAT.


sat.magoosh.com 24

HowtoTakeaPracticeSAT(TheRightWay)

TheCollegeBoard,thecreatorsoftheSAT,releasesabookoftenpracticetests.Manystudentswiselyusethis
book,asitcontainsactualquestionsfromtheSAT.AssomebodywhoteachestheSAT,thebookisthebibleand
o ersthebestpracticeoutthere.
Simplygoingthrougheachtestwillnottranslatetoabetterscore.Butpracticingfull-lengthSATtestsis
invaluabletoSATyourstudyroutine.Getthemostyoucanoutofthistestbykeepingthefollowingfourpointsin
mind.

Buildafoundation rst

IfyouhavelittleexperiencetakingtheSAT,donotjustdiveintotheCollegeBoardbook.Learne ectivetipsand
strategiesfromPrincetonReviewandBarrons.

Timeyourself

OnceyourereadytotakeapracticetestfromtheCollegeBoardbook,dontgiveyourselfallthetimeinthe
world.Youwanttomakethisasclosetotherealtestaspossible.Sosetyourtimer,anddontgiveyourselfany
moretimethanthetimeallowedforeachsection.

Dontgetinterrupted

Nowthatyouvegotthetimergoing,dontstopthetimer.Also,donottakeabreakinbetweensections.
SimulatetheactualSATbysittingfortheentirethreehours(turningo youriPhonewillde nitelyhelp!).

Figureoutyourwronganswers

Afteryouvetakenatest,dontjustmarkquestionsrightandwrong.Trytounderstandwhyyoumissedcertain
questions.Thisprocesswillhelpyoubetterunderstandhowthetestwritersthinkandwillhelpyouavoidsimilar
mistakesinthefuture.

Ifyoucant gureout,IverecordedexplanationsonMagooshSATsYouTubechannelfortheSAT.Havealook!


sat.magoosh.com 25

SATScoring


sat.magoosh.com 26

ScoringOnTheRedesignedSAT

ScoringonthenewSAThasgottenbothsimplerandawholelotmorecomplicated.Butletsstartwiththe
simple rst.

Thetestisnolongeroutofatotalof2400points,andisnowoutof1600points.Readingandwritinghave
beenlumpedintoonesectionthatisoutof800.Themathsectionwillconstitutetheother800points.And
rememberhowtheessayusedtobepartofyourwritingscore?Well,nowtheessaywillbeoptionalandwillnot
beincludedinyourwritingscore.

Inlinewiththesimplicitymotif,guessingwillnolongerbepenalized.Thatsright,thepeskypenalty(pesky
becauseitwastedioustocalculateanditredoundednegativelyonyourscore)hasgonethewayofdinosaurs
andbell-bottoms.

Sohowhavethingsgottenmorecomplicated?Well,therewillbedi erentdomainsofknowledgethatwill
stretchacrossmultiplesections.Forinstance,yourabilitytointerpretdata,akathegraph,willbetestedacross
bothsections;someofthereadingpassagesandsomeofthewritingsectionswillhavequestionsdealingwith
graphs.Anotherdomainwilltestyourabilitytounderstandhowwordsfunctionincontext.Theideaisthat
collegeswontjustknowyourscore,butthattheywillalsohaveabreakdownofyourskillsacrossavarietyof
areas.

Tothebestofmyknowledge,allquestionswillbeweighedthesame.However,thenumberofquestionsyou
needtogetcorrectinordertogetaspeci cscore(atopicreferredtoasscaling)issomethingCollegeBoard
wontreleaseuntilmultiplegroupsofstudentshavecompletedthenewexam.Hopefully,thatinformationwill
giveusabettersenseofhowscoringwillworkonthenewtest.


sat.magoosh.com 27

AvoidScoreDrama

Obviously,studyingfortheSATisnopieceofcake.Buttheobnoxiousdramathatsurroundsthewholeprocess
canmakeitnearlyunbearable.Hereareourtoptipsforhowtoavoidcat ghts,gossipandmore:

Dontcheckscoresaroundyourfriends

Wecanadviseagainstthisfrompersonalexperience:tearsbecausesomestudentsgotlowerscoresthantheir
friends;guiltoverhigherscores.Itsalwaysabadscene.

Moralofthestory:makesureyouhaveaprivateplacetobewhenyoucheckscores.Anddontfeelpressured
toshareyourscorewithfriendsifyoudontwantto.

Dontjudgepeoplesresponsestotheirscores

Ifyou ndyourselfinasituationwhereyouhavetobearoundyourpeerswheneveryoneischeckingscores,try
tofocusonyourself.Dontworryabouttheirresponses,happyorsad.Everyoneisdi erent,soeveryonewill
havedi erentreactions.

Ifsomeoneisenthusiasticallyhappywithwhatyouthinkisalowscore,letthembe.Ifsomeoneissobbingover
ascorethatshigherthanyours,letthemsob.Thatsalsotheirprerogative.Youneverknowhowmuchworka
studentputin,orwhatkindofemotionalstresstheyareunder.

Dontspreadscoresaround

Tomany,SATscoresareprivateinformation.Justbecausesomeonetellsyoutheirscoredoesntmeanthey
wantittobecomecommonknowledgetothewholestudentbody.

Newsofvariousstudentsscorescanbeabigsourceofgossipinhighschool.Butyoushouldntcare!Itsother
peoplesbusinessandithasnothingtodowithyou.

Understandcontext


sat.magoosh.com 28

Itdoesntmakesensetocompareyourselftoothersbecauseeveryonepreparesforthetestdi erently.If
someonegotahigherscorethanyou,itmostlikelymeansthattheystudiedmorethanyou.Maybetheywere
abletoa ordtutoringthatyoucouldnt.Youneverknow.


sat.magoosh.com 29

SATWritingTest


sat.magoosh.com 30

IntrotoSATWriting

Whatyouneedtoknow

Therearefour,350-450-wordpassages,containingelevenquestionseach
Testsbasicgrammar
Also,testsmorebig-picturegrammaticalissues,i.e.transitionsbetweensentencesandtransitions
betweenparagraphs
Testsstyle,tone,andsyntax
Onepassagewillhaveagraphwithonequestion

BasicTipsforSATWriting

Overtheyears,Ivehadseveralstudentswhowouldalwayssurprisemewhenitcametogrammar.While
mastersatgamingthegrammarsectionoftheoldSAT,theyfailedtospotevensimplegrammaticalerrorsin
theirownessays.Butitwasntjustonthelevelofstrugglingto ndasubject-verbagreementissueintheirown
writings.Theyseemedtomissthelargerconnectionsthosebetweensentencesinaparagraphandthose
betweentheparagraphsthemselves.

ImguessingthattheCollegeBoardrecognizedthesameshortcomingsintheirapproach(okay,letsbehonest,
theACTrecognizedtheshortcomingsinsuchanapproach).Soinsteadofhavingisolatedsnippetsofgrammar,
grammarthatbecomesverypredictableifyouknowwhattolookforandthusdoesntreallydeterminegrammar
knowledge,thenewSATishavingstudentsmakecorrectionstoafull-lengthessay.

Thischangeishighlywelcome(atleasttoeducatorswhowantthetesttoactuallymeasurewhatitpurportsto).
However,itmakesthetestlesseasytogame.Youllactuallyhavetobeadecentwriteronewhocantellan
okaytransitionsentencefromafelicitousonetodowell.Thatdoesntmeantherearentsomebasic
techniquestohelpyoubecomebetteratthissection.

1.Toreadornottoread


sat.magoosh.com 31

400wordsarealottoread.Multiplythatby4andthissectionsuddenlyhas1,600wordsthatsnotincluding
the44questions.Soshouldyoureadtheentireessay?

Idontthinkthereisaneasyanswer.Myrecommendationistoexperiment.Firsto ,timeyourselfwhereyoudo
onepassagefollowedbytheelevenquestions.Seehowmanyyougetcorrectandalsogaugehowcomfortable
youfeltwiththematerialasyouwentthroughit.Next,trythesamewithoutreadingthepassageatall.

Myhunchisthatthemoresuccessfulapproachmightbedeterminedbythespeci cpassage.Thatis,some
passageshavemoreparagraph-centricquestions.Youmightonlyneedtoreadthatparagraphtogetthe
questionright,andnothavetoworryaboutreadingtheentirepassagefromtheget-go.Conversely,some
passagesmightaskyoutoanalyzetherelationbetweenparagraphsortransitionsbetweenparagraphs.Without
initiallyreadingtheentirepassage,youmight ndyourselfscramblingaround,tryingto gureoutwhattoread.
Oryoumighttotallyoverlookthesupportingcontextandthinkyougotthequestionrightbut ubitcompletely.

NewStrategy

Readaparagraphatatime

Re-readpartsofthatparagraphifnecessary

Ibelievethebestpossibleapproachistoreadaparagraphatatimeandthendothequestionsrelatingtothat
paragraph.Thatwayyoullbeabletoquicklygobackandcorrectthegrammarissuesyoujustencountered.

Youllalsobeabletomoreeasilyidentifyanyconjunctionissuesthatcomebetweensentenceswithouthaving
toreadtheentireparagraph(thoughyoullhavetorereadthespeci csentencesrelatingtothatquestion).

Finally,youllbeabletobetternoticethetransitionsbetweenparagraphs,incaseaquestionasksaboutthat.

2.Knowyourrules(akaStandardEnglishConventions)

Ultimately,thisisatestofgrammar.Soyoullstillhavetoknowthebasicrules.Whatseemsfaireraboutthistest
isthatifyouhaveaprettystronggraspofagrammaticalruleyoullmostlikelygetthequestion.Sure,there
mightbeatrickyquestionortwo,inwhichyoullhavetoreadtheprevioussentenceto gureoutthecorrect


sat.magoosh.com 32

answer,butitwontbeanythingliketheoldSAT.Ontheoldtest,youcoulddrillsubject-verbagreementor
pronoun-antecedentuntilyoureyeballspopout,butyoulikelywouldstillmissthedreadedquestion#29inthe
writingsection.

3.Knowthequestiontypes

Thenewwritingsectionisntallaboutgrammar.Youllhavetounderstandhowideasaredevelopedoverthe
courseofaparagraph(andsometimestheentireessay).Thetestwillaskyouwhetheritshouldaddinformation,
deleteasentence,ormoveasentencetoadi erentpartofaparagraph.

The rsttimeyouseesuchaquestionyoumightbe ustered.Bypracticingthesequestionsandlearningwhat


thetestisexpectingofyou,youwillhaveaneasiertimewiththissamequestiontypeinthefuture.And
rememberthisisastandardizedtest,sothewronganswersandthereasonstheyarewrongareoften
consistentovertests.Onceyoupickuponthesepatterns,youllbecomebetteratthequestiontype.

4.Importanceofcontext

Itistemptingtoapproachthenewwritingsectionbylookingdirectlyattheunderlinedpartandthefewwords
aroundit,andthencomingtoaconclusion.Thismightsometimesnotgetyouintrouble,ifthequestionhasto
dowithpossessivesorastraightforwardsubject-verbagreementquestion.Butwhendealingwithsentence
structureortransitionsbetweensentences,itisalwaysagoodideatoreadtheentiresentencebeforethe
underlinedpartandtheentiresentenceinwhichtheunderlinedpartiscontained.

Sometimes,youwillhavetochooseananswerchoicethatasksyoutoconsiderinformationfromtheprevious
paragraph.(Wellseesuchanexampleinthepracticepassageattheend.)Often,whenthishappens,thetest
writerswillreinforcetheimportanceofcontext.Comparethesetwo:

Whichchoicemoste ectivelycombinestheunderlinedsentences?

Incontext,whichchoicebestcombinestheunderlinedsentences?

The rstpromptwillfocusongrammaticalissueswithinthetwosentencesthatyouhavetocombine(basically,
youdontneedtoknowthesentencesthatcamebefore).Thesecondprompt,however,clearlystatesin


sat.magoosh.com 33

context.Soyoubettermakesureyouunderstandthesentencesandsometimeseventheparagraphthat
comebeforethecombinedsentencesthequestionisaskingabout.


sat.magoosh.com 34

StandardEnglishConventions

Knowingthesebasicstrategiesandbeingfamiliarwiththequestionformatwillhelpupuntilapoint.Youwill
stillneedtoknowyourstandardEnglishconventions.Thisformidable-soundingphrasebasicallymeans
grammar.ThegoodnewsisyoudonthavetoopenuponeofthosemassivegrammartomesyourEnglish
teachershavebeenfoistingonyousincethe6thgrade.

Belowarethecommongrammarissuesyoullseeonthetest.Reviewandunderstandthem.Andthenand
herestheimportantpartpracticebyapplyingwhatyouvelearnedasyougothroughanactualpassage.

IshouldalsonotethattheheadingsofthesectionsbelowSentenceStructure,ConventionsofUsage,
ConventionsofPunctuationallowsmetoorganizeallthedi erentconcepts.Youdont,however,haveto
knowthatparallelismfallsunderSentenceStructureandthatagreementfallsunderConventionsof
Usage.Yousimplyhavetounderstandtheconceptofparallelismandwhenitisbeingviolated,andthesame
foragreement.


sat.magoosh.com 35

SentenceStructure

WhatImgoingtodohereisshowyouabasicexample(soyougettheconcept)andthenamuchmore
advancedSAT-levelexample.Mylogicisasfollows:manystudentsunderstandtheconceptatitsmostbasic
level(whichusuallymeansbasicSee-Jane-runlevelsentences),butthesesamestudentsfailtospottheerror
whenthesentenceismoreadvanced.

Additionally,ImgoingtostructurethelayoutofthegrammarexactlythewaythattheCollegeBoarddoes.

1.SentenceBoundaries
2.SubordinationandCoordination
3.ParallelStructure
4.Modi erPlacement
5.InappropriateShiftsinVerbTense,Mood,andVoice
6.InappropriateShiftsinPronounPersonandNumber

ButunliketheCollegeBoard,ImactuallygoingtodelveintoImtalkingdeepdivewhateachmeansinan
SATcontext.

1.SentenceBoundaries

Fragments

Sentencesaremadeupofbothasubjectandaverbthattellsuswhatthesubjectisdoing.Theexceptionwould
becommands,whicharenttestedontheSAT(forexample,Study!).

Fragmentsareincorrectbecausetheylackaverbthatdescribeswhatasubjectisdoing.Butitsnotthat
straightforward,astheexamplesbelowshow.

ManystudentswithatestonMonday.

Correct:ManystudentswithatestonMondayarepreparingovertheweekend.


sat.magoosh.com 36


Withpropertraining,manyathletes.

Correct:Withpropertraining,manyathletesshouldbeabletoavoidinjury.

(Theboldedpartsaretheverbsofeachsentence.)

SAT-level

Dailyvitaminsandmineralsthatareimportantinhealthycellularfunctioning.

Inthiscase,thatbeginsarelativeclause,whichfunctionsasalargeadjectivedescribingthesubject.Theverb
thatispartofthisclause(inthiscaseare)shouldnotbeconsideredtheverbthatpairswiththesubjectto
createacompletesentence.

Correct:Dailyvitaminsandmineralsthatareimportantinhealthycellularfunctioningareinmanyofthefoods
weeat.

CommaSplices

Thereareseveralwaystoconnectcompletesentences.Themostobviousisbyusingaperiod.Youcanalsouse
asemicolonoracommaANDaconjunction.Theandisbig;thatswhyIputitincaps.Ifyouhaveasentence
madeupoftwoindependentclausesandacommawithoutaconjunctionconnectingthosesentences,youhave
acommasplice.(Ihaveboldedthepartthatshowsthecommasplice).

Incorrect:StudyingeverydayisnothowIwanttospendmysummer,Iwanttomakelastingmemorieswith
friends.

Correct:(usingaconjunction):StudyingeverydayisnothowIwanttospendmysummer,becauseIwantto
makelastingmemorieswithfriends.


sat.magoosh.com 37

Correct:(usingsemicolon):StudyingeverydayisnothowIwanttospendmysummer;Iwanttomakelasting
memorieswithfriends.

SAT-level

Incorrect:Jupiteristhelargestplanetinthesolarsystem,italsohasthelargestmoon.

Correct:Jupiteristhelargestplanetinthesolarsystem,anditalsohasthelargestmoon.

Correct:Jupiteristhelargestplanetinthesolarsystem;italsohasthelargestmoon.

2.Subordinationandcoordination

Therearetwowaysofapproachingthis,oneofwhichismuchmoreimportantfortheSAT.The rstwayisto
explorethedi erencebetweensubordinationandcoordination.Afterall,thatisthetitlethattheCollege
Boardhasgiventothisgrammaticalidea.However,gettingtangledupinthenuancesofthedi erencebetween
subordinationandcoordinationde ectsfromthepurposeofthetest:todeterminewhetheryoucantellthe
di erencebetweenatransitionbetweenclausesthatsuggestscontrast(however,nonetheless,onthe
otherhand),similarity(additionally,furthermore,moreover)andcauseande ect(because,therefore,
thus).

Contrast

Shepracticestenniseveryday,thoughsheisstillunabletohitasolidbackhand.

Eventhoughmanystudentsapplytoout-of-stateschools,theyendupchoosingalocalcollege.

Contrastwords:however,(even)though,although,nonetheless,notwithstanding,despite

Similarity


sat.magoosh.com 38

Climatechangeiscausingmanyheat-relateddeaths.Moreover,itisleadingtoconditionsthat,inthelongrun,
willharmusall.

Studentsfeeloverwhelmedwiththenumberofhoopstheyhavetojumpthroughtogettocollege.Likewise,
theyfeel ustered,oncetheygettocollege,bythemanydemandsoftheirnewenvironment.

Similaritywords:likewise,moreover,additionally,furthermore,also

CauseandE ect

Becauseoftuitionhikesattheprivateschool,manyparentsareoptingtosendtheirchildrenelsewhere.

Thelevelofcompetitionincollegesportshasbecome ercerthanever.Therefore,athletesandcoachesare
seekingevermoresophisticatedtrainingregimens.

Causeande ectwords:therefore,thus,because,so,since

Mini-quiz:

1.TheSAThashistoricallybeenthetestthemajorityofhighschoolstudentstaketoentercollege;additionally,
withmorestudentsoptingtotaketheACT,theSAThasbeenforcedtoalteritscontent.

A)NOCHANGE
B)however
C)therefore
D)furthermore

2.Themeantemperatureofoceanshasbeenrisingsigni cantlyforthelasttenyears;however,many
organismshavebeenforcedtomovefromtheirtraditionalhabitatsortosimplyperish.

A)NOCHANGE


sat.magoosh.com 39

B)moreover
C)therefore
D)nonetheless

Answersandexplanation:

1)B

Acontrastexistsbetweenwhathashistoricallybeenthecase(TheSAThasbeenthemostpopularexamfor
college-boundstudents)andwhatisnowthecase(TheACTisbecomingincreasinglypopular).Thispointsto
thecontrastword,B)however.

C)andD)aretempting.However,thattheSAThashadtoalteritscontentdoesnotresultfromthefactthatthe
SAThaslongbeenapopulartest;theSATalteringthetestisaresultoftheACTbecomingmorepopular.

2)C

Thereiscauseande ecthere:oceantemperaturesincreasing(cause)andorganismsmovingfromtheir
traditionalhabitat(e ect).SoC)thereforeistheanswer.

Aquicknoteonsubordination

Thereisstillanimportantideainsubordinationthatislikelytocomeuponthetest.Ifaclauseissubordinate,it
isadependentclause,ornotastand-alonesentence.Itdependsonsomething;thatsomethingisan
independentclause.

Becausehewastired

Thoughhewassmart

Theabovearebothdependentclauses.Theyneedanindependentclausetocompletethem:


sat.magoosh.com 40


Becausehewastired,hewasntabletostudyallthematerialbeforethetest.

Thoughhewassmart,henevercaredtostudy.

TheSATmighttestthisinthefollowingway:

AlthoughElNioistypicallyassociatedwithasharpspikeinannualrainfallontheWestCoast,thoughthere
areothermeteorologicalfactorsthatcano setthise ect.

A)NOCHANGE
B)WestCoast.Thoughthere
C)WestCoast,thereare
D)WestCoast.There

Sowhatsexactlywrongwiththissentence?Well,adependentclausedependsonanindependentclause,
whichisfancy-speakforacompletesentence.However,thepartthatcomesafterthedependentclause,which
beginswithalthough,alsostartswiththough.Whenaphrasestartswithalthough,though,despite,
because,etc.,itisadependentclause.Therefore,wehaveback-to-backdependentsclausesabigno-no.
Gettingridofthethoughgivesusacompletesentence(Thereareothermeteorologicalo set).AnswerC).

3.Parallelstructure

Therearetwothingsyoullwanttorememberwhenitcomestoparallelism:partsofspeechandlists.

Partsofspeechincludeadjective,noun,verb,etc.IfIlistseveralthings,thosethingsshouldbeinthesameform,
i.e.theyshouldsharethesamepartsofspeech.Forinstance,inthesentencebelow,thelistismadeupofthree
things:readmagazines,watchtelevision,andplayvideogames.

Incorrect:Georgelikestoreadmagazines,watchtelevision,andheplaysvideogames.

Incorrect:Georgelikestoreadmagazines,watchtelevision,andplaysvideogames.


sat.magoosh.com 41


Correct:Georgelikestoreadmagazines,watchtelevision,andplayvideogames.

Inthe rsttwoexamples,wehavetwoverbsthatareparallel(readandwatch).WhatImeanbyparallelis
theyhavethesameform:theyarenotreadsandwatchorreadandwatchesorwatching.

Notthatwecouldntusesomeotherformoftheverb.Butthethreepartsthatmakeupthelistmustbeinthe
sameform.

Correct:Readingmagazines,watchingtelevision,andplayingvideogamesarethreethingshelikestodo
duringhisfreetime.

Parallelismandcorrelativeconjunctions

Thereisaspecialtypeofconjunctioncalledthecorrelativeconjunction.Youdontreallyhavetoknowthename,
butyouhavetoknowthefunction.Morespeci cally,youhavetorememberwhenyouseetheseconjunctionsto
thinkparallelism.

Correlativeconjunctions:

NotonlyAbutalsoB
BothAandB
EitherAorB
NeitherAnorB

WhatintarnationdoesAandBmean?AstandsforawordorphraseandBstandsforawordorphrase.
Thesewordsorphrasesshouldbeparallel.Inotherwords,AandBshouldbeparallel.

Notonlyishefunny,butheisalsoclever.

InthiscaseAandBareadjectives.


sat.magoosh.com 42


SAT-levelexample:

Incorrect:Notonlyhashesquanderedanimportantopportunity,butheisalsoupsettingmanypeoplecloseto
him.

Correct:Notonlyhashesquanderedanimportantopportunity,buthehasalsoupsetmanypeoplecloseto
him.

Squanderedisinthesimplepasttense;therefore,weneedthesimplepasttenseofupset,whichisupset.
Noticeintheincorrectexample,squanderEDdoesnotmatchupsettING.

Onmoreadvancedparallelismquestions,itwontjustbetwowordsthathavetobeparallelbutentirephrases.
Othertimesaquestionisdi cultbecausetheverbsareburiedinamorassofwords,astheexamplebelow
shows.

SAT-level(hard):

Playingvideogames,unlikewatchingtelevision,isnotapassiveactivity,becausedoingsorequiresthatthe
videogameplayerreacttowhatshappeningonscreen,strategizestoovercomeobstacles,andthatshe
perseveretoadvancethroughthemostdi cultstagesofthegame.

1)
A)NOCHANGE
B)thatshestrategizes
C)thatshestrategize
D)strategize

2)
A)NOCHANGE
B)shepersevere


sat.magoosh.com 43

C)sheperseveres
D)persevere

AnswersandExplanation:

Thevideogameplayerhastodothreethings:react,strategize,andpersevere.Theverbformissomething
calledthesubjunctive,whichoftenpopsupinwordsthatindicatedacommand,request,orarequirement.You
dontactuallyhavetoknowthis.Butyoudohavetonoticethatitdoesnotsay,thevideogameplayerreacts;a
verbusuallytakesanstheendwhenitreferstoathirdpersonsubject(hewalks,shedances,etc.).

Herethoughitisreact,notreacts(again,thatsbecausewehavetheverbrequirethat,whichremovesthe
sfromtheendofaverbreferringtothethirdperson.Therefore,theothertwoverbsmustbothbeinthisform,
givingusstrategizeandpersevere.Therefore,theanswertoboth1)and2)isD).

4.Modi erPlacement

TheSAThasalwayslovedtotestthisconcept.Thereisaclearlogicalreasoningbehindthecorrectanswer,yet
manydisregardthisintheireverydayspeech.Andwhenwhatsoundsrightandwhatisactuallyrightcon ict,
youcanbettheSATiswaitingtherewithacarefullyengineeredquestiontotraptheunwary.

Sotakealookatthefollowingsentences:isanythingwrong?

Studyingfor nalsandplayinghighschoolsports,itishardformanyto ndtime.

LivingforsevenmonthsinMadrid,Marthas uencyinSpanishincreasedrapidly.

Notsosure?Well,whataboutthefollowingexamples?

Runningdownthestreet,abicyclehithim.


sat.magoosh.com 44


Flippingthroughchannels,thetelevisionsuddenlyturnedo .

Somethingweirdisclearlygoingon.Wasthebicyclerunningdownthestreet?Wasatelevisionsittingona
couchandeatingDoritos,while ippingthroughchannels?

Toavoidsuchabsurdscenarios,wehavetomakesurethatwhenwehaveaphrasebeginningwithaningverb
(calledaparticiple)thatthephrase,whichendsrightbeforethecomma,accuratelydescribethesubjectthat
comesrightafterthecomma.

Runningdownthestreet,hehadtojumpoutofthewayofanoncomingbicycle.

Flippingthroughchannels,Dexterthrewhishandsupinfrustrationwhenthetelevisionsuddenlyturnedo .

Nowletsgobacktothose rsttwoexamples.Canyouthinkofwaystoimprovethosesentences?Giveitashot.
Makessuretheingphrasecorrectlymodi esthesubject.Thecorrectversionofthesentencesisimmediately
below(dontpeek!)

Correct:
Studyingfor nalsandplayinghighschoolsports,manystudents ndithardtofocusonanythingelse.

Incorrect:
LivingforsevenmonthsinMadrid,Marthabecame uentinSpanish.

Modi cationisbasicallyafancywayofsayingdescribing.Andyoucanthinkofthe-ingphraseasalarge
adjectivethatshouldlogically(dontforgettheDoritos-eatingtelevision)describethesubject.

Sometimes,though,modi cationcomesafterthesubject.

JohnsatonthecoucheatingDoritosandwatchingablankscreen.


sat.magoosh.com 45

Johnsatonthecouch,eatingDoritosandwatchingablankscreen.

OneofthesesentencesimpliesthatthecoucheatsDoritos(whichisnttooabsurdifyoulookundersome
couches).Theotheriscorrectbecauseitdescribes(correctly)JohneatingtheDoritosandwatchingablank
screen.

Thecorrectsentenceusesacommatoseparatethephrase,Johnsatonthecouchfromthephrasethatsays
eatingDoritos.Whenthe-ingphrasecomesafterthecomma,theactioninthatphraseshouldalways
describethesubjectofthesentence.Inthiscase,Johnisthatsubjectofthesubject.

Ontheotherhandifyoudonthaveacommaseparatingthe-ingphrasefromtherestofthesentence,then
thatphrasemustlogicallydescribethenounthatcomesimmediatelybeforeit.Inthe rstsentence(theone
withoutthecomma),thereisnocommaseparatingcouchandeating.Therefore,thatsentenceimplies
(incorrectly)thatthecouchiseatingDoritosandwatchingablankscreen.

Mini-Quiz

Studentsmultitaskeveryday,indeedmanytimesaday,studentsbelievetheyareveryadeptatjugglingtwoor
threedi erentactivitieswhilestudyingforamidterm.Thoughtheymaywellbeabletolearnwhilemultitasking,
itisnotnearlyase cientasfocusingonlyonstudying.Yetmanystudentscontinuetopassupanoptimal
studyingenvironmentpreferringtomultitaskateveryopportunity.

1)
A)NOCHANGE
B)Tomultitask
C)Studentsmultitasking
D)Multitasking


2)
A)NOCHANGE
B)environment;preferring


sat.magoosh.com 46

C)environment,preferring
D)environment,theyprefer

Answersandexplanation:

1.Asis,thissentencehastwosubjects(studentsandstudents).Sincestudentsisalreadythesubject
oftheclausebeginningstudentsbelieve,itiseasytoaddadependentclause,speci callyan-ing
phraseandvoila!Wehaveavalidsentence.Answer:D).C)iswrongbecauseitalsorepeatsthesubject,
students.


2.Asis,thesentenceimpliesthattheenvironmentpreferstomultitask.Byputtingacommabetween
environmentandpreferring,thesentenceiscorrectlystructuredtoindicatethatpreferringrefersto
thesubject,manystudents.Answer:C).

5.InappropriateShiftsinVerbTense,Mood,andVoice

Verbtense

Consistency.Thatsthe rstthingyouhavetorememberaboutverbtense.Ifyouaretellingastoryaboutthe
timeinthe7thgradewhereyougotlosthikinginathunderstorm,dontsuddenlyshifttothepresenttense.

Whilethatmaysoundprettystraightforward,whenthetopicisnotthatfamiliarandtheSATistheonewriting
aboutit,wetendtolosetrackofthepropertenses.Sometimes,tensesalsoshiftfrompasttopresent,andvice
versa.Italldependsoncontext.

19thcenturymusicianshadgruelingpracticesessionstohelpthemmastertherigorsoftheirrespective
instrument.Today,instrumentalistsmustalsopracticemanyhoursaday.Somearguedthat,withthe
make-it-or-break-itcompetitionmusiciansmustfacetoday,practicesessionsaremoregruelingthanat
anyotherpointinhistory.


sat.magoosh.com 47

A)NOCHANGE
B)hadargued
C)wouldhaveargued
D)argue

Thecontextherecomparesmusiciansofthe19thcenturytomusicianstoday.Noticehowthepassagedescribes
the19thcenturymusiciansusingthepasttenseandhowitswitchestopresenttensetodescribetodays
musicians.Noticehowthepartbeingtested,argued,isinthepasttense.Notice,too,howthesomedoing
thearguingarediscussingpracticesessionstoday.Therefore,wewantpresenttense.AnswerD).

Thetestcangetalittlemorecomplicatedbyaskingyoutodi erentiatebetweenpresentperfectandpresent
tense.

Sincethe6thgrade,Ihavekeptadailydiary.

SATlevelexample:
Eversincetheadventofmovingpictures,directorshavebeenre ning lmtechniques.

Thetestwantsyoutobeawareofthefactthatwheneveryouhaveanactionthatstartedinthepastbut
continuestoday,youshouldusethepresentperfect.Tomakethatclearerthetestwillalmostalwaysthrowinthe
wordsince.

Mood

Moodisastrangewordchoicetodescribeagrammaticalsubtlety.Andforthatreason,Ithinkitturnsmany
studentso (andputstheminabadmood,asitwere).
Whatwemeanbymoodingrammar-speakiswhethertheverbisacommand(imperativemood),aquestion
(interrogativemood),orconditionality(subjunctivemood).

Imperativemood


sat.magoosh.com 48


Thewaythisisgoingtoshowuponatestisintheformofaverb.Commandwords,orwordssuchasrequest
that,orderthat,requirethatarefollowedbybe+verbparticiple.

Theteacherdemandedthatthehyperactivebeseatedtheentireclass.

Herequestedthatshebepresentatthemeeting.

Subjunctivemood

Ingeneral,moodissorareonthetestthatIthinkitshowedupinonequestionofthefourpracticetestsinthe
CollegeBoardbook.Soifyouhavetoskiponesectionofthegrammarreview,thismightbetheone.

Thesubjunctivemoodimpliesconditionality,inotherwordsahypothetical,i.e.,somethingthatisntandcantbe
reality.Toshowthis,wechangetheverbwastowere.Iftheverbisalreadywere,thenwejustleaveit
(Werewespacealiens,wecouldtravelthecosmos).

Weresheresponsibleforthebreak-in,shewouldnothaveanairtightalibi.

IfIwerepresidentoftheUnitedStates,IwouldmaketacoTuesdaysanationalholiday.

Bothoftheseexamplesareinthesubjunctivemoodbecausetheydescribesomethingthatcouldntpossibly
be.Itslikesayingimagineif.(Basically,ImimplyingthereisnowayIdeverbepresidentoftheUnited
Statessorry,notacoTuesdays).Whensomethingispossible,thenyoudontneedtochangetheverbto
were.

NoticehowIdidntmentiontheinterrogativemood.Thatsbecausenothingchangeswiththeverb;therefore,
theresnoeasywayfortheSATtotestthattheinterrogativemoodisbeingused.


sat.magoosh.com 49


Inappropriateshiftsinvoice

ThroughoutthiseBook,Ihaveusedtwolevelsofspeakingwhencomingupwithexamplesentences:Onethat
iscasualandchummy(thetacoTuesdaysvoice)andanotherthatismoreformal(theSATvoice).TheSAT
wantstomakesureyouknowwhentousetheappropriatelevelofvoice.Thatis,youdontwanttosay
somethinginacasuallaxwaywhenwritinganessay,or,inthiscase,editinganessaythattheSATisprovided.

Inthesentencepairsbelow,onemaintainsaconsistenttonewhereastheotherlapsesintocasualspeak.

1a)TheSATplacesharshdemandsonastudentsattention,forcingthemtobefocusedforalmostfour
hoursstraight.

1b)TheSATisreallyhardbecauseyouhavetokeepyourmindonthetaskforfourstraighthours.

2a)Workingingroupscanleadtohighemployeeproductivitywhilemakingpeoplefeelgoodabout
workingwithothersintheo ce.

2b)Workingingroupscanleadtohighemployeeproductivitywhilefosteringgoodwillbetween
colleagues.


Answer:1band2aaretoocasualforthelikesoftheSAT.

Languagecanalsobetoostilted,orformal.TheSATandanymanualofstyle(andgoodsense!)willtellyouto
avoidoverlyformallanguage.Canyouspottheo enderbelow?

3a)TheSATishighlyonerousonmindsexhibitingapropensityfordivagation.

3b)StudentswhoareseriousaboutSATpreparemorelikelytoseekoutsidehelp,andarethusmore
likelytoseeascoreimprovement.


sat.magoosh.com 50

Ifyouarenotevenquitesurewhat3a)issaying,youarenotalone.Itusesoverin ated,pompouslanguage.

Thereason,Ithink,theSATistestingthisisbecauseamongststudents(andyoumightevenbeoneofthem)
theresanotionthatsprinklingfancy-soundingSATwordsintoyouressayisamarkofgoodwriting.That
thinking,however,isagrossmisconception.See,thereisamajordi erencebetweenthediscernmentusedin
choosingaparticularlyaptwordandthedesperationtosoundintelligentbyusingwordslikedivagation(dont
worrythatwordwillnotbeonthetest!).Thelatterwillconfuseyourreader,muddywhatyouaretryingto
communicate,andultimatelyturnthereadero fromwhatyouhavetosay.

Mini-Quiz

Thereis,ofcourse,adarksidetovideogames.Likeanyactivitythatisbasedonasystemofrewards,video
gamescancausepeopletooverindulgeandneglectotheraspectsoftheirlives,allsotheycanreachthenext
levelintheirfavoritegames.

A)NOCHANGE
B)stopdoingotherstu
C)ceaseintheengagementwithothersigni cantlifeactivities
D)failtoattendtourgentfacetsoflife

Explanation
B)istoocasual.BothC)andD)aretooformalandwordy.A)isconsistentwiththetoneandlevelofformality
foundintherestofthesentence.

6.InappropriateShiftsinPronounPersonandNumber

EverytimeyouseeapronounontheSATthatpronounshouldbeclearlylinkedtoanoun.Thatnouniswhatwe
calltheantecedent.Typically,itllcomebeforethepronounbutnotalways.Themainpointisyoudonthavea
mysterioushe oatingaroundinaparagraph.TheheshouldclearlyrefertoJim,Bob,orwhatevermaleyou
arewritingabout.


sat.magoosh.com 51

Jimwasvotedclasspresidentmainlybecauseheisverypopularamongstthestudentbody.

SAT-level

ThoughEinsteinislaudedforhisgenius,hewasntparticularlyadeptatmathematicsand,tosupporthis
morecomplextheories,heoftenreliedonhismorecomputationallymindedpeerstocrunchthe
numbers.

Number

Inthelastsentence,youcanseethatheandbothofthehispronounsrefertoEinstein.Thisuseofpronouns
shouldbeprettystraightforwardtomost.Thingsgetalittlemorecomplicatedwhenwedropthewordnumber
inthere.Numberasfarasverbsandpronounsgo,referstoeithersingularorplural.

Forinstance,thenumberofIissingular;thenumberofweisplural.However,mostofusareunlikelytomix
thosetwoup.TheSATwillmostlikelytestyoubytakingapluralsubjectandusingtheitpronountorefertoa
pluralsubject,orviceversa.Seeifyoucanspottheerrorbelow.

Bobbyforgottodoseveralassignmentsbutturneditinlater.

Inthissentence,whatdoesitreferto?Commonsensetellsustheassignments.Indeed,wespeakthisway
andwouldunderstandexactlywhatthepersonissaying.However,inwriting,youhavetomakesurethatthe
pronounmatchesthesubjectintermsofnumber.Inthiscase,assignmentsisplural,thereforeweneedthe
pluralpronounthem(somestudentsareoftensurprisedtoseethemrefertonon-humansubjectsand
abstractnouns,thoughthisusageis100%correct).

Correct:Bobbyforgottodoseveralassignmentsbutturnedtheminlater.

Bytheway,ifyouarestillnotconvincedandthinkthatitreferstohomework,whichissingular,rememberthat
thenounhomeworkactuallyhastoshowupsomewhereinthesentence.

Inappropriateshiftsinperson


sat.magoosh.com 52


TheSATlikestotrickyoubyswitchingfromthethirdperson(one)tothesecondperson(you)orviceversa.
Alwaysrememberthatthecorrectpronounistheonethatappearsinthenon-underlinedpart.

Togiveyoualittlerefresher,hereisatableshowingthedi erentnumberandpersonofpronouns.

1stperson 2ndperson 3rdperson

Singular(subject) I you he/she/it

Singular(object) me you him/her/it

Plural(subject) we you they

Plural(object) us you them

Mini-quiz

Wehaveallreceivedthem:emailsclaimingthatwehavewonorinheritedalargesumofmoney.Whilemostof
usseetheseemailsforwhattheyareutterscamsasmallpercentageareluredin,believingthattheyare
indeedtherecipients.YetInternetscamsarenotalwayssoobviousandso1)oneneedstobeonguardagainst
farsubtlerformsofonlinedeception.

1)

A)NOCHANGE
B)youneed
C)weneed
D)theyneed


Oftenascammerwillposeasalegitimatecompany,sendinganemailthathastheinsigniaandbrandingofthat
company.Calledphishing,thismethodofextortingcon dential nancialinformationfromonlineusersisonthe


sat.magoosh.com 53

rise.Indeed,ifonelooksatrecentemails,theyarelikelytocomeacrossanemailthatseekstophish.Of
course,youprobablydonotevenbothertoopensuchanemailinthe rstplace,becauseitseemslikejunk
mail,ormassemail.


2)

A)NOCHANGE
B)heandsheis
C)youare
D)oneis


3)

A)NOCHANGE
B)weprobablydo
C)theyprobablydo
D)oneprobablydoes

Answersandexplanations:

1)Fromtheverybeginningoftheparagraph,thepronounweisused.Thereisevenanus,signalingthatwe
areusingthethirdpersonplural.Youisthesecondperson.Wecantjustchangethepronounoftheaudience
weareaddressing.Whicheverpronounisnotunderlineddeterminesthepronountheessayshoulduseto
addressthereader.AnswerC).

2)Heretheauthorusesthethirdpersonsingularpronounone.Tokeepthisconsistencythe rstunderlined
partshouldbeoneis.AnswerD).

3)Herethepronounchangestoyou.Again,keepitconsistentwiththepronounone,whichappearsinthe
non-underlinedpartoftheparagraph.AnswerD).


sat.magoosh.com 54

ConventionsofUsage

1.PronounClarity

Seeifyoucan gureoutwhatswrongwiththefollowingsentences.

NancyandMarytookhermothertothemovies.

GarygaveDavehislaptop.

Inthe rstsentence,whodoesherreferto?IsitNancyorMarysmotherwhoisgoingtothemoviewiththem?

Inthesecondexample,arewetalkingaboutGarysorarewetalkingaboutDaveslaptop?Thehisdoesnttell
us,soitisanambiguouspronoun,meaningitcanlogicallyrefertomorethanonething.OntheSATthisisabig
no-no.

Thoughthiserrorisntascommonaspronounissueswithnumberandperson(seeprevioussection,in
StandardEnglishConventions),itdoescomeupsometimes.

SAT-levelexample:

Testsonhumansandlaboratoryratscanyielddramaticallydivergentresults.Whentheydo,peopleare
boundtonotice.Butinthosemanyinstancesinwhichtheyhavetestedexperimentaldrugsonbothratsand
humans,andtheresultsarethesame,fewtakenotice.

The rsttheyshouldclearlyrefertoapluralnounintheprecedingsentence.However,therearemany
pluralnounsinthe rstsentence:test,humans,rats,results.Therefore,thetheyispotentiallyconfusing.So,
insteadoftheyweshouldrepeatthesubject:thetests.Notethatifthesentencehadcontainedonlyoneplural
noun,theywouldhavebeenokay,especiallyifthesentencewereshorterand/orthepluralnounwerecloser
tothepronounthatreferstoit.i.e.,Clinicaltestscanyieldusefulinformation.Wheytheydo


sat.magoosh.com 55

Thesecondtheyisconfusinginadi erentwayfromthe rst.Whereasthe rstwasdebatablyunclear,


meaningmostwouldconcludethatlogicallyspeakingtheycanonlyrefertotests,thesecondthey,whichis
closesttothepluralsubjectpeople,createsanunintendedmeaning.Itimplies(absurdly)thatpeopleingeneral
aretestingexperimentaldrugsonhumansandrats.Itisexperimentersingeneral.Agoodideaistoreplacethe
secondtheywithexperimentersorscientists.

2.Possessivedeterminers

Withoutanycontext,doyouknowthedi erencebetweenthefollowing:

Itsvs.Its

Theirvs.Theyrevs.There

Yourvs.Youre

Ifnot,hereisthebreakdown:

Its=Itis(Itsgoingtorain.)
Its=possessivepronoun(Thedoghaditstailinitsmouth).

Their=possessivepronoun(Theirfriendsarethesnootiestofthebunch).
Theyre=theyare(TheyrethesnootiestbunchIveevermet).
There=place(Iamgoingovertheretostudy).

Your=possessive(Yourfriendistotallystuck-up).
*Youre=youare(YourethesnootiestpersonIknow).

*Soremembertoalwayswriteyourewelcome.Yourwelcomeimpliesthatyoupossessthewelcome,youve
outlivedyourwelcome.


sat.magoosh.com 56


Mini-quiz

Workersgivenfreedomtosettheirownagendastendtobehighlyproductive.Accordingtoseveralrecent
studies,however,1theirthemostproductivewhentheynotonlydeterminetheirownagendabutalsowhen2
theyreisadailysystemofaccountability.3Itssomethingbackedupbystudyandpractice,theideathat
workersaremostproductivewhenthereissomeoversightbutwhenworkersgettoownourprojects.

1)
A)NOCHANGE
B)there
C)theyre
D)theirs


2)
A)NOCHANGE
B)there
C)theres
D)theirs

3)
A)NOCHANGE
B)Its
C)Theyre
D)Theres

Answers:
1)C
2)B
3)A


sat.magoosh.com 57

3.Agreement

TheSAThasalwayslovedagreementitseasytotestanditseasytomakethequestiondi cult.But rstlets


talkaboutwhatagreementis:itiswhenthesubjectandverbareconsistentintermsofnumber.

Wevetalkedaboutnumberinthecontextofpronouns.Itsnodi erenthere,inthesensethatnumberrefersto
whetheranounissingularorplural.Thedi erenceistheverb.Someverbswilltakeansattheenddepending
onwhetherthesubjectissingularorplural.

Hewatchesmanymovies.

Theywatchmanymovies.

Iwatchmanymovies.

TheSATismoreconcernedwithabstractnounssuchastheanalysis,theobservation,thedescription.Allof
thesenounsaresingularandcorrespondtothethirdpersonpronounit.

TheanalysisshowsthatTimisthebettersoccerplayer.

ItshowsthatTimisthebettersoccerplayer.

Noticethesinshow.Thisthrowsalotofstudentso at rst.Ifsomethingisplural,theythink,shouldntithave
ansattheendofit?Well,thatsomethingisthesubject.Theswearetalkingaboutcomesattheendofthe
verb.Andverbsarenotsomethingthatyoucanpluralize.

Granted,itisstillalittleconfusing.Butrememberthatthe-sonlycomesattheendoftheverbifthe
subject/nounissingular(he,she,it,orsomeabstractnounthatissingular).

IfyouvegottenwhatIvesaidsofarandarethinking,whatssohardaboutthat?TheSAThasalittletrickupits
sleeve.

TheanalysisofthetwosoccerplayersshowthatTimisthebetterplayer.


sat.magoosh.com 58


Whatsthesubject?

Manystudentsthinkthatthesubjectistwosoccerplayers,whichispluralandthatshowistherightanswer.
However,twosoccerplayersisnotthesubject.Thesubject,atleastontheSAT,willalwayscomebeforethe
rstpreposition(wordssuchasof,in,on).Noticeinthesentenceabovetheofthatcomesafteranalysis.
Thesubjectwillalwayscomebeforethatpreposition.

Takeanotherstabtoseeifyoucan ndthesubject.

Researchintothehabitatsofmeerkatsshowthattheanimalishighlysocial.

Whatswrongwiththesentence?

Well, rst gureoutwhatthesubjectis.Remembertolookatthe rstpreposition.Ifyouspottheof,dontthink


thatsthe rstprepositionbecauseitwassointhepreviousexample.Gobackfurtherinthesentencetointo,
whichisapreposition.Therefore,thesubjectitwillcomebeforethe rstprepositionisresearch,whichis
singular.Therefore,showshouldbeshows.

Seeifyoucan gureoutwhatthesubjectsinthefollowingsentencesare:

Amasteryofcardiopulmonarytechniquesandotherlifesavingtacticspotentiallyturnsanaverage
personintoaninstanthero.

Thenumberofstudentswhoarepursuingpostsecondaryeducationisincreasingannually.

Inthe rstsentence,thesubjectismastery;inthesecondsentence,thesubjectisnumber.Botharesingular
sothemainverbofthesentencetakesansattheend(turnsinthe rstsentence;isinthesecond.
Remember,thatansattheendoftheverbindicatesasingularsubject.Inthecaseoftobe,thatsnot
necessarilythecase).

BynowyoumighthavenoticedhowtheSATmakesagreementquestionsdi cult:itincreasesthedistance
betweenthesubjectandtheverbthatreferstothatsubject.


sat.magoosh.com 59


Toillustratethis,Iveeliminatedallthewordsthatcomeinbetweenthesubjectandtheverb:

Amasteryofcardiopulmonarytechniquesandotherlifesavingtacticspotentiallyturnsanaverage
personintoaninstanthero.

Thenumberofstudentswhoarepursuingpostsecondaryeducationisincreasingannually.

Tobecomestrongatthesequestiontypesthatiswhatyouhavetodo:Quicklyignoretheextrawordsandhome
inonthesubject,whichcomesbeforethe rstpreposition.

4.Frequentlyconfusedwords

UnlikemanyoftheothersectionsIvegonethroughthusfar,Icantshowyouthelogicoraquickruleortwothat
youneedonlyapplycorrectlytodowell.Knowingthede nitionsofwords,andthoseofwordsthatlooksimilar,
takesalotofpractice.

Luckily,therearetwopiecesofgoodnews:thisquestiontypeisntverycommonandtherearelistsofcommonly
confusedwordsallovertheInternet.Hereareafewofthegoodones,thoughIencourageyoutotypein
commonlyconfusedEnglishwordsinordertogetthefullymonty.

exceptvs.accept
precedevs.proceedvs.proceeds
a ectvs.e ect
consciencevs.conscious
allusionvs.illusion
principlevs.principal

Basically,IwouldfocusonotherSATgrammarconceptsbeforemovingontothis.Butifyouareeverunsure
howtospellawordbecauseitsoundslikeasimilarword,makesuretolookupbothwordssoyouknowthe
di erence.Whoknows,thatverywordpairmayshowuptestday.


sat.magoosh.com 60


5.Logicalcomparison

Aneasywaytothinkofthisisyoumustcompareapplestooranges.Youcantcompareanorangetothetaste
ofanapple,orthetasteofanorangetoanapple.Youcan,however,comparethetasteofanorangetothatof
anapple.Thethatreferstotaste.Insteadofrepeatingtaste,anounalreadymentionedinthesentence,we
usethepronounthatof(forsingularnouns)orthoseof(forpluralnouns).

TheSATisveryparticularaboutthisrule,soeventhoughitsclearwhattwothingsarebeingcompared,you
mustalwaysbeasliteralaspossible:

Incorrect

Accordingtosome,theuseofsrirachaasacondimentwillsoonsurpassmustard.

Correct

Accordingtosome,theuseofsrirachaasacondimentwillsoonsurpassTHATOFmustard.(Weare
comparingtheuseofsrirachatotheuseofmustard).


Incorrect

MozartspianoworksaremucheasiertolearnthanChopin,areasonwhymanybeginnerswillknow
howtoplayapiecebyMozart,butnotonebyChopin.

Correct

MozartspianoworksaremucheasiertolearnthanTHOSEOFChopin,areasonwhymanybeginners
willknowhowtoplayapiecebyMozart,butnotonebyChopin.(thoseofreferstoworks).


sat.magoosh.com 61

6.Conventionalexpression

TheCollegeBoarddescribesconventionalexpressionasthefollowing:

ConventionalExpressionquestionsdont tneatlyintooneoftheusagetypeslistedearlier,butlikethemtheyfocuson
recognizingandcorrectinginstancesinwhichwordchoicedoesntconformtothepracticesofstandardwrittenEnglish.

ThissoundslikeE ectiveLanguageUsetome,andunlessCollegeBoardgivesanexamplethatdi erentiates


betweenthesetwo,Iwouldntworryaboutthedistinction.Thisquestionwillbasicallyaskyoutochoosethe
rightwordintermsofitsprecisionandlevelofformality.


sat.magoosh.com 62

ConventionsofPunctuation

1.End-of-sentencepunctuation

Forthe rstinourlistofconventionsofpunctuations,thisoneisprettyobscureandnotthatlikelytoshowupon
thetest.Soifyouskipthisanddontrememberanythingofit,itsnotthatbigofdeal(makesurethatyouare
payingattentiontotherestofthislist).

Sohereitis:whensomebodyasksanindirectquestion,youdontwanttoendthatsentenceinaquestionmark.
Whatisanindirectquestionyouask?Takethefollowingsituation.

Mark:IsPattygoingtothemovieswithus?
John:Idontknow.IllguessIllaskherwhenIseehernext.

(Laterthatday)

John:Hey,Patty,Markaskedifyouaregoingtothemovieswithus.
Patty:No,youguysarelosers.

NoticethatwhenJohnasksPattywhethersheisgoingtothemoviesthatsentencedoesnotendinaquestion
mark.Andthatsit.Again,veryunlikelyyoullseethisonthetest.

2.Within-sentencepunctuation

Anotherwaytothinkofthisis,doyouknowwhenitisappropriatetousesemicolons,colons,andem-dashes.
Yes,thedreadedsemicolonIhadtogettoiteventually.Thegoodnewsisthatthetestisnotaskingyoutouse
asemicoloninyourwriting;itsaskingyoutorecognizewhenasemicolonisappropriate(asitwasinthemiddle
ofthissentence).

Belowisaquickbreakdownofeach.AndIpromise,semicolonsarenotasbadasyouprobablythinktheyare.


sat.magoosh.com 63


Semicolon

Thebubonicplague,publicspeaking,nothavingyoursmartphoneonatransatlantictripfewthingsareas
dreadedinlifeasthesemicolon.Butthereisnoreasontofearthisoft-malignedpunctuationmark,especiallythe
wayitisusedontheSAT.Itreallyisprettystraightforward:

Itisusedtoseparatetwoindependentclauses.

Thatsit.Thatisallyouhavetoknow.Sure,theessenceofthesemicolonisthatthetwoindependentclauses
arecloselyconnected,andsoitwouldntmakeasmuchassensetouseaperiod.Ofcoursethatbecomesa
judgmentcallandisatleastpartofthereasonthatmanyfeeluncomfortableusingasemicolon.ButtheSATis
onlygoingtoaskyouaboutwhetherornotasentencecorrectlyusesasemicolon;itisnotgoingtoaskyouto
writeasentenceusingasemicolon(LikehowIdroppedthatsemicoloninthere?)

Soletsputyoutothetest.Whichsentencescorrectlyuseasemicolon?

1)Formyupcominghike,Ipackseveralthings;acompass,a rstaidkit,awaterheatingcontraption,
andapocketknife.

2)Ivisitedseveralcollegesduringmysenioryear;noneofwhichIliked.

3)IwashopingtoworkinthePeaceCorpsaftergraduating;Iwashopingtomakeadi erence.

4)Mostsmartphoneusersoperateundertheassumptionthatsmartphoneswillsomehowmakethem
smarter;yetbecauseexcessivecellphoneusediminishesmanybasiccognitiveabilities,suchas
workingmemory,thisisnotnecessarilythecase.

Answersandexplanations:

1)Incorrect

Thissentencerequiresacolon(seecolonsection),notasemicolon.


sat.magoosh.com 64


2)Incorrect

noneofwhichIlikedisnotanindependentclause/legitimatesentence.

3)Correct

Theclausestoeithersideofthesemicolonarebothindependentclauses.

4)Correct

Thisisameatysentence,aboutascomplexasanythingtheSATwillthrowatyou.Thoughtheclause
immediatelyfollowingthesemicolonstartswiththewordsyetbecause,itispartofacompletesentencesince
itislinkedbyacommawiththeindependentclausethisisnotnecessarilythecase.

Colon

Thecolonhastwomainuses,oneofwhichwelearningradeschool.Ifyouaregoingtodescribealistofthings
youuseacolon.

Iwantmanythingsinlife:anew atscreentelevision,anexercisebike,amassagechair,avideogame
system,andyearsupplyofOreocookies.

TheSATknowsthatmoststudentsarefamiliarwithabasiclistofthingsfollowingacolon,sotheylltrytojazz
thingsupbyaddingalistofcomplexclauses.

AssoonasSandrasteppedoutsidetheairportsheknewshewasinaforeignland:thesmellofmangos
waftedinthelightbreeze,theshoutsoffoodcartvendorspunctuatedthehumidair,andalanguage
sherecognizedonlyfromRosettaStonecoursesranginherears.

Thecoloncanalsobeusedtoseparatetwoclauses,thesecondofwhichelaboratesonthe rst.Iamnowgoing
toillustratethis:hereisasentencethatusesacolon.Youcanthinkofthe rstpartannouncingthatitisgoingto
tellussomethingandthesecondclauseisthisthing.


sat.magoosh.com 65


Shewantedonlyonethinginlife:tobecomeseniorclasspresident.

Thisuseofthecolonisgoodtoknowfromawritersstandpoint;IdontthinktheSATwillexplicitlytestit,though.
Butitisgoodtoknowjustincase.

Em-dashes

WhenI rstmentionedem-dashesaminuteagoandyouwerethinkingImnotworriedaboutthesemicolon,
whatthe%$&isanem-dash,well,youvecometotherightplace.

Ifyouseeanem-dashmid-sentence,ithasoneoftwofunctions.

1.Alistofthingsmid-sentence

Usually,weseealistofthingsafteracolon.Butifalistcomesmid-sentence,wewanttosetito using
em-dashes.

Therearesomepunctuationsmarkscolons,semicolons,em-dashesthatscarethecrapoutofpeople.

Noticeinthelist,Ididnotuseanandtoconnectthe2ndand3rdnoun,thewayyouusuallywould(hehates
usingcolons,semicolons,andem-dashes).IdontthinktheSATisgoingtoexplicitlytestyouonthis
convention,butitisgoodtokeepinmindsinceyouwanttoeliminateacorrectanswerusingem-dashesjust
becausethereisnoandbetweenthelastandthesecond-to-lastitem.

2.Itemphasizesaparentheticalstatement

Sentence#1-Shehelpedhimasmuchasshecould(shepassedupgoingtoseeTaylorSwiftinconcert),
andyethemademoredemandsonhertime.


sat.magoosh.com 66

Sentence#2-ShehelpedSylvesterasmuchasshecouldshepassedupgoingtoseeTaylorSwiftin
concertandyethemademoredemandsonhertime.

Thedi erencebetweenthesetwosentencesandthereasonthatsentence#2isprobablywhatthewriteris
goingfor(thoughbotharegrammaticallycorrect)isthatsentence#2emphasizeshowmuchshegaveupto
helpSylvestershemissedtheTaylorSwiftconcert.Werethatnotthatbigofadeal,justapassingthoughtthe
writerwantedtoslipinthere,thenSentence#1wouldbe ne.

Luckily,thetestisnotgoingtoaskyoutochoosebetweenem-dashesandparentheses.Youjusthavetomake
surethatifanem-dashisusedtoemphasizeathoughtthatitisseto bytwoem-dashes:oneatthebeginning
ofthephraseandoneattheend.

Correct:

Haroldconsumedlotsoffoodfourhotdogs,nineslicesofpizza,acheeseburgerbeforepassingout.

Incorrect:

Haroldconsumedlotsoffoodfourhotdogs,nineslicesofpizza,acheeseburger,beforepassingout.

Haroldconsumedlotsoffoodfourhotdogs,nineslicesofpizza,acheeseburger;beforepassingout.

Haroldconsumedlotsoffood;fourhotdogs,nineslicesofpizza,acheeseburgerbeforepassingout.

Haroldconsumedlotsoffood,fourhotdogs,nineslicesofpizza,acheeseburgerbeforepassingout.

Thebasicrule:ifyoustartwithanem-dashmid-sentence,makesuretouseanotherem-dashmid-sentence.

3.Possessivenounsandpronouns


sat.magoosh.com 67

Weallknowhowapostropheswork.Whatthetestisgoingtotrytotrickyouwithissingularvs.pluralcases,and
viceversa.

Correct:Theteamsvictorywasunexpected,eventotheplayersmostdiehardfans.

Incorrect:Theteamsvictorywasunexpected,eventotheplayersmostdiehardfans.

Inthesecondsentence,noticehowteamismadeapluralpossessive.However,thereisonlyoneteam.Its
victoryiswhatisbeingmentioned.Also,notehowplayersbecomesplayers,whichissingular.Obviously,a
teamiscomposedofmanyplayers,notjustone

4.Itemsinaseries

WhenasentenceliststhreethingsA,B,andCthesethingsarecalleditems.

Shelikestoeatcookies,icecream,andcottoncandy.

A=cookies
B=icecream
C=cottoncandy

TheconventionisthatthereisacommabetweenAandB,andBandC.Interestingly,thecommabetweenB
andC,calledtheOxfordcomma,isnotnecessary.NolessthanthefancypantsnewspaperTheNewYorkTimes
hasdispensedit,sothatasentencecontainingthreeitemswilllooklikethis:

Shelikestoeatcookies,icecreamandcottoncandy.

TheSATdoesntcareaboutthedi erence;atleastitwonttestyouonit.Whatitdoescareaboutislongerlistor
liststhatcontaincommasintheirclauses.Withallthosecommas oatingaround,themeaningmightget
muddied.Onewaytodealwithallthesecommasistoseparatetheclausesbyasemicolon.


sat.magoosh.com 68

IgrewupwithcopiesofDickens,Thackeray,andTolstoysittingonshelves;un nishedliteraryessaysand
snippetsofinscrutablepoemsgracingthekitchentable;andceaselessliterarychatterechoinginthehallways.

A=Dickens,Thackeray,andTolstoy
B=un nishedliteraryessaysandsnippetsofinscrutablepoems
C=ceaselessliterarychatterechoedinthehallways

Speakingofinscrutable,bynotusingasemicolontobreakupA,B,andC,wegetthismess:

IgrewupwithcopiesofDickens,Thackeray,andTolstoysittingonshelves,un nishedliteraryessaysand
snippetsofinscrutablepoemsgracingthekitchentable,andceaselessliterarychatterechoinginthehallways.

Sometimes,commascancometotherescuewhenyouhavemuddledmeaning.

5.Nonrestrictiveandparentheticalelements

Thenitty-gritty

Thisisascarytitle.SoIwanttomakeitabiteasiertounderstand.

Comparethefollowingtwosentences:

MybiologicalMom,wholivesinSeattle,isvisitingthisweekend.

MybiologicalMomwholivesinSeattleisvisitingthisweekend.

Oneofthesesentencesisimpossible.Thatsright!Eventhoughthesentencesareidenticalwordforwordthey
arecompletelydi erent.Andthatsallduetoonelittlecomma.Compare:

MybiologicalMom,who

MybiologicalMomwho


sat.magoosh.com 69


Inthe rstfragment,wearede ningMom.Sheistheoneandonly.AndshehappenstoliveinSeattle.

Thesecondfragment,bydroppingthecomma,issayingthatoutofallmybiologicalmoms,Iamspeci cally
talkingabouttheonewholivesinSeattle.Butthatisabsurd.Everybodyhasalwayshadoneandonlyone
biologicalmom.Thereisnoneedtospecifywhichbiologicalmom.However,thatiswhathappenswhenwe
removethecommabetweenMomandwho:wearespecifyingwhichofyourmoms.

Ifwewanttospecifywhichsubject,wedontuseacommatoseparatethatsubjectfromthatorwho.The
phrasethatfollowsthewhoiscalledanessentialorarestrictiveclause,(Yes,Iknow,itstotallyannoyingwhen
grammarianshavetwotermsbothofwhicharetechnicalsoundingtodescribethesamethingandthenuse
thetermsinterchangeably.Forsimplicitysake,Illsticktothetermsrestrictiveandnonrestrictive,thewaythe
testdoes).

Comparethefollowingtwosentences:

Mydogthatsleepsoutsidehasanastycaseof eas.

Mydog,whichsleepsoutside,hasanastycaseof eas.

Unlikethemomsentences,bothofthesesentencesarepossible.Butoneofthemisimplyingyouhavemore
thanonedog.Rememberwhenyoudonothaveacommabetweenthesubjectandthewordwhoorthat,
youarespecifyingwhichsubject.Therefore,the rstsentenceisspecifyingthedogthatsleepsoutsideandis
implyingthatyouhaveatleastoneotherdogthatdoesntsleepoutside.

Whenacommaseparatesthesubjectfromawhichorawho,wearedealingwithasubjectthatistheone
andonly.Sothesecondsentenceistalkingaboutyouroneandonlydog,whichsleepsoutsideandhasabad
caseofthe eas.Theuseofwhichsignalsthebeginningofanonrestrictiveclause.Thereasonitiscalleda
nonrestrictiveclauseisyoucouldeliminateitwithoutmuchchangeinmeaning.

Mybiologicalmom,wholivesinSeattle,iscomingtovisitthisweekend.

MyfriendwhohastheloanIllneedtopayrentisvisitingthisweekend.


sat.magoosh.com 70


Inthe rstsentence,westillknowthekeyinformation:yourmomiscomingtovisit.Thereisabsolutelyno
vaguenessbecauseyouknowexactlywhichmom:youroneandonlybiologicalmom.

Bygettingridoftheclauseinthesecondsentence,essentialinformationislost.Wehavetoknowwhichfriend
outofallofyourfriendsisvisiting.Becausethisfriendistheonewhoisgoingtogiveyouamajorhelpinghand.

InanSATcontext

SohowexactlywilltheSATtestthis?

Well,thegoodnewsisthatitnotgoingtobenearlyascomplicatedaseverythingabove.Buttobecon dent,it
helpstoknowthenitty-grittyabove.However,theSATwillmainlycareaboutproperpunctuation.

Butitwontjustbeaquestionoftocommaornottocomma;youwillalsohavetorelyonabasicknowledgeof
semicolonsandem-dashes.

1)Moststudentswhochoosetogotograduateschoolmustoftentakeoutstudentloansinorderto
attend.

A)NOCHANGE
B)studentswhochoosetogotograduateschool,
C)studentswhochoosetogotograduateschool
D)students,whochoosetogotogrammarschool

2)ManyknowonlyafewfamousvistasoftheGrandCanyonthatactuallywends270milesthroughthe
desert oorandhasanaveragedepthofonefoot.
A)NOCHANGE
B)theGrandCanyon;
C)theGrandCanyon,that
D)theGrandCanyon,which


sat.magoosh.com 71


Answersandexplanation

1)A

Wearetalkingaboutasubsetofmoststudents:thosewhogotogradschool.Ifweweretoputcommasafter
moststudentsanddirectlyafterschool,wedbede ningmoststudentsasthosewhogotogradschool.

Luckily,thetestdoesntaskustochoosebetweentherestrictiveandnonrestrictivephrasebasedonthis
meaning.Thedi erencebetweenthecorrectanswerandotheranswerchoicesisbasedonpoorgrammar:(B)
shouldhavenocommadirectlyafterschool;(C)hasanunnecessaryem-dash;(D)hasonlyonecommaandnot
two,thewaywewouldexpectwithanonrestrictivephrase.Thecorrectanswer,(A),usesarestrictivephrase
(remember:nocommas).

2.D

WeneedwhichtoshowthattheGrandCanyonisde nedasactuallywends.Theoriginal,answer(A),
impliestherearemorethanoneGrandCanyonbyusingthewordthat,whichsetsuparestrictivephrase.That
setsuptheunintendedmeaning:outofalltheGrandCanyons,itistheoneTHAT.

6.Unnecessarypunctuation

Sometimes,whenasubjectisseparatedfromitsverbbymanywords,wearetemptedtouseacomma.

Incorrect
Thespreadofairborneinfectionsinpublicplacesorincloselycon nedareas,isnothingnew;our
e ortstocombatthem,however,aremorevigorousthanever.

Thesubjectofthesentenceisspread;theverbmatchingthatsubjectisis(waydownthere,nexttothe
nothing).Becauseofthisdistancewearetemptedtoputacommathere.However,ifthesubjectisfollowedby
arestrictivephrase(basically,thereisnoothercommaseparatingthesubjectfromaphrasethatreferstoit),
thenyoushouldnotputacommabeforeis.


sat.magoosh.com 72


Correct

Thespreadofairborneinfectionsinpublicplacesorincloselycon nedareasis
nothingnew;oure ortstocombatthem,however,aremorevigorousthanever.

Ifaphrasehasanonrestrictiveclause/nonessentialclausebetweenthesubjectandverb,alwaysrememberto
usetwocommas:onerightafterthesubjectandonerightbeforetheverb:

Incorrect

At900pages,DonQuixote,whichsomeconsideredthe rstnoveleverwrittenisnoeasyread.

Incorrect

At900pages,DonQuixotewhichsomeconsideredthe rstnoveleverwrittenisnoeasyread.

Incorrect

At900pages,DonQuixotewhichsomeconsideredthe rstnoveleverwritten,isnoeasyread.

Correct

At900pages,DonQuixote,whichsomeconsideredthe rstnoveleverwritten,isnoeasyread.(Notice
thetwocommas).


sat.magoosh.com 73

ExpressionofIdeas:TheArtofWriting

Intro

Yes,thisisaprettypretentioustitle.ButitistheonethattheSATgaveus.Forshorthand,expressionofideaswill
become(EOI),andIwilldropTheArtofWritingentirelylestIstartsoundingallhighfalutin.

EOIconsistsofthreesections:Development,Organization,andE ectiveLanguageUse.

TheCollegeBoardfurtherbreaksthesesectionsdown;however,theyitinsuchawaythatthetitlescanbe
technicalanddi culttoremember.Forinstance,propositionissimplyafancywayofsayingthatyouwillhave
toconsideradding,keeping,ordeletingsentences.So,Illjustgiveanoverviewofdevelopment,organization,
ande ectivelanguage.Thatwayyoullknowwhattoexpectwithoutgettingboggeddowninallthetechnical
terms.

Finally,Imnotgoingtogoasin-depthasIdidwithStandardsConventions(thegrammarpart).Itmakesmore
senseforyoutoreadaboutthedi erenttypesofEOIandthendoactualpracticequestionsfromtheCollege
Boardorothersources.Ido,attheendofthissection,haveapracticewritingpassageinwhichIexplainsome
oftheEOIsubcategorieswhentheyarerelevanttoaquestion.

Development

Thesequestiontypeswillaskyoutoadd,delete,orkeepasentencedependingonhowitrelatestothemain
ideaofthepassageortheparagraph.Oftenthismeansyoullhavetoevaluatethesissentencesortopic
sentences.Andthatmakessense:yourteachershavemostlikely(correctly)stressedthatthemostimportant
partofyouressaysisthethesissentencefollowedbythetopicsentence.ThisisSATswayoftestinghowwell
youcanhandlethebigideasofanessay.

Bytheway,theCollegeBoardthrowsthegraphquestionintothisgroup.Thatfactistotallyirrelevant.Just
remember:outof44questions,therewillbeonegraphquestion.Myadvice?Doafewpracticegraphquestions
fromtheCollegeBoardbook.YoullseethatthisisactuallyoneoftheeasierquestiontypesontheWritingtest.


sat.magoosh.com 74


Organization

Developmentisaboutthemainideasoftheessay;organizationisaboutthedetailstheessayusestosupport
thesemainideas.Again,thisissomethingyoudowhenwritingastandardessay.Thatis,youdontjustdropa
claimintheplaceofyourtopicsentencewithoutensuringthatthesentencesthatfollowsupportthisclaim.

Thisquestionwilltakeaninterestingform,asyoullseeinthepracticepassagebelow.Basically,oneofthe
paragraphswillhaveabracketednumbernexttoeachsentence,signifyingthepositionofthatsentenceinthe
paragraph(itsalotclearerifyoujustlookatthepassage).Yourjobisto gureoutthebestplacementforthe
sentence.Thisisverycommonandyouwillbegintothinkoforganizationastheonethatasksyouwhereto
putthesentence.Andthatsperfectly ne.Youdonothavetoknowthespeci cnameofanyquestiontypefor
thetest;youjusthavetoknowhowtoapproachthequestion.

E ectiveLanguageUse

IactuallylikehowtheCollegeBoardbreaksupthissection,soIlluseitsterminologyhere.

Precision

Incertaincontextssomewordsmightbesynonymsbutinothercontextsifyouusethewordsinterchangeably
youaregoingtogetsomesurprising(andunintended!)meanings.

1a)Asmidnightapproached,mostofthecampersretiredtotheirtents.

1b)Asmidnightapproaches,mostofthecampersreturnedtotheirtents.

2a)Mostofthepeopleinthecitywhohaveretired,settledownintheoldpersonscommunitydownthe
road.

2b)Mostofthepeopleinthecitywhohavereturned,settledownintheoldpersonscommunitydown
theroad.


sat.magoosh.com 75


In1aand1b,thewordsareinterchangeable.Butwhenthecontextisdi erent,asin2aand2b,themeaningof
retiredchanges,thenyouhaveaproblem.

HeresanotherexampleintheformofanSATquestion:

DickensLondonwasnotthegleamingmetropolisoftodaybutacityinwhichtheaverageliving
conditionsweresogrossthatcholeraepidemicswerenotuncommon.

Whichchoicebestmaintainsthetoneestablishedinthepassage?

A)NOCHANGE
B)negative
C)squalid
D)immoral

Explanation:

Onsomeprecisionquestions,youllalsobeaskedtoeliminateanswerchoicesthataretoovague(negative)or
tooinformal(gross).D)Immoralisaspeci cword,butremembertheretiringexampleabove;itdoesntquite t
thecontextsinceyoucanhaveimmoralpeopleorpracticebutyoucanthaveimmorallivingconditions.SoC)
squalidisthebestchoice.

Concision

Dontsaythesamethingtwice;inotherwords,dontrepeatyourselfwhenyouvealreadysaidthesamething
elsewhere.

Yes,thatsagrammarjoke.Igaveyouanexampleofredundancy.IsaidsomethingandthenIrepeatedit,not
wordforword,butprettyclose.


sat.magoosh.com 76

Thetestwillalwayswantyoutoavoidtheseredundanciesinthespiritofconcision.Alsocalledeconomy,ina
writingcontext,concisionaimstouseasfewwordsaspossibletoexpresstheexactsamepoint.

Whilesuchcriteriamightstrikeyouasbeingabitvague,thetestisalwaysveryclearwhenitistesting
redundancyyoujusthavetopickuponthefactthatitistestingconcision/redundancyandnotprecision/word
choice.Letstryafewexamples:

1)Onayearlybasis,thecompanyhasbeenmakingover100milliondollarsannually.

2)Johnnywasnotpopularamongsthisclassmatesbecausehewasalwaysspitefulandheshowed
maliceateveryturn.

3)Presently,IdonthavetheabilitytohelpyouatthismomentbecauseImtoobusyand
needabreak.

Howwouldthisactuallylookonthetest,giventhatanunderlinedpart,andnotthewholesentence,isbeing
tested:

Presently,IdonthavetheabilitytohelpyouatthismomentbecauseImtoobusyandneedabreak.

A)NOCHANGE
B)currently
C)reallyspeaking
D)deletetheunderlinedportion

Herearethethreewordsthatareredundantforeachofthethreequestionsabove:

1)yearly=annually
2)spitefulmeansshowingmalice
3)presently=atthismoment

A nalnoteonconcision


sat.magoosh.com 77


Oftentimes,anentirequestionwontbededicatedtoconcision.Rather,aquestionwillbeaboutsomeother
grammaticalrule,butoneoftheanswerchoiceswillhappentolackconcision.Thatanswerchoiceisalmost
alwayswrong.

Styleandtone

Tone

Sometimeswhenwewrite,welapseintophrasesthataretoocasualforanessay,butareoneswemightvery
wellusewithourfriends.Onthe ipside(orshouldIsayinversely),wesometimestrytoimpressourteachers,
orwhoeveritisreadingouressay,byusingoverlyformalandvocabulary-heavyphrases.

Findingthatproperlevelofformalityisthefocusoftonequestions.SincetheSATwritingpassagesarealways
writteninprofessionalstyle,thekindyoudencounterinanarticlefoundinamagazine(thinkNational
Geographic)ornewspaperarticle(thinkthefrontpageoftheWallStreetJournal),thetoneisneverextremely
casualorfrighteninglysti andformal.

Extremelycasual

1.ThatsneatwhathappenedduringtheOlympics.
2.Myjobissuperspecial.
3.Findingoutstu aboutthepresidentsisntasboringasIthought.

Overlyformal

1.SuperlativesaboundinrenderingthephenomenonconcomitantwiththeOlympics.
2.Myvocationa ordsmorethanamodicumofautonomy.
3.ImbibingknowledgepertainingtotheformerheadsoftheOvalO ceprovedmorescintillatingthanIcould
havepossiblyenvisioned.

WhattheSATwouldwrite


sat.magoosh.com 78


1.ThisyearsOlympicsprovedtobeamemorableone,withnumerousworldrecordsbeingset.
2.Myjoballowsmethe exibilitytoworkfromhomeandsetmyownschedule.
3.LearningaboutthelivesofU.S.presidentswasnotnearlyasdullasIhadanticipated.

Noticethatboththeoverlycasualandtheoverlyformaltendtobevague,whereastheproperlevelofwritingis
clearandspeci cwithouteverbeingverbose(Imbibingknowledgelosesouttotheunpretentiouslearningin
theconcisioncontestanyday).

Style

SometimestheSATwantstoknowifyoucanpickuponacertainstyleawriterisusing.Sinceitisdi culttotest
stylewithoutdirectlyaskingwhattheauthorisdoing,thetestwritershavefocusedmainlyonsentence
structure.Speci cally,therewillbeanuncommonwayofarrangingasentenceorsentencesthatthetestwants
youtopickupon.

Thoughyouarentlikelytoseethiskindofquestion,youmightbewonderinghowyoullknowwhenyouare
dealingwiththiskindofquestion.Well,thetestwillmostlikelytellyouinthequestionstem:

Whichofthefollowinganswerchoicesismostconsistentwiththestyleusedbythewriterofthepassage?

Standinginfrontofanaudienceofthousandswasthegreatestmomentoftheyoungsingerslife.Faces
glowedwithadulation.Eyes xatedonherwithwonder.Peoplecouldnotwaittohearher.


A)NOCHANGE
B)Peoplewereanticipatinghersinging.
C)Earsperkedupinanticipation.
D)Shewasgoingtosinginthecrowdsears.

NoticehowC)parallelsthestructureusedbytheprevioustwosentences:bodypart+verb+preposition.


sat.magoosh.com 79


Syntax/CombiningSentences

Letme rstgiveyouthegeneralde nitionofsyntax;then,Illtellyouexactlyhowthetestwriterswilluseit


(thatsthemoreimportantpart).

Syntax:thearrangementofphrases,clausesandsentences.

Whatthismeansasfarasthetestgoesishowtobestcombinesentences.Thatsthekeyphrasehereandyou
canforgetthewordsyntax.ThinkofthissectionasCombiningSentences.Thetestwillneveraskyouwhether
youshouldcombinesentences(theywontmakeyoutojudgeofwhethersomethingistoochoppy)butwillonly
askyouhowtobestcombinetwoormoresentences.

Shortunderline

MilesDaviswasaninnovativejazztrumpeter.Hedevelopedastyleknownascooljazz.

Thissentenceisslightlychoppy,sothetestwantsyoutomakeitlessso.

Whichchoicemoste ectivelycombinesthesentencesattheunderlinedportion?

A)NOCHANGE
B)jazztrumpeterwhodeveloped
C)jazztrumpeter;andinhismusic,hedeveloped
D)jazztrumpeter,butdeveloped

Explanation:

A)ischoppy,meaningthatitdoesntprovideatransitionwordorphraseshowingtheconnectionbetweenwhat
comesbeforetheperiodandwhatcomesafterit.Noticehowthe rstparttalksabouthowMilesDaviswas
innovative.Thesecondpartgivesusanexample:hedevelopedanewstyleofjazz.D)iswrongbecauseit
indicatesacontrastbetweenthetwosentences.C)iswrongforpunctuationreasons.Theanddirectlyafterthe


sat.magoosh.com 80

semicolonisunnecessary.B)iscorrectbecauseitavoidsthechoppinessinA)byconnectingthetwosentences.
WhoavoidstheunwantedcontrastedsuggestedinD).

Entiresentenceunderline

Letstakethesamesentenceandshowyouwhatelsethetestcando.

MilesDaviswasaninnovativejazztrumpeter.Hedevelopedastyleknownascooljazz.

Whichchoicemoste ectivelycombinesthesentences?

A)DevelopingastyleknownascooljazziswhyMilesDaviswasaninnovativejazztrumpeter.
B)MilesDaviswasaninnovativejazztrumpeter,andhealsodevelopedastyleknownascooljazz.
C)AstyleknownascooljazzwasdevelopedbyMilesDavis,aninnovativejazztrumpeter.
D)Aninnovativejazztrumpeter,MilesDavisdevelopedastyleknownascooljazz.

Explanation:

Onethingyoullwanttolookforincombiningsentencesquestiontypesisalogicalbalancebetweentwo
sentences.HerethelogicalconnectionisthatMilesDavisscreationofanewstyleresultedfromthefactthathe
wasinnovative:INNOVATIVEresultsinCREATIONOFNEWSTYLE.

A)Ivementionedbeforehowconcisionissomethingthatcanpertaintomanyquestiontypes.Here,iswhy
leadstoasentencethatlacksconcision.Also,theoriginalsentenceisnottoexpresswhatmadeDavisan
innovativetrumpeter(hedevelopedanewstyle)butthathewasinnovativetrumpeterwhoinventedthisnew
styleofjazz.

B),byusingandalsodoesnotshowthislogicalconnection.ThissentencemakesitsoundasthoughDavis
wastworelativelyunrelatedthings:hewasinnovativeand,bytheway,healsocreatedanewstyle.


sat.magoosh.com 81

C)isinpassivevoice.Thinkofpassivevoiceasthebytense,e.g.,TheballwashitBYhim.Thisisalmost
alwaysconsideredincorrectontheSAT.Thatis,therewillalmostalwaysbeaperfectlygoodanswernotin
passivevoice.

D)doesagoodjobofshowingtheimportanceofthetwopartsofthesentence.Aninnovativetrumpeterisnot
asimportantashecreatedanewstyle.Therefore,wemakeitasubordinateclause(subordinatemeansof
lesserimportance).Thatputsthefocusonthecontentofthemainclause.


sat.magoosh.com 82

PuttingitallTogether:PracticePassage

Asyouwillsee,onlyafewofthemanygrammaticalprinciplesabovewillbetestedinanyonepassage.Overthe
courseofanentiresection,youlllikelyseemostofthecategoriestestedatleastonce.However,somequestion
typesaresorarethattheymightpopuponlyonceeveryfewtests.Ihavetriedtogiveageneralsenseofthe
likelihoodthatagrammarorstylisticconceptwillbetested.Still,youlltendtogetabettersenseofthisasyou
gothroughpracticesectionsintheCollegeBoardbook.

Heresapracticepassagetogetstartedwith:

PracticePassage

Manyhaveheardofthesequoiatrees,giantCalifornianredwoods.Butfewknowthatthetreesarenamedafter
anactuallivingperson,onewhowasagiantamongsthisnativepeople:TheCherokees.

Borninthelate18thcenturyinOklahoma,orwhatwasthentheCherokeeNation,SequoyahSENTENCE
STRUCTURE1displayingearlyinlifeaknackfor guringoutthingsonhisown.Forinstance,toEFFECTIVEUSE
OFLANGUAGE2increasethee ciencyofdairyproductiononthelandhisfamilyownedheconstructedadairy
houseanddevisedasystemofmilktroughs.ThisingenuityservedSequoyahwellinhislatervocationasa
CONVENTIONS3silversmith,wherehewasabletocreatesuchinterestingdesignsthathishandiworkwas
highlysoughtafter.

WhatSequoyahisrememberedfortoday,SENTENCETRANSITIONS4however,hadafargreaterimpactonthe
CherokeeNation:hewasthe rsttoeverdevelopafunctionalalphabetforaNativeAmericanlanguage.Asa
silversmith,Sequoyahcameincontactwithmanywhitesettlersandnoticedthewritingontheirpiecesof5
paper,hedescribedthemastalkingleaves.SENTENCESTRUCTURE6Sequoyah,spottingtheutilitysuchan
innovationo ered,hopedtoconvincetribaleldersthattheCherokeenationneededasimilarwayof
communicating.Theelders,however,expressedskepticism,thinkingitimpossibleforsomebodyto
communicateapersonsmessageifthatpersonVERBTENSE7wasnotpresent.Someevenbelievedthat
writingwasaformofsorcerythatshouldbeavoided.


sat.magoosh.com 83

SENTENCEPLACEMENT8[1]Reassuredbythisreception,Sequoyahbegan ndingawaytoturnthe
complicatedsoundsofhislanguageintoasystemofsyllables.[2]HopingtoprovethattheCherokeelanguage,
likeEuropeanlanguages,couldbeusedtocommunicateimportantmessages.[3]Afterisolatinghimselffora
year,Sequoyah nallyemergedwithaviablesystemofwriting.[4]Sincemuchofhisfamilybelievedthat
Sequoyah,duringhisyearinseclusion,waspracticingwitchcraft,hissix-year-olddaughterbecamehisunlikely
pupilandquicklylearnedtoreadthelanguage.[5]Toconvincetheeldersthatthesystemactuallyworked,he
neededawillingapprentice.[6]Securinganaudiencewiththeelders,Sequoyahaskedthemtosayawordthat
hewouldEFFECTIVELANGUAGEUSE9proceedtowritedown.[7]Then,hewouldsummonhisdaughter,who
hadbeenfarbeyondearshot,andshefaithfullyPUNCTUATION10read,totheastonishmentoftheelderseach
wordthathadbeenwrittendown.[8]Withinmonths,Sequoyahhadsuccessfullytaughtthewritingsystemto
manyCherokees.MAINIDEA11

1.

A)NOCHANGE
B)displayed
C)displays
D)isdisplaying

Di culty:VeryEasy

2.

A)NOCHANGE
B)makefarmproductionbetter
C)augmentthee ciencyinthegenerationofdairy
D)upstheamountofmilkthatisproduced

Di culty:Medium


sat.magoosh.com 84

3.

A)NOCHANGE
B)silversmith,which
C)silversmith,inwhich
D)silversmiththat

Di culty:Medium

4.

A)NOCHANGE
B)Forexample,
C),additionally,
D),moreover,

Di culty:Easy

5.

A)NOCHANGE
B)paper,describing
C)paper;describing
D)paperdescribing

Di culty:Medium

6.
A)NOCHANGE
B)Sequoyahspotted
C)Sequoyah,hespotted
D)Sequoyahspotting


sat.magoosh.com 85


Di culty:Hard

7.

A)NOCHANGE
B)hadnotbeen
C)werenot
D)wouldnothavebeen

Di culty:VeryHard

8.Whichofthefollowingisthemostappropriateplaceforsentence5?

A)Whereitisnow.
B)Immediatelybeforesentence4
C)Immediatelyaftersentence2
D)Immediatelyaftersentence6

Di culty:Hard

9.

Incontext,whichchoicebestcombinestheunderlinedsentences?

Reassuredbythisreception,Sequoyahbegan ndingawaytoturnthecomplicatedsoundsofhislanguageinto
asystemofsyllables.HopingtoprovethattheCherokeelanguage,likeEuropeanlanguages,couldbeusedto
communicateimportantmessages.

A)NOCHANGE


sat.magoosh.com 86

B)Sequoyahwasdiscouragedbythereactionoftheeldersandspentseveralmonthsavoidingthetaskoftrying
totranscribethecomplicatedCherokeesoundsintoaconsistentwritingsystem.
C)Undeterred,Sequoyahbeganworkingonawayoftranscribingthecomplicatedsoundsofhislanguageintoa
systemofsymbols,hopingtoprovethattheCherokeelanguage,likeEuropeanlanguages,couldbeusedto
communicateimportantmessages.
D)BelievingthattheCherokeelanguagecouldbeusedtocommunicateimportantmessages,likeEuropean
languages,Sequoyahbegan ndingawaytoturnthecomplicatedsoundsofhislanguageintoasystemof
syllables.

Di culty:Hard

10.

A)NOCHANGE
B)precede
C)succeed
D)progress

Di culty:Medium

11.

ThiswriterwantstoconcludethepassagewithasentencethathighlightstheenduringlegacyleftbySequoyah
tohistribe.Whichchoicewouldbestaccomplishesthisgoal?

A)ManyoftheelderspresentweresoimpressedthattheybecameexpertsofSequoyahswritingsystem,and
taughtmanyothermembersoftheCherokeenation.

B)In1825,nearlytenyearsafteritscreation,theCherokeenationo ciallyadoptedSequoyahswritingsystem,
anactallowingpartsoftribeseparatedbylongdistancestocommunicatee ectivelywithoneanotherand
mergethedividedEastandWesttribes.


sat.magoosh.com 87

C)ThemostimportantaspectofSequoyahsworkwasthatittookacomplicatedphoneticstructureandturnedit
intoanalphabetthatisreadilyaccessibletoanybodywhowantstolearnit.

D)Sequoyahwillbeforeverrememberedforthewritingsystemhehelpedcreate,aninnovationthatisstartling
eveninthisdayofhigh-poweredcomputers.

Di culty:Hard

AnswersandExplanations:

1.The rstphrase,startingwithBorninthelate...:describesthesubjectSequoyah.However,thesubjectneeds
averbinthepresentorpasttense,nottheparticipialform(youwouldntsay,Ieatingmyfood).Sincethe
paragraphdescribespasteventsandstickstothepasttensethroughout,wewantanswer(B).

2.Inquestionsaskingforthemostappropriatewaytophrasesomething(seethesectiononsyntax),youwantto
chooseananswerthatisneithertoocasualnortooformal.Youllalsowanttomakesurethattheanswerisnt
tooverbose(seesectiononconcision)orvague.Inthiscase,(B)istoovague.(C)istooformalandlacks
concision.(D)usessuchcolloquialwordsasups.Also,dairyistherightlevelofformality.Milkisnotquite
appropriateandsomethingmoreformalandLatin-based,suchaslactal(notmentionedintheanswerchoices)
wouldbetooformal.So(C)istheanswer.

3.Thefocushereisvocation.Itisnotanactualplaceso(A),whichuseswhere,isincorrect.Whichis
tempting.Butifyouplugitintothesentence,itimpliesthatSequoyahcreatedthevocationofthesilversmith.(D)
su ersfromthissameproblem.Whendescribinganounthatanactionrefersto,wewanttouseinwhich.
Example:ThiswasthegameINWHICHheSCOREDthewinninggoal.(C)istheanswer.

4.ThepreviousparagraphgivesaquickbiographyofSequoyahsaccomplishment.Thenextparagraph
transitionstotheaccomplishmentthatheisreallyknownfor.Therefore,thereisacontrastbetweenthe
accomplishmentsmentionedintheparagraphprecedingthisquestionandhismainaccomplishment:the
creationoftheCherokeelanguage.Only(A)givesusacontrastword.

5.Asis,thisquestioncontainsaclassicSATerror:thecommasplice.Thishappenswhentwoindependent
clausesarejoinedbyacomma(seecommasectionabove).Byputtingacommaandtheaparticipleimmediately


sat.magoosh.com 88

afterthatcomment,wemakesurethatthephrasedescribingthemastalkingleavesreferstothesubject,
Sequoyah.(C)isincorrectbecauseitusesasemicolontoseparateadependentclausestartingwithaparticiple
andanindependentclause.(D)impliesthatthepaperdescribeditselfastalkingleaves--anabsurdstatement.
So,(B)istheanswer.

6.Thisisamoreadvancedtestofsentencestructure.Wecanseparateasubjectfromaverbusingaparticipial
phrase,aslongasthatphraseisseto bytwocommas.Therefore,theoriginaliscorrect.(B)iswrongbecause
wecanthavetheparticipleinthe-edformifacommaseparatesitfromanindependentclause,thewaywecan
withaparticipleendingin-ing.(C)unnecessarilyrepeatsthesubject.(D)omitsthecommabetweenthesubject
andspotting.(A)istheanswer.

7.Thisisadi cultquestionbecauseitusesthesubjunctivecase(seesectiononmood).Whenthereisa
hypotheticalsituation,the rstandthirdpersontakesonapluralformofaverb.Therefore,wasshouldbe
wereinthatpersonwerenotpresent,sincewearetalkingaboutasituationthatishypothetical(thinkingit
impossible).Answer:(C).

8.Sentence5saysmoreorlessthatSequoyahneededawillingpupil.Sentence4describeshowmostofhis
familythoughthedgonealittleloopy,sohewasforcedtoturntohisdaughterasanapprentice.Itmakessense
toputthesentencesayingthatheneededapupilbeforethesentencethatdescribetheprocessofrecruitinga
pupil.Answer:(B).

9.Theoriginalhasafragmentinthesecondpart,sinceitlacksasubjectandsimplybeginswiththeparticiple
hoping.(B)isnotgrammaticallyincorrect.However,whenevertheSATusespassivevoice(discouragedbythe
reaction),youwantto ndananswerthatusestheactivevoiceANDisgrammaticallycorrect.(C)isexactlythat
answer.(D),byusinglikeimpliesthatSequoyahislikeEuropeanlanguages.Answer:(C).

10.Toproceedtodosomethingistogoaheadanddoit,whichisexactlywhatwewanthere.
(B)meanstocomebefore,andisthecorrectanswer.ItisunidiomatictosaysucceedTO;thecorrectidiomis
succeedIN.ItisalsooddtosayprogressTOdosomething.

11.Thekeytoansweringthisquestioncorrectlyisenduringlegacy.Wedontwantamerepositiveoutcome;we
wantthereasonSequoyahsinnovationswashighlymomentousforhispeople.(A)isonesuchanswer.Thats
greatthatmanyelderslearnedthelanguage.Butitdoesntreallytellusthemajorpositivee ectonthe


sat.magoosh.com 89

CherokeesthatSequoyahswritingsystemhad.(B)givesusaclearreasonwhythisinnovationwassoimportant:
ithelpedconnecttheCherokeeoverlongdistancesandmendtheriftbetweenEastandWestCherokees.(C)
justexplainsthatitwasinnovative.(D)givestheinaptcomparisonwithmoderndaycomputers.(B)istheanswer.


sat.magoosh.com 90

SATReadingTest


sat.magoosh.com 91

IntroductiontotheSATReadingTest

ThenewSATReadingtestisquitedi erentthanthatoldhe-who-shall-not-be-namedReadingtest.Gonearethe
sentencecompletionquestionsthatquizzedyouontoughvocabulary.Now,itsallaboutthepassages.Meaning,
itsallaboutreadingcomprehension(ok,sotheresalittlevocabsnuckintheretoo;welltalkaboutthatinabit.)

WhattoKnow:
TheSATReadingtestis65minuteslong.
Ithasfourpassagesplusonepairofpassages(so vepartstotal).Eachpassageis500-750wordslong.
Ithas52questions(10-11questionsperpassage).
Itsallmultiplechoice.
OnepassagewillbefromU.S.andWorldLiterature,twopassages(or1passageand1pair)willbefrom
History/SocialStudies,twopassages(or1passageand1pair)willbefromScience.
Therewillbe1to2graphicsembeddedin1History/SocialStudiespassageandin1Sciencepassage.
Passagecomplexityrangesfromgrades9and10toearlycollege.
YourscorewillbecombinedwithyourWritingscoreforoneEvidence-BasedReadingandWritingscore
outof800.

WhattoStudy:
Readingcomprehension
Understandingbasictablesandgraphsofdata
Textsfromavarietyofgenresincludingcontemporaryandclassicliterature,science,andsocial
studies,includingonetextfromU.S.FoundingDocumentsortheGreatGlobalConversation(This
meansthingsliketheDeclarationofIndependenceoraspeechbyNelsonMandela).

WhatNottoStudy:
Esotericvocabwords(suchasesoteric).De nitely ipthose ashcardsifyouhaveaweaker
vocabularysothatyoucanbetterunderstandthepassages.However,thequestionsthatspeci cally
addressthemeaningofwordsonthenewSATprimarilyconcernmorecommonwordswithmultiple
meanings.


sat.magoosh.com 92

TheSATReadingTest:Strategies

Letsstartwiththebasics.TheReadingsectionoftheSATrequiresintensefocus.Youllbefacinglongreading
passagesandwillevenhavetocomparetwopassagesonsimilartopics.Sobeforewelaunchintothenitty-gritty
ofthepassageandquestiontypesontheSAT,letstalkaboutthebigpicture:thegeneralreading
comprehensionstrategiesthataregoingtohelpyougetthemostoutofyourreadingtimeonthetest.

1.ReadtheentireReadingpassage rst
ThereusedtobethisurbanmyththatyoucouldacetheReadingpassagesbyreadingthequestions rstand
thengoingbacktothepartsofthepassagethequestionstellyouto.Iamguessingthislegendmightfadeabit
nowthattherearefewerlinenumberquestionsontheSAT,meaningthequestionsarentgoingtotellyou
exactlywheretolookfortheanswer.Youhavetohuntforit,orrememberwhereyousawit.Thecruelty!

Nowwearentsayingyoucantstillgetafewquestionsrightusingthistactic,butifyouskipthepassage
altogether,youarelikelytomissmanyquestionsthatrelatetothegeneralideasinthepassages.Youarealso
likelytospendmoretimetryingtochoosebetweentwoanswers,becauseyousimplydonthavethecontext
thatyoucanonlygetbyreadingthepassage.
Soletussayitloudandclear:Alwaysreadtheentirepassage rst.

2.Getthebigpicture
DoingwellonSATReadingpassagesrequiresunderstandingwhatthepassageisabout,ingeneral.Ifyou nd
yourselfstringingwordstogether,hopingtoreachtheendofatorturouspassage,youareactuallyhurting
yourself.Itisnotaboutgettingtotheendofthepassage;itisaboutunderstandingthepassage.Sopause
frequentlyasyoureadanddigestwhatyoujustlearned.

3.Watchoutfortheswamp!
Somepeopletaketheideaoftryingtounderstandthepassagetotheotherextreme.Ihavetounderstand
everydetail,theytellthemselves.Manyofthesepassagesareconstructedinsuchawaythatthereisalotof
dense,nastymaterialburiedinthemiddle.Studentsoftengetpulledintothisswampofwordsandcomplex
ideas,believingthattoanswerthequestions,theyhavetounderstandthemostcomplexpartofthepassage.

Thisisoftennotthecase,asthequestionstypicallydealwitheasierpartsofthepassageoratleastnot
exclusivelyonunderstandingtwoback-to-backdi cultsentences.Thekeyistounderstandthetopic


sat.magoosh.com 93

sentencesoftheparagraphsandfeelcomfortableglossingoverthetoughstu insteadofgettingstuckin
aswampofwords.

4.Takesnapshots
No,werenottalkingabouttakingpictureswithyouriPhone.Snapshotsreferstothoselittlementalsummaries
youmakeinyourheadasyouread.Eachparagraphisaunitofinformationimportantinformationthatyou
shouldmakeaquicksummaryofwhileyouread.Forinstance,whenyouhave nishedwiththe rstparagraph,
youshouldthinksomethingalongthelinesof,Ok,thatwasaboutacoupleofreasonsradiotelescopesare
importantinhuntingforaliens.Hmmthisparagraphjusttalksaboutoneofthosereasons,whichisthat.

Thesesummariesshouldnottakeyoulongonlyabout vesecondsorso.Forthoseofyouwhoarentfast
readers,orarentusedtosummarizingstu inyourheads,youcanalsowritemini-paragraphsummariesinthe
margins(thoughwerecommendbuildinguptowhereyouarecomfortablemakingmentalsnapshotsofeach
paragraph).

Andremember:youarejustgoingforthebigideas.Dontgetburiedintheswamp,becauseitwilldisruptyour
abilitytounderstandthemainideasofthepassage.

5.Getreallyexcited
Weknowthissoundsweird.Afterall,youaredealingwithSATpassagesthatoftenseemedintentionally
designedtotortureyou,whatthe%$@istheretogetexcitedabout?!Butthatsthepoint:ournaturaltendency
uponstartingareadingpassageistofallasleeporatleastgetbored.Byconvincingyourselfthatwhatyouare
abouttoreadissofunandentertainingandthusboostingthatpulseeversoslightlyyoullbefarmorealert
asyouread.

Bycombiningalltheelementsabove,youaregoingtobefarmorepreparedforthosesneakyquestions,andall
thosecarefullyplacedtrapsintheanswerchoices.Thekeywhenapplyingthesetechniquesispatientpractice.
Youarenotgoingtoautomaticallystarttakingrealcleansnapshotsasyouexcitedlymakeyourwaythrougha
passageaboutthedisputeregardingLinnaeusstaxonomiccontributiontonaturalscience.Butwithpractice,
hey,youjustmight.


sat.magoosh.com 94

ActiveReading

Readthefollowingpassageandthenwelltalk:

OnceAmericanmenreturnedfromtheWWIIbattle elds,theyquicklydisplacedthewomenwhohad
temporarily lledjobsotherwisereservedformen.Withmostwomenrevertingtotheirdomesticroles,the
dramaticincreaseinthenumberofinfantsbornisperhapsnottoosurprising.Yet,suchfactorsalonecannot
explaintheincreaseinthenumberofbirthsfrom1946-1951.Murraysuggeststhatbothwomenandmens
perspectiveschanged,mostlybecauseofAmericassuccessinthewar.Thisoptimism,inpart,fueledtherapid
growthinpopulation.However,manyarguethatwomen,inreturningtothehome,wereabletofocusonraising
afamily,regardlessoftheirlevelsofoptimism.

Oh,itsyouagain.Welcomeback!Withoutlookingatthepassageabove(youmightwanttoputyourhandover
thepassage,orscrolldownabittohideit),tellmewhatyoujustread.

Yourelikelytopauseforaminuteandtrytograbontooneofthewordsorphrases oatingaroundinyourhead
(women,jobs,numberofinfants)andthenformulateastatementlike:ItwasaboutwomeninAmericaand
howtheyhadmorekids.Yourattentionlikelywanedafterthe rstcoupleofsentencesandmighthaveeven
derailedbythetimeyougottothenameMurray(youmightnotevenrememberreadingthatname).

Passivereading

Besidescomingupwithsomevaguegeneralities,youprobablyhaddi cultyformulatinganythingcoherentand
thorough.Muchofthereasonforthisisyourbrainwasinpassivemode:itwasstringingwordstogether,and
oncewordwaspileduponword,itgotlostinthewoods,sotospeak.

Butdontworry.Youarenotalone.99%ofyourfellowSATtest-takerswillprobablyhaveaverysimilarresponse.
Youarealsonotaloneifinsteadofstoppingandthinkingaboutwhatyouread,youkeptplowingforwardinthe
mistakennotionthatifyougottotheend,thepassagewouldsuddenlyallmakesense.


sat.magoosh.com 95

Thethingis,thepassagesontheSATarewritteninsuchawaythatafterthe rstcoupleoflinesorparagraphs,
yourattentionverywellmightwandero todaydream-land.Thegoodnewsiswecanusethepredictabilityof
SATpassagestoouradvantage.Todoso,weneedtounderstandhowapassageisdesigned.

Activereading

Bysimplifyingallthemajorcomponentsinyourhead,youwilleasilybeabletocomeupwithwhatwecalla
snapshot,whichisasimpli cationoftheimportantpartsoftheparagraph.Bypayingattentiontostructure
words,youwontgetlostintheseaofwordsbutwillbeabletofocusonwhatisimportant.

Soletstalkabouthowtoreadactivelysoyouarenotdesperatelystringingwordstogetherhopingforamiracle
attheend.

Thethreetenetsofactivereading

1.Noticeconnectionsbetweenparagraphs

Thereadingpassageaboveisbutonepossiblewayinwhichaparagraphcanunfold.Youllwanttoopenupan
SATbook(preferablytheO cialCollegeBoardguide)andseehowthepassagesareorganized.

HeresatypicalstructureforanSATnon ctionpassage.Thetopicisintroduced,maybewithananecdoteor
someothergeneralexamplesorstatements.Evidencesupportingatheoryonthetopicwilltypicallyfollow.Orif
thepassageexaminesmultiplesidesoftheissues,maybeseveralperspectiveswithevidencewillbediscussed.
Thenaconclusionwrapsuptheissue,maybeo eringsomefoodforthoughtforthefuture.

(SpecialnotefortheU.S.FoundingDocumentsandGreatGlobalConversationpassage:thisstructuremaynot
applyasneatlyifwearelookingatafamousspeechorletter,buttypicallytheauthorwillstillbeintroducinga
concernandthensupportingitwithvariousargumentsandthenwrappinguphisorherthoughts.)

Once,youcananticipateandrecognizethetypicalstructureofanSATpassage,itwillbealoteasierforyouto
categorizethecopiousinformationthepassagethrowsatyou.

2.Payattentiontostructurewords


sat.magoosh.com 96

Structurewordsarethegluethatholdstheparagraphtogether.Buttheyaremorethanthat;theyshowushow
thesentencesarelogicallyconnected.

Hereare veofthemostimportanttypesofstructurewords.Outofthese,theoneyoushouldpaymost
attentiontoiscontrastwords.

Contrastwords

however,(al)though,still,nonetheless,atthesametime,ontheotherhand,otherwise,but,yet,notwithstanding

Intensifyingwords

indeed,moreover,infact

Illustrativewords

forexample,forone,toillustrate

Causewords

because,since,forthatreason

E ectwords

therefore,thus,hence,consequently,asaresult

Underlinestructurewordswheneveryouseethem.Theyareincrediblyhelpfulcluesindeterminingthetwists
andturnsthepassageistaking.

3.Makeconnectionswithinparagraphs

Itsokaytoslowdownforasecondandevenlookawayfromthepage.Youllwanttodigestwhatsbeingsaid.
Thisisthegoldenpillartoactivereading.Speci cally,askyourself,whatistheparagraphsaying?Icallthese


sat.magoosh.com 97

paragraphsnapshots.Youforceyourselftomakeimportantconnectionsintheparagraph,whilesummarizing
keypointsinyourhead.Dothisattheendofeveryparagraph(ortwoiftheyarereallyshort).

Youmightbalk,thinking,Doesntthattakealongtime?Well,whenyougettotheendofthepassage,youll
haveaverygoodideaaboutwhatthepassageisabout.Consequently,youllbeabletoanswerthequestions
muchmoreeasily,savingyoutime(insteadofhavingtogobackandforthbetweenpossibleanswerchoices,
whichtypicallyhappenswhenyouhaveanimperfectunderstandingofthepassage).

Finally,youdontneedtounderstandeverydetailineveryparagraph.Especiallytowardstheendofthe
paragraphtherewillbeadensityofdetail.Itisbesttocomebacktothisonlyifthismaterialisrelatedtoa
question(whichittypicallyis).Seeingthistextforasecondtime,andwithinthecontextofthequestion,willoften
makeitiseasiertodigest.

Applyingwhatyouvelearned

Itseasytounderstandhowactivereadingworks.Itismuchhardertoapply.See,ourbasicinstinctistotrytoget
throughthepassageasquicklyaspossible.Therefore,ifyouarecurrentlytimingyourself,thinkingthattheonly
wayyoullgetbetteristogetfaster,youmightwanttoreevaluatehowyouapproachthepassageinthe rst
place.Again,bymoree ectivelypackagingtheinformationinthepassagethe rsttimearound,youllbeboth
moree cientandmoreaccuratewhenyouanswerthequestions.

Herearefewtipstohelpyouactivelyread:

1)Lookawayfromthepassageaftereachparagraph

Thisstrategyistogetyourbrainusedtotakingsnapshotsoftheparagraph.Bynotlookingawayfromthepage,
itiseasytobecomedistractedbythewordsdancinginfrontofyourface.Lookingaway,youcaneasilycomeup
withaquicksummary/snapshot:Itsabouttwodi erenttheoriesonthepopulationincreaseafterWWII.Oneis
thatpeopleweremoreoptimisticaboutstartingfamilies;theotherthatwomenwhowereworkingwereableto
gobackhomeandstartafamily.

Sayingthatinyourhead,andyoucandosoinanevenmoreabbreviatedfashion,takesamerefewseconds.
Then,whenthenextparagraphdealswith,say,ananalysisofthattheory,youllhaveamucheasiertime
followingalong.


sat.magoosh.com 98

2)Takenotes

Sometimes,itshardtoorganizetheclutterofthoughtspinballingthroughyourheadasyoureadapassage.
Takeshorthandnotesaftereachparagraph.

3)Lookawayafteryouvereadthepassage/jotdownthemainidea.

Thesameaspoint#1.Except,nowyoulljustwanttoanswerthequestion:Whatwastheprimarypurposeofthe
entirepassage?Puttingthisinyourownwordsisagoodidea,sincetherewilllikelybeaquestionthatasksyou
todoexactlythis.Jotitdown(inshorthandform)soyoucancon dentlyrefertoitwhileansweringquestions.

Activereadinginaction

Now,letsactuallytaketheparagraphfromthebeginningofthissectionanddissectit,muchthewayyourmind
wouldifactivelyreadingthepassage.

Withmostwomenrevertingtotheirdomesticrole,thedramaticincreaseinthenumberofinfantsbornis
perhapsnottoosurprising.Yet,suchfactorsalonecannotexplaintheincreaseinthenumberofbirthsfrom
1946-1951.Murraysuggeststhatbothwomenandmensperspectiveschanged,mostlybecauseofAmericas
successinthewar.Thisoptimism,inpart,fueledtherapidgrowthinpopulation.However,manyarguethat
women,inreturningtothehome,wereabletofocusonraisingafamily,regardlessoftheirlevelsofoptimism.

Themostimportantstructurewordsarecontrastwords,becausetheychangethedirectionoftheparagraph.In
otherwords,apersonitcouldbetheauthororsomebodytheauthortalksaboutinthepassagedisagrees
withsomethingorsomebodyelseinthepassage.Itisunderstandingthetwistandturnsintheparagraphthat
resultfromcontrastwordsthatisthekeytounderstandingthepassage.Remember,thepassageisnotjust
aboutimpartinginformation;itisaboutthedistinctionsthatarisefromadebateinwhichpeopletakevarying
positions.

Intheparagraphabove,noticehowthe rstsentenceintroducesthetopic.Afterthat,itisstraightintoacontrast
word.Therefore,thesecondsentencedoesnotagreewiththe rst.

1stsentence:womenreturningtohomebecauseofmorebabies


sat.magoosh.com 99

2ndsentence:otherfactorsalsoaccountformorebabies

Notice,thatthe2ndsentencedoesnotcompletelydisagreewiththe1stsentence.Itjustquali esorlimitswhat
thatsentencesays(animportantdistinctionforthoseaimingfortopscorestonotice!)

InthenextsentencewegetMurraysview.Alwaysnoticewhentheauthorbringsupanotherpointofview.

Murraysview:peoplebecamemoreoptimistic;hadmorebabies

Now,thereisanothercontrastword:however.Thissignalsthatwearegettinganotherpointofview.

Otherview:womenwereabletofocusonraisingafamily

One nalpoint

Onceyouvenoticedtherelationshipbetweensentencesinaparagraphandthemeaningtheyconvey,youwill
beabletodealwiththeSATquestionspertainingtothepassagefarmorecon dently.Indeed,youllbeableto
formulateananswerinyourhead.Andifyoudohavetoconsultthepassage(asyoushould),youllknowwhere
tolook.

Allinall,becominganadeptactivereaderwillhelpyouhoneinonthecorrectanswer,insteadofhavingto
fumblefranticallythroughthemassofwordsthatmakeuptheanswerchoices.


sat.magoosh.com 100

Pacing
Wewontbeataroundthebush.TheresalottoreadontheSATReadingtest.Andifyouarentcarefulwith
balancingyourtimebetweenreadingandquestionanswering,youverywellmight ndyourselfinapanicwhen
theproctorannounces5minutesleft.Soletstalkabouthowyoucanlearntopaceyourselfappropriately.

Timeperquestion
Onaverage,youhaveoneminuteand fteensecondstoanswereachquestionontheSATReadingtest,but
thisincludesreadingtime.Thetimeyouactuallyhaveperquestionwilldependonexactlyhowlongthepassage
is,howcomplexthepassageis,andhowfastofareaderyouare.

Timeperpassage
Sinceeveryonereadsatadi erentpaceandsomepassagesaredenserthanothers,itsbettertothinkabout
theapproximatetimeyoucanspendperpassageratherthanperquestion.With65minutesforthewholeSAT
Readingtest,thismeansyouhave13minutesperpassageforeachofthe5passagesifyoupaceevenly.So
checkyourwatchaftereachpassageandmakesureyouareroughlyontrack.Dontpanicifyouarealittlebit
over;somepassagesmighttakeyoulongerthanothers,buttrytomakeupthegroundasyougo.

Timetospendreading
ThetricktoconqueringtheSATReadingsectionis ndingtheperfectbalancebetweenreadingtimeand
question-answeringtime.Thisisgoingtovarybasedontheindividual,butinanidealworldyouwanttohave
enoughtimetocarefullyreadandunderstandthepassageandenoughtimetoanswereachquestion
thoughtfully.

Formostpeople,thisiseasiersaidthandone.Butyoushouldde nitely gureoutwhetheryouarewastingtoo


muchtimetryingtounderstandeverymorselofwhatyoureadorwhetheryouarereadingtooquicklyandthus
wastingtoomuchtimere-readingorsimplygettinganswerquestionswrongbecauseyoublewthroughthe
passageinamadsprint.

Tohelpyou gureoutyourpattern,wesuggestthatonceyoubecomefamiliarwiththetest,youspendatleast
onepracticesectioncarefullynotingyourtimebothforreadingthepassageandforansweringquestions.


sat.magoosh.com 101

Dontchangeyournaturalpaceorworryabouttheoveralltimelimitforthepassage!Justjotdownhowlongit
tookyoutoreadthepassageandthenhowlongittookyoutoanswerthequestionsforeachone.Ifyouare
takingtheSATunderregulartime,youshouldideallybe nishingthetotaltestwithin60to65minutes.Ifyou
nishmuchunderthat,thenyouarentusingyourfulltimetoreadpassagesoranswerquestionscarefully.Ifyou
nishinmuchmorethanthat,thenyoumightneedtomakesomedecisionsaboutincreasingyourreadingtime,
skippingsomequestions,orevenskippinganentirepassagesoyoucanbemorecarefulontheonesyoudo
do.(Ofcourse,alwaysmakesuretobubbleinguessesforeverything:NOPENALTYFORINCORRECT
ANSWERSonthenewSAT.Happydance!)


sat.magoosh.com 102

ANoteonSATReadingPassages

Beforewegetintotheindividualpassagetypes,letstalkalittleaboutSATReadingpassagesasawhole.SAT
Readingpassagesarentwrittenspeci callyforthetest.Instead,theyretakenfromhighschoolorcollege-level
readingsourcesandadaptedtomakethem tintothe700wordsorsothattheSATlikes.Ofcourse,thatmeans
thattest-makershavetomakesomeprettysigni cantchangestotheoriginaltextscreatingclearintroductions
andconclusionsbecausethesebooksoressayswerentwrittenwiththeSATexaminmind.

Unlessyoureavoraciousreader,theresnotmuchchanceyouregoingtocomeacrossapassagethatyou
recognize(UNLESSitsapassagefromaU.S.FoundingDocumentthatyouvestudiedinschool;thenyoumight
getlucky).Butitsrare,andnomatterhowmuchyoureadbetweennowandthedayofyourSAT,yourenot
goingtobetteryouroddsinanysigni cantway.

SATreadingpassagesareacademic
Generally,SATreadingscomefrombooksthatyoumightreadinhighschoolorcollege,andthatmakessense.
Ofcourse,thereareallsortsofdi erentthingsyoumightstudyincollege.SothatswhythenewSATmakes
sureyouseealittlebitofeverythingfrom ctiontohistorytoscience.

IfyoureadthroughanarticleortwofromTheNewYorker,TheEconomist,orothersimilarpublications,youllget
asenseofthelevelofreadingtheSATexpectsofyou.

SATreadingsarentsuperdense,old,orfullofjargon
Eveniftheyreacademic,SATReadingpassagesaresupposedtobereadableforpeoplewhoarentactuallyin
the eldofstudythatthepassagescomefrom.Soyouwontgetanythingreallyold(e.g.ShakespeareorKant)
orhighlyspecialized(e.g.linguistictheoryfromChomsky).Aslongasyoustayfocusedwhilereading,youllbe
abletounderstandtheinformationinthepassagewithoutanybackgroundknowledge.

FictionontheSAT
TheresalwaysaliteraturepassageontheSAT,butitsnotusuallythekindofthingmostpeoplehavereadfor
funsorry,noHarryPotter.Itsmorelikelytobesomething,well,literary.Thebooksyoureadinhighschoolare
agoodcomparison.WhileyouwontseeTheGreatGatsbyonyourSAT,sincesomanystudentsreaditin
school,itstherighttypeofbook.


sat.magoosh.com 103

PassageTypes

TheLiteraturePassage

OneverySAT,theresone ctionpassagefromU.S.orWorldLiterature(yeah,thatdoesbasicallymean
anythingintheworld,aslongasitswritteninEnglish).Theliteraturepassageisthe rstpassageintheReading
section.

TheSATlikesrelativelyrecent ction,butitsnotunheardoftoseesomethingolder--worksanywherefromthe
nineteenthtotwenty- rstcenturyareprettyfairgame.Theresawiderangeoftimeperiodsandwriting
traditionsthatthestorymightcomefrom.TherearetwothingsyoucanbesuretheReadingpassagewontbe,
though:writtenintotallyantiquatedEnglish(suchasMiltonsParadiseLost)ortakenfromayoungadultseries
(suchasTwilight.Sorry,Bella).

Sometimestheliteraturepassagewillbetheeasiestreadingintheentiresection,andsometimesitwillbethe
hardest.Youllusuallyknowwithinthe rstparagraph.Sometimesyoullthinkthestoryisengaging,whichis
suchanicebonus.Butdontgettoocaughtupinthetale.ThistheSAT,afterall.Questionsarecoming,andwe
needtobereadytoanswerthem.

HerearesometipsonapproachingtheLiteraturepassage:

Readtheintroinformation
BeforeeveryReadingpassageontheSAT,thereareacoupleofsentencesthattellyouwherethetextistaken
fromandgiveyoualittlebitofbackgroundinfoifnecessary.Youshouldalwaysreadthis,especiallywhenitsa
ctionpassage.Forone,ittellsyouits ctionandthata ectswhatyournote-takingstrategywillbe(moreon
thatinamoment).Besidesthat,itmaygiveyousomeimportantbackgroundinfoonthesettingandcharacters
tohelporientyou.Thecopyrightdateisalsoanimportantclueregardingtheeraapassagewaswrittenin,
whichcantellyoualotabouttheauthorandhisorherworld.

Graduallydescribecharacters
Asyouread,keeptrackofthecharactersyoumeet.Therewillonlybeafewmaybetwoorthreesothis
shouldntbetootough.Butasyouread,buildupalistofdescriptionsofthosecharacters.Focusontheir
personalitiesandmotivationsespecially;howdoestheauthorpaintthem?Anyadjectivesyouseetodescribe


sat.magoosh.com 104

theirpersonasareworthunderliningorcircling.Itsgoodtobethinkingabouttheauthorsintentionswhen
youredoingthis.Istheattitudetowardacharacterpositive,negative,orneutral?

Describetherelationshipsbetweenthecharacters
Youwanttobuildupnotjustanimageofeachpersonalitybutadescriptionoftherelationshipsbetweenthem.
Takecarefulnoteofhowcharactersfeelabouteachotherorreacttoeachother.TheSATwillalmostalwaysask
youaboutthis.

Writethesecharactertraitsinyournotes
Jotdownnotesaboutthecharactersalongsidethepassageasyouread.Theydontneedtobeextensive.
Mary=mean-spirited;Susan=naivewillsu ce.Thiswillbeareallyhelpfulguideforwhenyouanswer
questionsaboutcharacters.

Notetheturningpoint(s)
PrettymucheveryliteraturepassageontheSATisgoingtohavesometypeofturningpointwheresomething
happenstoacharacter,acharacterrememberssomethinghappeningtothem,oracharacterhasarevelation.
Thisturningpointisoftencrucialtounderstandingthepointofthestory.Putabigstarbyitwhenyou ndit.Asa
bonus,lookingfortheturningpointhelpskeepyouactivelyengagedinyourreading.

TheNon ctionPassages

AftertheLiteraturepassage,youllseetwoHistory/SocialStudiespassagesandtwoSciencepassages--typically
alternated.Thesepassagesshouldbeapproachedalittledi erentlythantheFictionpassage.Hereswhatyou
needtoknow:

OneoftheHistory/SocialStudiespassagesandoneoftheSciencepassagesisgoingtoincludeagraphic.

Welltalkabouthowtodealwithquestionsongraphicsinthenextsection,butfornow,Iwouldsuggestnot
focusingonthechartorgraphatallwhenyouarereadinguntilyougettothequestion(s)onit.Thequestions
mightbeveryspeci cormoregeneral,andyouneverknowexactlywhattheyaregoingtoask.Thereisgoing
tobealotmoreinformationonthegraphicthanyouneedtoanswerthequestion,sodontwasteyourtimeuntil
thequestiontellsyouexactlywhatyouneedto nd.


sat.magoosh.com 105


OneoftheHistory/SocialStudiespassagesisgoingtobefromaU.S.FoundingDocumentortheGreat
GlobalConversation.

Itsparticularlyimportanttochecktheauthorandthedateonthesepassages,whichwillappearinsmallerfont
beforethepassage.Youmaybefamiliarwiththeauthor(Ohhellothereagain,Dr.MartinLutherKingorHarriet
Tubman!)orthetimeperiodinwhichitwaswritten(Hmmm,1775soundssuspiciouslylikethestartofthe
AmericanRevolution).Althoughthequestionswillneverrelyonoutsideknowledge,somefamiliaritywiththe
authororsituationwillorientyoubeforeyoubeginreadingsoyoucangetmoreoutofthepassagewithout
havingtopiecetogethertheclues.

Justbecauseitsnon- ctiondoesntmeantheauthorwonthaveapersonalperspective.

TheHistory/SocialStudiespassagemaybesomethinglikeamemoir;inthiscase,itmaysoundalmostlikea
ctionpassage.Oritmightbealetteroraspeechthatismakingapersuasive(andpersonal)argument.Ifit
seemsprettypersonal,bepreparedforquestionsthataskabouttheauthorsfeelingsorattitudes.Youcanbet
yourbottomdollartherewillbeoneortwo.

MoreontheHistory/SocialStudiesPassage

Theresarangeofsubjectsthatcanbedrawnoninthesepassages,buttheretendstobeaheavierfocuson
sociology,psychology,economicsandpoliticalscience.Asasampling,theo cialpracticetestsintheCollege
Boardguidehavepassagesonthepsychologyofgiftgiving,ethicaleconomics,publictransportation,theories
oneducationin18thcenturyAmerica,growthofcities,speechesbyVirginiaWoolfandElizabethCadyStanton
ontherolesofwomen,andtheFrenchRevolution.

MoreontheSciencePassage

DontbetoointimidatedbytheSciencepassages,althoughtheywilloftenincludesomejargon,theyarewritten
foreverydaypeople.However,atthesametimedontbelulledintoafalsesenseofbeliefthatastrong
backgroundinsciencewonthelpyououthere.ItsaloteasiertomakesenseoftheoriesontheDNAdouble
helixifyouvestudiedtheminclass.Ifyouareuncomfortablereadingaboutscience,wesuggestreadingsome


sat.magoosh.com 106

articleswrittenfortheeverydaypersononsciencetopics.Scienti cAmerican,NationalGeographic,orDiscover
magazineswillgiveyouagoodfeelforthetypeofpassagesyoumightencounterontheSAT.

TheSciencepassagespullfromarangeoftopicsinthenaturalsciences:thismeansearthscience,biology,
chemistry,andphysics.AsamplingfromtheO cialGuideincludespassagesaboutDNA,theprospectofmining
inspace,thee ectoftheinternetonourbrains,oceanwaves,evolutionofbirds,thedisappearanceof
honeybees,geneticmodi cation,andsourcesofvolcaniceruptions.

TheTakeaway

TheSATcallsmostofthenon ctionpassagesinformationalpassagesbecausethisispreciselywhattheydo:
communicateinformation.Yourjobistodistillthisinformationintoitsmostimportantelements:
1. themainidea
2. anydi erenttheoriesorperspectivesonthetopicpresented
3. theexamplesusedtosupportthetopic
4. theauthorsconclusionaboutthistopic

Ifyoucandothis,youwillbepreparedforalmostallofthequestionsthatfollow.


sat.magoosh.com 107

ThePairedPassage

Soheresthegist:therewillalwaysbeonpairedpassagesetontheSAT(twopassagesaddinguptothetypical
singlepassagelengthof500to750words.)Whatexactlyisapairedpassage?Well,justasitsnameimplies,itis
asetoftwopassageswrittenonasimilartopic.Thepassagesusuallydonotcompletelyagreewithoneanother,
butthisdoesntmeantheyarealwaysonopposingsideseither.Moreoften,therelationshipbetweenthemwill
bemoresubtle.Maybethesecondonepicksuponadetailinthe rstanddescribesitfurther.Ormaybeit
providesapersonalperspectiveonaglobalissue.Inanycase,theseexcerptshavebeencarefullychosenas
passagestocompare,soyoucanassumetherearegoingtobeseveralconnectionsbetweenthem.Thankyou,
CaptainObvious,yousay?Yourewelcome.

Onthepairedpassage,youcanexpectroughly4or5questionstobeonbothpassages.Theotherquestions
willonlypertaintooneortheother.

TheSATsFavoritePairedPassageQuestions

HeresoneoftheSATsabsolutefavoritequestionstoaskaboutbothpassages:

Whichchoicebestdescribestherelationshipbetweenthetwopassages?

Nowyouknow.Andknowingishalfthebattle.Youcanexpectthatalmosteverysinglepairedpassageisgoing
toaskyouthisquestionoravariationofit.Sometimestheanswerchoiceswillhavetodowiththedi erent
perspectivesofthepassages(howdoeseachauthorfeelaboutthetopic?).Sometimestheywillhavetodowith
thecontentorstructureofthepassage(e.g.Passage1takesahigh-levelviewwhilePassage2describesone
exampleindepth).Youshouldbeonhighalerttotherelationshipbetweenthepassagesasyouread,asyou
arealmostguaranteedtoseethisquestion.

OtherpopularSATReadingpairedpassagequestionsinclude:

HowwouldtheauthorofPassage1respondtotheauthorofPassage2?(orviceversa)
Onwhichofthefollowingpointswouldtheauthorsofbothpassagesmostlikelyagree(ordisagree)?


sat.magoosh.com 108

Comparisonquestionsmightbeaboutadetailinthepassages;thesetendtobetheeasiest,althoughyoumight
havetohunttheanswerdown.(e.g.BoththeauthorofPassage1andPassage2describepigeonsasbeing),
butoftentimestheyareabouthigherlevelissues,soyouwanttobetrackingthemainideaofeachpassageand
anysimilaritiesanddi erencesbetweenthemasyouread.Ifyoudothisinadvance,youwillbemuchmore
preparedtoanswerthesynthesisquestionsthatfollow.

ReadOnePassageataTimeifyouStrugglewithReading

Typically,questionsonpairedpassageswillappearinthisorder:1.questionsonlyonthe rstpassage2.
questionsonlyonthesecondpassage3.questionsonbothpassages.IfyouarenotstrongontheReading
section,youcanchunkyourreadingbytacklingPassage1 rstandansweringthosequestionsandthenreading
Passage2andansweringthosequestionsbeforeansweringquestionsonboth.Thiswillhelpyouretainmore
informationandnotgetdistractedbyanswerchoicesthatappearedintheotherpassage.

Readingonepassageatatimeisalsoagreatstrategyifyouarerunningoutoftime,butinthiscase,startwith
whicheverpassagehasmorequestionsonit.

Belowyoull ndanexampleofapairedpassagetopracticewith.Asyouread,trytoanticipatethecomparison
questionsthatmightfollow(guesswhat,therewillone!).

Passage1

Itsapatternasoldastime.Somebodymakesanimportantscienti cbreakthrough,whichexplainsapieceof
theworld.Butthenpeoplegetcaughtupintheexcitementofthisbreakthroughandtrytouseittoexplain
everything.Thisiswhatshappeningrightnowwithneuroscience.The eldisobviouslyincrediblyimportantand
exciting.Frompersonalexperience,Icantellyouthatyougetcaptivatedbyitandsometimesgoo to
extremes,asifunderstandingthebrainisthesolutiontounderstandingallthoughtandbehavior.

Thisishappeningattwolevels.Atthelowbrowlevel,therearetheconferencecircuitneuro-mappers.Theseare
peoplewhotakeprettybrain-scanimagesandclaimtheycanusethemtopredictwhatproductsomebodywill
buy,whatpartytheywillvotefor,whethertheyarelyingornotorwhetheracriminalshouldbeheldresponsible
forhiscrime.


sat.magoosh.com 109

Atthehighbrowend,therearescholarsandtheoriststhatsomehavecalledthenothingbuttists.Human
beingsarenothingbutneurons,theyassert.Onceweunderstandthebrainwellenough,wewillbeableto
understandbehavior.Wewillseethechainofphysicalcausationsthatdetermineactions.Wewillseethatmany
behaviorslikeaddictionarenothingmorethanbraindiseases.Wewillseethatpeopledontreallypossessfree
will;theiractionsarecausedbymaterialprocessesemergingdirectlyoutofnature.Neurosciencewillreplace
psychologyandother eldsasthewaytounderstandaction.

Thesetwoformsofextremismarerefutedbythesamereality.Thebrainisnotthemind.Itisprobably
impossibletolookatamapofbrainactivityandpredictorevenunderstandtheemotions,reactions,hopesand
desiresofthemind.

Passage2

CriticsoffMRIciteitsinabilitytopinpointexactareasofthebrainresponsibleforcomplexemotionalstates.The
thinkinggoesthatifscientistscantidentifyacomplexstateinthebrainthatstateexistselsewhere,insome
nebulousmind.Butthatclaimissimplyfalse.Justbecauseagivenactivityorresponseisspreadacrossthe
braininvolvingmanydi erentregionsratherjustasinglesectiondoesnotmeanitisbeyondunderstanding,
orthatitdoesntexistinthebrainatall.Itjustmeansweneedtoworkhardertodiscernitsunderlying
principlesevenifdoingsoentailsunderstandinghowmanydi erentregionsofthebrainworkintandem.

Inthecurrentbacklashagainstbrainscience,itisalsoimportanttorealizethatneuroimagingisjustoneofmany
toolsusedinneuroscience.Equallyimportantisthefactthatitiswidelyviewedasrudimentaryinitscurrent
statetheequivalentofaone-megapixelcamerawhenwearestrivingtobuildagigapixelcamera.Itseemsall
butcertainthatwewillcontinuetounderstandthebrainbetterastechnologyallowsustozoomintighter,with
greaterprecision.

Buttheideathatthemindisseparatefromthebrainnolongermakessense.Theyaresimplydi erentwaysof
describingthesamething.Totalkaboutthebrainistotalkaboutphysiology,neurons,receptors,and
neurotransmitters;totalkaboutthemindistotalkaboutthoughts,ideas,beliefs,emotions,anddesires.Asan
oldandelegantphraseputsit,Themindiswhatthebraindoes.

Theworstpossibilityofafull-scale,recklessbacklashagainstneuroscience,totheexclusionofthe eldsbest
work,isthatitmightsacri ceimportantinsightsthatcouldreshapepsychiatryandmedicine.Ifcriticsaretoo
pessimisticaboutwhatthefutureholds,theyarerightaboutonething:overthepastdecade,neurosciencehas
becomeover-privilegedasamethodofexaminingthemind.Journalists,courts,andsometimesevenscientists
seemtobelievethatabrainscancanbemoretellingthanapro leofanindividualsbehavior.Perhapsas


sat.magoosh.com 110

neuroscienceprogresses,itispossibleforobjective,physiologicalassessmentofthebraintowinoutasthe
ultimatearbiteroftruthwhenitcomestothemind.Butthatsalongwayo ,ifiteverwillbepossibleatall.For
now,westillneed eldslikepsychologyandpsychiatry,whichtakethemindastheirstartingpoint,ratherthan
thebrain,tocomplementneuroscience.Thebasicelementsofpsychology,likebeliefs,desires,goals,and
thoughts,willlikelyalwaysplayakeyroleinourunderstandingofhumanbehavior,whichiswhyscienceneeds
researcherswhostudythemindeverybitasmuchasitneedsresearcherswhostudythebrain.Ouraimshould
notbetopickthebrainoverthemind,orviceversa,buttobuildstrongerbridgesbetweenourunderstandings
ofthetwo.

PracticeQuestion

Howwouldtheauthorofpassage1regardtheideastatedinpassage2that(Perhapsasneuroscience...the
mind)?

A. Withlittlereservation
B. Withreluctantapproval
C. Withmarkedskepticism
D. Withoutrightenthusiasm

Tips

Toanswerthisquestion,youwillnotonlyhavetoreadbothpassages,butwillalsoneedtohaveagoodideaof
whateachauthoristalkingabout.Heresyourplanofattack:

1.Getthebigpictureofbothpassages(asyouread).
2.Understandhowthepassagesdisagreeand,whenitapplies,howtheyagree(again,asyoureadyou
shouldbeonthelookoutforthis).
3.Answerquestionsbygoingbacktothepassage, ndingrelevantinformation,andthenphrasingaresponse
basedonthetext.

Explanation

TheauthorofPassage1speaksoutverystronglyagainstthenotionthatneurosciencewillbeabletotellus
everythingaboutthemind.Thetwoaredi erent,hebelieves:Thebrainisnotthemindhesays.Inother
words,hebelievesabrainscanwillbeabletotellyouallaboutthebrain,butnotmuchaboutthemindorsuch


sat.magoosh.com 111

subtlestatesasemotion,mood,etc.ThismatchesupbestwithC),whichmeansstrongdoubt.A)isincorrect
becausetheauthorofPassage1doeshavereservationsthatneurosciencewillbeabletoeventuallytellus
everythingaboutthemind.Ifyoupickedthisone,youmighthavemadethemistakeofreadingtheopinionof
thenothingbuttistsastheauthorsopinion.Thenothingbuttistsbelievethatneurosciencewillreplace
psychology,buttheauthordoesnt.B)andD)areincorrectforthesamereason:theauthordoesnotapprove
oftheideathatthebrainwillbecometheultimatearbiteroftruthwhenitcomestothemind,andheiscertainly
notenthusiasticaboutit!

Ifyoustruggledwiththisone,practice,practice,practicewithcomparingtextsand ndingallthesimilaritiesand
di erencesbetweenthem.Itwillgeteasier!


sat.magoosh.com 112

PassageComplexity

IfyouareatallfamiliarwiththeSATReadingtest,youprobablyknowthatthepassagesvaryindi culty.You
mightbreezethroughsomeofthemandthenbesweatingbulletsthroughothersnothavinganyideawhatthe
heckthelastparagraphwastalkingabout.Thisisok.ThisiswhatmakestheSAThard,andyoushouldknow
thattherearealotofotherstudentsstrugglingalongrightbesideyou.But,ifyouareprepared,therecanbea
hugedi erencebetweenyouandtheseotherfreaked-outstudentsgnawingo theirpencilerasersaroundyou,
andthatis,thelevelofpanicyouexperiencewhenyouencounteradi cultreadingpassage.Knowingwhatto
expectcanhelpyoumakestrategicdecisionsaboutwhichpassagestodo rst.

ThenewSATmakesthisalittleeasieronyousinceyoullface5di erentpassages,allaboutthesamelength
andallwiththesamenumberofquestions.SoifyoustrugglewithReading,youcanstartwiththeeasier
passages rstandmakesureyouhavethetimedoasolidjobansweringthosequestions.

Althoughyouwontknowinadvancewhichpassagesaregoingtobeeasierorharder,youdoknowthatthe
SAThasapredetermineddi cultyrangeforthesepassages.Theeasiestpassageisgoingtobeatabouta9th
gradereadinglevelandthehardestpassageisgoingtobeatanearlycollegelevel.Theothersaregoingto
besomewhereinthemiddle.

Youcanapplyoneofthefollowingstrategiestoquicklydeterminewhichpassagesyoushouldtackle rst:

1. Readthe rstparagraph(or rsttwoparagraphsifthe rstparagraphisonlyasentenceortwo).


Particularlyonnon- ctionpassages,theintroductoryparagraphshouldbeoneofthemostreadable
paragraphsofthetext,andthepassagesoftengetmoredenseinthemiddleortwo-thirdsoftheway
through.Soifyoureadthe rstparagraphandthink,Whoa,thisisgoingtobehard,chancesareitisa
hardpassageandyoucancomebacktoitafteryouveknockedo someeasyones.

2. Skimthroughthepassagequickly,readingjustafewselectsentencesthroughout.Youreprobably
betterateyeballingpassagecomplexitythanyouthinkyouare.JustthinkaboutifyourEnglishteacher
handedyouaprintoutofexcerptsfromTheThreeLittlePigsandCrimeandPunishmentwiththetitles
removed.Youcouldprobablytellataglancewhichistheharderpassage,right?Thedi erencesmight
notbesoextremeontheSAT,butchancesareyourgutinstinctaftera10-secondscanmighttellyou
whichpassagestotackle rst.


sat.magoosh.com 113


De nitelydontwastealotoftimemakingdecisions,andifyouvealreadyreadhalfofapassagebefore
decidingitsdi cult,thisisnotthetimetobail.Skippingaroundthetestreadingpartsofpassagesisnotgoing
tohelpyougetanswersonyourbubblesheet,somakequickdecisionsandgoallin.

Anoteonpassagetopics

SomestudentsjustHATE ction.ItistheWORST.Otherstudents ndthemselvessti ingsnoreswhentheyread


aboutscience.Ifyouhavestrongfeelingsaboutgenreorpassagetopics,youmaywanttomakesomeordering
decisionsbasedonthisratherthanpassagecomplexity.Rememberthattherewillbeone ctionpassage,two
history/socialstudiespassages,andtwosciencepassagesoneverytest(hint:the ctiongenerallycomes rst
andthehistory/socialstudiesandsciencepassagesalternate),soifyouvehadabadbreakupwithonesubject
oranotherinthepast,youmightwanttoarrangeyourreadingorderbasedonpersonalpreferenceratherthan
passagecomplexity.


sat.magoosh.com 114

SATReadingQuestionTypes

CommandofEvidence

EvergetagradedessaybackfromyourEnglishteacheranditsbleedingredwithpassive-aggressivequestions
suchasExamples?Support?Evidenceforthis?Howdoyouknow?orthebackhandedcompliment,
Interestingarguments,buttheyneedsupport?

Well,thisispreciselywhatthenewSATistryingtotestwithitscategoryofquestionspertainingtoCommandof
Evidence.Youll ndCommandofEvidencequestionsthroughouttheSAT,butontheReadingtest,theybreak
downintothreecategories:

1.questionsthataskyoutodeterminethebestevidenceinapassageorapairforthe
answertoapreviousquestion

2.questionsthataskyouhowtheauthorofanargumentusesevidencetosupporta
claim(thesearemoregeneralthanthe rstcategory,buttheideaisthesame)

3.questionspertainingtoinformationalgraphics

BestEvidenceQuestions

The rstcategoryshouldbeprettyrecognizabletoyouifyouvelookedatanSATReadingtest:theyarethe
questionsthataskyou,Whichchoiceprovidesthebestevidencefortheanswertothepreviousquestion?
followedbyanswerchoicesquotingdi erentlinesfromthepassage.

Youhavetwobasicapproachesyoucanemploytoanswerthesequestions:1.Ifyourememberwherethe
supportforthatanswerisinthepassage,youcananticipatetheanswerchoice,and nditinyouranswer
choices,butbeverycareful!Alwaysmakesuretocheckeveryanswerchoicetomakesurethereisntabetter
pieceofsupportthatyoumissed.2. ndeachanswerchoiceandmarkitinthepassage:puttingbrackets
aroundtheselectedlinenumbersisagoodwayofhighlightingtheexcerptwithoutscribblingonthepassage
toomuch.Thisway,youcanevaluatethemalltogetherandmakesureyoupickthebestanswer.Thissecond


sat.magoosh.com 115

methodispreferableformoststudents,because,well,theSATistricky,dontletittrickyouintopickingawrong
answerbecauseyouhaventseenthemall.

BecausethereisnoincorrectanswerpenaltyonthenewSAT,youshouldalwaysbubbleinananswerforthese
questions,butifyouareparticularlyweakinReadingorifyouarenotcon dentinyourpreviousanswerchoice,
youmaywanttoquicklyputinaguessandcomebacktothequestionifyouhavetime.Theseonescanbe
di cultandtheyarenotworthagonizingover.

Theupsidetothisquestiontypeisthatyoumight ndthatanevidencequestionwillhelpyou xamistakeyou


madeinthepreviousquestion,butdontcountonit.TheSATisverygoodat ndinganswerchoicesthatwill
matchupwithallthepreviousanswerchoices,soyoumaynotevennotice.

InformationalGraphics

ThenewSATissprinkledallovertheplacewithfunlittlechartsandgraphs,includingontheReadingand
Writingtests.OntheReadingtest,youwillencountertwoinformationalgraphics.Oneononeofthe
History/SocialStudiespassagesandoneononeoftheSciencepassages.Relatedquestionswillaskyoutouse
theinformationpresentedonthegraphicsincombinationwiththeinformationinthetext.Maybethechartwill
presentabargraphofthenumberofbutter iescapturedinspeci careas,forexample,andaquestionwillask
youwhichclaimprovidedinthepassagecouldbesupportedbythegraph.Itsoftennotasscaryasitsounds.
Andyoullonlyseeafewquestionslikethisonthetest,soitsnobigdealifyouarentafan,butyoushould
knowthattheywillbethere.

Wesuggestthatyoudontworrytoomuchaboutstudyingthese gureswhileyouarereading;theyoften
includefarmoreinformationthanisrequiredtoanswerwhateverquestionthetestthrowsatyou.Sowaituntil
yougettothequestionandthenstudythetableorgraphto ndthespeci canswerthequestionrequires.

WordsinContext

TheoldSATtestedalotofdi cultvocabulary.Nomore.Nowthefocusisonhigh-utilityacademicwordsand
phraseswhichbasicallymeanswordsthatareusedinmultiplesubjectsandgenresandwordsthathave
multiplemeanings.


sat.magoosh.com 116


TherearetwotypesofWordsinContextquestions.The rsttyperequiresyoutocorrectlyidentifythede nition
ofaword(rememberthesearetypicallycommonwordswithmultiplemeanings).Thesecondtypewillaskyou
toidentifyhowanauthorusesawordorphrasetoin uencethemeaning,tone,orstyleofapassage.

Heresanexampleofwhatawordincontextquestionlookslike:

Asusedinline22,thewordcontainedmostnearlymeansto
A. sheltered
B. suppressed
C. enclosed
D. incorporated

Containisnotahardvocabword.Soitsnotaboutknowingwhatcontainmeans;itsaboutwhatitmeansin
context.Indi erentcontexts,containcanmeandi erentthings.Soweneedto gureoutwhatitmeansinthis
passage.

Yourstrategyshouldbetogobacktothepassageandreadnotonlythesentencethewordappearsinbutalso
thesentenceaboveandbelowit.Makesurethatyouunderstandhowthatsentenceissupposedtoconnectto
theonesaroundit.

Thenputablankinthesentencewherethewordappears.Goaheadandcrossitout.Comeupwithyourown
wordorphrasethatexpresseswhatthesentenceissaying.Dontworryabout ndingtheperfectword,justget
somethingdownthatcommunicateswhatthesentenceistryingtocommunicate.

Letstryit.

ItmayseemasiftheBubonicPlagueisavestigeoftheMiddleAges.Mosthighschoolstudentshaveread
aboutitanditsmassdevastationofEuropeanlivesinthefourteenthcentury.Butalthoughtheplaguemay
occasionallyhavebeencontained,ithasresurfacedperiodicallyinvariouslocationsaroundtheglobefromthe
6thcenturytothe21stcentury.Itishardlyahistoricalrelic.


sat.magoosh.com 117

IfIreplacethewordcontainedmyself,Imightcomeupwithstoppedorpreventedfromspreading.These
matchupbestwithanswerchoice(B)suppressed.Wecanseehow,inothercontexts,containedmightmean
theotheranswerchoices:Theschoolcontained/shelteredthestudentsduringthetornado.Thesheepwere
contained/enclosedwithinthepen.Thelessoncontained/incorporatedsixmodulesongoodwriting.Butthats
notwhatitmeanshere.

Trynottojuststartpluggingintheanswerchoicesfromtheverybeginning.Thisturnsonthehow-it-soundspart
ofyourbraininsteadoftheanalyticalpartofyourbrain(youcanprobablyguesswhichonetheSATrewards).
Onlyplugwordsinifyouaretotallyconfusedandcantcomeupwithyourword.

RememberthattheSAToftenchoosesasecond(orthirdorfourth)de nitionofawordandnottheonethatyou
aremostfamiliarwith.Soalwaysgobacktothepassage.

Althoughtheyarefarlesscommon,awordincontextquestionmayalsoaskyoutodeterminewhyanauthor
chosetouseacertainwordtoachieveacertaine ect.

Heresanexample:

Inline34,theauthormostlikelyusedthewordheraldedto:
A. echotheideathatthepressreleaseactedlikealivemessenger.
B. emphasizetheglobalacclaimthediscoveryreceived.
C. implythattheannouncementwasfortuitous.
D. highlightthefanfarethepressreleasereceived.

Thesequestionsaretougherbecauseyoucantusethe ll-in-the-blanktechnique.Ratherthesequestionsrely
onanunderstandingofmainideaandtoneandarereallymorelikeanalysisquestions.Soletstalkaboutthose
next:

AnalysisQuestions

YourSATReadingtestwillincludemanyquestionsthatlooksomethinglikethis:

Theprimarypurposeofthispassageisto


sat.magoosh.com 118

Theauthorstonetowardshissubjectis

Whichchoicebestdescribesthedevelopmentalpatternofthepassage?

Andinorderpicktherightanswersforbigpicturequestionsliketheseones,youneedtozoomout.Therearea
lotofdetailsinSATReadingpassages,ofcourse,andnotbeingclearonthemoreimportantonescanreally
throwyouo .Therewillbeacoupleofwronganswersthatfocustoocloselyonspeci cdetailsinthepassage,
whichjustarentuniversalenoughinscope.

Itsprettyeasytogettrickedbythesetypesofquestions,unlessyouhaveamethod.

Sketchingthebigpicture

Ifyoutakeonethingawayfromthissection,itshouldbethis:takenotesaboutthebigpicturewhileyouread.In
additiontokeepingyoufocused,notesalsohelpbygivingyouazoomedoutpicture.Youreonlygoingtonote
themostimportantdetailsandhowtheyrelatetoeachother(thinkingabouttheirfunctionintheoverall
passage)sothatwhenyoulookatthenoteslater,youwontbedistractedbythelittledetails.

Whyzoomingoutisimportant
ImagineIhaveapictureofariver.Itookthepicturewhilesittingonthebank,skippingstonesandeatinga
sandwich.Whatsinthepicture?Water,trees,rocks,sky,moss,bugslotsofstu .ThenIaskyouwhatshapethe
riveris.Isitcurvy?Straight?Whileyoumightseeacurveinthepicture,youdhaveaprettyhardtimesketching
theriversoverallshape.EvenifIgaveyouawholebunchofriversnapshots,determiningtheentirerivers
shapewouldbeprettyhard.
Youdontwantthat;youwantasatelliteimagetoseetheriversshape.Sure,itwontshowthebugs,therocks,
ormysandwich,butitllshowthebigpicture.AndthatshowthesePrimaryPurposequestionswork.Theyre
askingforthemainpointofthepassage,notthedetails.

Makingsureyourereadyforthemainpoint
TakingtherightkindofmarginnotesonyourSATisaskillthattakespractice.Youhavetoremembertoask
yourselfsomequestionstokeepyournotesfocused:Whatsthemainideaofthisparagraph?Howdoesthis
paragraphrelatetothenextone?
Practicethat,andthesebig-picturequestionswillbeacinch.


sat.magoosh.com 119

Bewaryofextremes
Onquestionsaboutmainidea,purpose,ortone,alwaysbewaryofanswerchoicesthatseemtooextremefor
thecircumstances.Theyarealmostalwayswrong.Jotdowntheauthorsperspectiveandtoneattheendof
eachpassageandyoullbemuchbetterequippedtoanswertheseanalysisquestions.

DirectReference/LineReferenceQuestions

Thisquestiontypewilldirectyoutoaspeci cpartofthepassageorevenaspeci cline.Oncewereadthe


passage,wewanttoanswerthequestionourselves.Thatsrightdonotdivestraightintotheanswerchoices
thinkingtheywillo ersalvation.Theanswerchoicesaremeanttotrickyouandcorruptyourinterpretationof
thepassage.Nextthingyouknow,youimaginethepassageissayingsomethingcompletelydi erentfromyour
mini-narrative.Onceyouhaveananswerinyourhead,matchitwithoneoftheanswerchoices.

Letstakealookatafull- edgedexampleforthisquestiontype(remembertocomeupwithyourownanswer
rst!):

Irecentlydugupaphotographofmyselffromfreshmanyearofcollegethatmademesmile.Ihaveon
thewrongshoes,thewrongsocks,thewrongcheckeredshirttuckedthewrongwayintothewrong
slacks.IlooklikewhatIwas:aboysprungfromamiddlebrowburgwhoa ectedasecondhand
preppiness.Ilooknervous.Comparethatimagetoonefrommysenior-classdinner:nowIamattiredin
agraytweedjacketwithagreenplaidbowtieandasensiblebutton-downshirt,allpurchasedatthe
YaleCo-op.Ilookcon dent,albeitstillabitcontrived.

Inthe rstparagraph,thechangetheauthorobservesinhisformerselfcanbestbedescribedasonefrom
A. uncertaintytodespair
B. con dencetoconformity
C. awkwardnesstopoise
D. genuinetophony

Explanation:Inthe rstparagraph,theauthorislookingattwopicturesoneofhimselfasafreshmanatYale,
theotherasasenior.Asafreshmanheiswearing,thewrongsocksshirtslacks.Henotesthatheisnervous,


sat.magoosh.com 120

awarethathedoesnt tin.Intheseniorphoto,heiswearingwithcon denceasuitandshirtboughtfromthe


Yalestore.
Answer(D),awkwardnesstopoise,bestcapturesthistransition.Thisisatoughquestionifyougetdistractedby
thewordsa ectedandcontrivedandaretemptedtoanswer(D)--thisisaclassicSATtrap.Butifyouread
carefully,youllseethattheauthorthoughthewastryingabittoohardbothatthebeginningandtheendofhis
collegecareer,sothetransitionisnotfromgenuinetophony.Thinkthroughyourownanswer rstandyoullbe
lesslikelytobedistractedbywronganswerchoices.

InferenceQuestions

Inferencequestionsareatrickybunch.Wehavetochoosetheanswerthatcanbestbesupportedby
informationinthepassage.Thetrickisnotfallingpreytothoseanswerchoicesthataresomewhatcorrect,but
goalittlebeyondtheinformationinthepassage.

Questionstypicallytrytotrickusinavarietyofways.Manyarentincorrectthatis,nothinginthepassage
directlyrefutesthem.However,theseanswerchoicesassumetoomuch.Theycannotbecompletelybackedup
bythepassage.

Letshavealookatthefollowingchallengingpassageandquestion.

Thatitmeanslittlenow,tomostAmericans,isevidenceofhowstronglylanguagedrivesthe
perceptionofmentalstruggle,bothitssourcesanditsremedies.Inrecentyears,psychiatristshave
developedamorespecializedmedicalvocabularytodescribeanxiety,thecorecomponentofneurosis,
andasaresultthepublichasgainedagreaterappreciationofitsmanydimensions.Butintheprocess
wevelostentirelytheromanceofneurosis,aswellasitsphysicalembodimentarestless,grumbling,
needypresencethatoncefunctionedinthecollectivemindasanearlywarningsystem,aninnervoice
thathedgedagainstexcessiveoptimism.
Intodayseraofexquisiteconfusionpolitical,economicandotherwisetheneuroticwould
beawelcomeguest,nervouscompanyfornervousdays,alwaysreadytoprovidedosesofthatmost
potentvaccineagainstgloominess:wisecracking,urbanegloominess.
Someofthereasonsthatneurotichasfallenoutofcolloquialusageareobvious.Freudian
analysislostitsholdonthecommonconsciousness,aswellasinpsychiatry,andsomeofFreuds


sat.magoosh.com 121

languagelostitspower.Andscientistsworkingtode nementaldisordersbegantosliceneurosisinto
ever nerpieces,likepanicdisorder,socialanxietyandobsessive-compulsivedisorderallevocative
termsthatpercolatedupintocommonusage,nottomentionintoonlineusergroups,rocklyricsandTV
shows.

Accordingtothepassage,itcanbemostreasonablyinferredthattheFreudianschoolofpsychology

A. coinedthetermneurosis
B. wasassociatedexclusivelywiththewordneurotic
C. ultimatelyabandonedtheuseofwordneurosis
D. employedthetermneurosistodescribecertainbehavior

Explanation:

A:ThepassagesaysthatFreudmadethetermneurosispopular,andthatheusedittodescribecertainstates.
However,wedonotknowifFreudcameupwiththeword.
B:Thered agisthewordexclusively.Itmeansonly,andisalmostalwaysastretchinanInference
Questionwetypicallywanttogowithasafeanswer,meaningtheanswerthatdoesntassumetoomuch.To
saythattheFreudianschoolwasonlyassociatedwiththewordneuroticisastretch.
C:Thetermfellintodisuse.ThepassageneversaysthattheFreudianschoolitselfabandonedtheterm.
D:Thisisthesafeanswer.ItissimplysayingthattheFreudianschoolusedthewordneurotic.Heretheselines
backuptheanswer:beingneuroticmeantsomethingmorethanmerelybeinganxious,andsomethingother
thanexhibitingthehysteriaorotherdisablingmoodproblemsforwhichFreudusedtheterm.(Sothisisthe
answer.)


sat.magoosh.com 122

HowtoImproveonNewSATReading

WrongAnswersonSATReading

AlthoughtheSATisperhapslesstrickythanitwasinthepast,thatdoesntmeanyoudontneedtobeonguard.
TheNewSATReadingsectionisinsomewaysevenbetteratluringyouintotemptingwronganswerchoices
becausemanyofthemseemsoplausible.ThenewSATisallaboutcloseandcarefulreading,sotread
cautiouslywhenitcomestotheanswerchoicesandalwaysgobacktothepassage.

HerearesomeofthereasonswronganswersonthenewSATarejustsodarntantalizing:

1.Theyaretruebasedoninformationinthepassage
Sometimesananswerchoiceiswrongbecauseitdoesntactuallyanswerthequestionbeingasked.Sure,you
can ndsupportingtextforthiswronganswerchoice,butsinceitisnotansweringthespeci cquestionbeing
asked,itsnottherightanswer.

2.Theysoundplausible

ThisisparticularlytrueonthenewSATReadingtest.Manyoftheanswerchoiceswillseemtobeontheme
withthemainideaofthetext.Youmayeventhinkyourememberreadingawronganswerchoiceinthe
passage.Thisiswherecarefulreadingcomesin.Youshouldalwaysbegoingbacktothetexttobackupyour
answer;youmay ndthatthepassagesayssomethingalittlebitdi erentfromwhatyouremembered,meaning
thiswronganswerisrelated,butnotapreciserecollectionofthetest.

3.Theyrelatetothepassage(butdistortitsmeaning)

Becareful!Didthepassageactuallysaywhattheanswerchoiceissaying?Ordiditjusttalkaboutsomething
similar?Ordidittalkabouttheoppositeofwhattheanswerchoicesays?Again,alwaysgobacktothetextand
staytruetoyouroverallunderstandingofthemainideaandauthorspurpose.

4.Theyusewordsandphrasesfromthepassage(butincorrectly)


sat.magoosh.com 123

Sometimesyouaregoingtostrugglewithunderstandingapartofthepassage.Ifaquestiondealswiththispart
ofthepassage,youlloften ndyourselfgoingstraightfortheanswersinthehopethattheywillo ersome
guidance.Doingso,however,isdangeroustheSATiswaitingforyou.

Itwilloftentakewordsthatappearinthepassageandthrowthemintoananswer.Butifyouarenotprocessing
theentireanswerchoice,andarejustgraspingontothosefamiliarwords,youarelikelytobecometrapped.

5.Theyaretoospeci cortoogeneral

Particularlyforbig-picturequestionssuchasthemainideaofthepassageoraparagraph,orthepurposeofa
passageorsection,makesureyoudontpickananswerchoicethatonlymentionspartofwhattheentire
paragraph/passageaddressesorsomethingthatisfarbiggerthanthescopeofthepassage.

6.Theyseemok(butnotthebest)

Thisoneisspeci ctotheCommandofEvidencequestionsyoull ndonthenewSAT,theoneswhereyouhave


topickthebestlinesofevidenceinthetexttosupportyouranswertoanotherquestion.Becarefultocheckall
oftheanswerchoices!Youmightcomeacrossananswerchoicethatseemstobedecentsupportforthe
answertothepreviousquestion(oratleastyoucanrationalizethatitis),buttheremightbeaBETTERlineto
quote.Somakesureyoulookatalloftheoptions,eventhoughitmayfeeltedious.

7.Theyarealmostreferringtotherightline

Thisoneisalsospeci ctotheCommandofEvidencequestionsonthenewSAT.Theanswerchoiceswilllook
somethinglikethis:A.Lines32-35Thewitch.herprey.Sometimesyoumightseeanotheranswerchoicethat
lookslikethis:B.Line32Themonster...themu ns.Inthiscasethesamelinenumberappearsinbothanswer
choices,becausetheirrespectivetextsharealine,andifyouarentbeingcarefulyoumightpickthewrongone,
eventhoughyoucorrectlyidenti edwheretheanswerwas.Sobesupercarefulyouarelookingattheright
lines.

BelowisashortparagraphwithanSATquestionfollowingit.Yourgoalistoreadthepassageandanswerthe
question(duh!),but,moreimportantly,todeterminewhichwronganswerchoicesfallintowhichcategoryabove.
Onceyouvedonethis,lookattheanalysisbelowthepassagetoseeifyouareright.


sat.magoosh.com 124

PracticePassage

ArvoPart'sfamousmusicalcompositionCantusinMemoriamBenjaminBrittenbeginsandendsinsilence.After
threebeatsofstillness,onemusicianringsabellthreetimeswiththeslowsolemnityofadeathtoll.Thesounds
ofsilenceanddeathgivewaytothepurevoiceofstringswhich owsalongintheirwake.Afterasublime,
sorrowfulopeninginAminor,theviolinbeckonsthewarmerCmajorscaleintothepulseofthepiece.The
stringsfollowoneanotheronaquestforthedeepestnote,until nallyeachholdsalong,steadylowC,then
breaksintosilence.ItisjustoneofPart'smoderncompositionsintheclassicalstyle,anditperfectlyrevealsthe
manandhismusic.

PracticeQuestion

Themainpurposeoftheparagraphisto:

A. explainthepurposeofPartsmusic.
B. describetheexperienceoflisteningtoPart.
C. explainhowPartsmusicalcompositionrevealshispersonalfeelings.
D. demonstratetheimportanceofsilenceinmusic.

ExplanationandAnalysis

Firstofall,theanswerisB.Theparagraphdescribesthemovementsofapieceofmusic,focusingonthe
listener'ssubjectiveexperienceasheorshelistenstotheslowsolemnityofadeathtollfollowedbythepure
voiceofstringsandsoon.

Nowletstalkaboutwhatmakeseachoftheotheranswerchoiceswrong,andwhytheyaresuchcommon
wronganswersontheSAT:

A.Yes,theparagraphisaboutPartsmusic,butgobacktothetextandlookcarefully.Isananswerevergiven
astoitspurpose?Iwouldputthisanswerchoiceincategory2:soundsplausible.Youhavetogobacktothe
passageandthinkcarefully.


sat.magoosh.com 125

C.Thereisaphraseattheendoftheparagraphthatmightmakethisonetempting:revealsthemanandhis
music.Andtherearesensorywordsthroughoutthatmightseemtore ectfeelings:sublime,sorrowfuletc.
ButtheparagraphdoesnotsayanythingaboutPartspersonalfeelings.Anditdoesnotsaythatthefeelingsthe
musicmightevokeinlistenersarePartspersonalfeelings,sobecarefulofinferringtoomuch.Thisisacategory
3error:itisbasedinthepassagebutdistortsitsmeaning.

D.Silenceismentionedseveraltimes,makingthisatemptinganswerchoice.Butbecareful!Thisparagraphis
speci callyaboutPartsmusic,notmusicoverall.Thismeansthiswronganswerchoice tsintocategory5;its
toogeneral.

WhenyoupracticeSATReading,andreviewyourpracticetests,trainyourselftolookforthepatternsinwrong
answerchoices.Doingsowillhelpyouavoidmakingthesemistakesontherealdeal.


sat.magoosh.com 126

ReadingSATPassagesFaster

Okay,sopicturethis:itsSaturdaymorningandyouretakingtheSAT.YoureworkingontheReadingtest,and
youvegotabout5minutesleftinthesection.Youthinkyouregolden;yourejustabouttoanswerthelast
questiononyourlastreadingpassage.

Then,afteryoububbleinyouranswer,yourealizeyourescrewed.Onthenextpage,theresanotherpassage.
Yourenotsurehowyoudidntrealizethatbefore,butitsgoingtobeimpossibletoreaditandgetanyofthe
answersbeforetimesup,right?

Maybethisisntsuchahypotheticalsituationforyouweveallfacedmomentsofpaniconatestwhenwe
realizethehourglassisquicklyemptyingout.Butdontletthisparalyzeyou;insteadyoucanswitchgearstoa
specialstrategythatcangreatlyimproveyourchancesofpickingupsomemorepointsonSATreading:speed
reading.

SpeedreadingfortheSAT

Alright,soImnotgoingtotellyouyouregoingtoreadthispostandcomeoutaspeedreadingmaster.AndIm
notgoingtorecommendanycourses,videos,orsoftwarethatdoespromiseyouthat,becauseIhaveahard
timebelievingthatanybodysgoingtogofromreading200wordsperminutetoreading500wordsperminute
andkeeptheirlevelofcomprehension.

Thatbeingsaid,theressomethingtobelearnedfromthecommonspeedreadingwisdom.Firsto ,dont
reread.Minimizeit,atleast.Yourgoalistogetthestructureandkeyinformationfromthepassage,notto
understandeverydetailperfectly.
Secondly,trytoseelargerchunksoftextatonce.Youshouldbelookingatsentencesoratleastsigni cant
chunksofsentencesnotatindividualwords.

Focusontherightpartsofthepassage


sat.magoosh.com 127

SATpassagestendtohavethemainpointinthe rstparagraph,soreadthatparagraphmorecarefully.Each
paragraphisalsogoingtohaveonemoreimportantidea,andthatsmoreoftenthannotbroughtupineitherthe
rstorlastsentenceoftheparagraph.

Sothen,readthe rstparagraph(ortwoiftheyrereallyshort)atyournormal,comfortablepace.Makesureyou
reallyabsorbthatone.

Keepthatpaceforthe rstsentenceofthenextparagraph,butthenspeedup.Ifyourcomprehensiongoes
downabit,thatsalright.Youreonthelookoutforthemainideasofeachparagraphnotthedetails.

Onceyougettotheendoftheparagraph,slowdownagain.Readthatlastsentenceorsomorecarefully,
lookingforhintsaboutthemainpointofthatparagraph.
Afteryou nishtheparagraph(havingonlyskimmedthemiddleofit),askyourselfquestions.Whatdidthe
authorwanttocommunicate?Howdiditrelatetootherparagraphs?andnoteitdown.

Usethequestionsasaguide
Onceyouhavethatoverviewunderstanding,moverightontothequestions.Youregoingtodoalotof
rereadingasyouanswerthem,andthatsthetimetopayattentiontodetailespeciallyifthequestionisasking
youaboutspeci clinesoftext.

Thatswhyyoudontwanttorereadwhileyouregoingthroughthepassagethe rsttime.Youregoingtosee
theimportantpartsagainanyway.

ReadalotofSATpassagesbeforethedayofyourtest
BecausetheSATisstandardized,therearealotofsimilaritiesbetweenpassages.Theyreonsimilartopics,are
fromsimilareras,anduseprettycommonvocabularyandtypicalSATgrammar.Thebestwayyoucanget
comfortablewiththatlanguageandthosetopicsistogetexposed.

Stopsubvocalization
Ifyouarelikemanypeople,yousaythewordsaloudinyourheadwhenyouarereading.Thiscanseriously
hinderyourspeed.Youcanactuallyreadalotfasterwithoutthesubvocalization,butonceithasbecomeahabit,
itsverydi culttoshakeo .Trytoquietdownthatvoiceandletyoureyesdotheworkonthepage.


sat.magoosh.com 128

Followthesetipsand,Idaresay,readingbecomesmuchmorelikeafungame.(Oryoucanatleasttellyourself
that.)

HowtoStayFocusedonSATReadingPassages
Haveyoueverfoundyourselfreadingthesamesentenceorparagraphoverandover?Itdoesnthavetobe
whenyouregoingthroughsomethingasdryasanSATreadingpassage;itmightevenhappenwithsomething
yourereadingforfun,likeamagazine,abook,orablogpost.(Isithappeningnow?)

Evenifweresupposedtobeenjoyingitorweretryingreallyhardtopayattention,sometimesoureyesget
stuckinwhatseemslikeanin niteloop.Welookatthewords,sure,buttheydontdoanything.Theydont
meananything.Sowelookatthemagain,andhuh?stillnothing.Weveallgonethroughwholepageslikethat,
readingonautopilot,thensuddenlyrealizingthatwevesoakedupawholelotofnothingintheprocess.

ThedangerofthishappeningonyourSATishuge,andthereisntasecondtowaste.

WhySATReadingcanbeboring

Personally,wethinkreadingpassagesonthenewSATarefarmoreinterestingthantheyusedtobe.Thetest
hasmovedlightyearsawayinthissense;ontheoldtest,itsometimesalmostfeltlikethetestrewarded
studentswithahightolerancefortediummorethananythingelse.Nowyoumighteven ndthatyoulearnsome
coolthingsasyougo.Whilethisisahugeimprovement,youllprobablystill ndyourselfboredwithsome
topics.Theo cialareasthatSATreadingpassagesdrawfromaresocialsciences,naturalsciences,humanities,
andliterary ction,whichisaprettybroadscope.Thatincludesjustabouteverythingacademic,excludingmath.
(Sorry:noKimKardashianorBrunoMars.)

Sohowdoyoumakeyourselfcare?OtherthanremindingyourselfoftheimportanceoftheSAT,ofcourse.

Howtostaysharp

Thebestwaytostayontaskistofocusontakingmentalsnapshotsasyouread,ortakingbrief,purposeful
notesifyou ndithardtokeeptrackofyourmentalnotes.Butthatdoesntmeantryingtocommiteverything
youreadtomemoryormindlesslycopyingdowndetailsinthemargin.Youshouldbefocusingonwhatthe
functionofeachparagraphisasyoureadthroughthepassage.Withtheexceptionof ction,SATreading


sat.magoosh.com 129

passageswillprettyoftenfollowapredictablepatternofintroducingatopic,explainingsomecontextorhistory,
givingsomespeci cdetailsonthetopic,andwrappingupwithsomemoregeneralthoughtsonthemainpoint.
Thetruthis,thatsmostnon- ctionwritinginanutshell,includingyouressay.

Soyoushouldconstantlybeaskingyourselfquestionslikethese:
Whatsthemaintopicgoingtobe?
Isthisbackgroundinformation?
Whatinformationinthisparagraphisthemostsigni cant?
Doesthisparagraphagreewiththepreviousone?
Willtheauthorreturntohispointinparagraph2?
Doestheconclusionhaveadi erentmessagethantheintroduction?

Constantlyaskingyourselfwhytheauthorwroteeachparagraphandhowitrelatestotherestofthepassageis
thebestwaytostayinvolved.Ifyoukeeptryingtogetblueprintsforeachpassagejotteddowninthemargins,
youllbealotmorestimulated.

Readwithapencilinhandatalltimes.


sat.magoosh.com 130

TakingNotesonSATReadingPassages

ThroughoutthiseBook,wevebeenstressingtheimportanceofpausingandevaluatingwhatyouarereadingas
yougo(takingmentalsnapshots).Heresaquickrecapofeverythingyoushouldbenoticingasyouread:

1. Mainideaoftheentirepassage
2. Mainideaofeachparagraph
3. Authors/narratorspointofview/tone
4. Authorspurpose
5. Structure(howtheparagraphsconnect;transitionwords)

MentalSnapshotsatWork

Aftereachparagraphstopandnoteitsmainidea.

Attheendofthepassagestopandsummarizethemainideaofthepassageasawhole.

Asyougobeawareofhowtheauthorisapproachinghisorhertopic(pointofviewandtone).Iftheauthor
seemstohaveaparticularlystrongopinionorangleonthetopic,chancesaretherewillbequestionsonit.

Asyougonoticethestructureofthepassage:howdoesaparagraphbuildonapreviousone?Isit
developinganexample?Isito eringacounter-argument?Youshouldincludethisinyourparagraphsummaries:
e.g.BeanieBabyexampletosupportprev.paragraphorarguesagainstideathatcollectingisafad.Ifthe
authorstoneisparticularlystrong,noteitattheend,e.g.hesangryorskeptical.

WrittenNotes

Now,weknowthatisalottokeeptrackof.Aswementionedearlier,ifyouneedtotakenotesandworktowards
mentalsnapshots,thatistotally ne.Butyoumay ndwhenitcomestimeforthetest,youarestillinthe
note-takingphase.Ormaybeyou ndthatitsdi cultforyoutokeeptrackofallyourmentalnoteswhenit
comestothelongerReadingpassagesthatdominatethenewSAT.Itstotallyoktousethestrategyofjotting
downsummarynotesontheside.Justbecarefulthatyoudontspendalldayannotatingthepassage--theclock


sat.magoosh.com 131

isticking!A4-6wordsummarynexttoeachparagraphcandothetrick,andremember,theonlyonewhoneeds
tounderstanditisyou.

Ifyoucommityourselftotakinggoodnotes(eitherinyourheadoronpaper),youllneverfacethatterrible
momentagainwhenyoureachtheendofthepassageknowingyoureadtheentirethingbuthaveno
recollectionofwhatyoujustread.Thatscalledpassivereading,anditgetsyounowhereontheSAT(orinlife).

ATipifYouareStillFindingitImpossibletoFocus

SomepeoplehaveaREALLYhardtimekeepingfocusedontheirreading.Therearesomanymoreimportant
thoughtsgoingonintheirheads.Ifyouvetriedtakingmentalsnapshotsand/orjottingdownverybriefsummary
notes(remember,youneedtoatleasttryafewtimes!Theseareskillsthatneedtobehonedbeforeyousee
thembearfruit!),underliningasyoureadmayhelp.

Ifyouunderlineasyouread,dontfreakoutaboutwhetherornotyouareunderliningtherightthings;thevery
actoflookingforwhatyoushouldunderlinekeepsyoufocused,andthatisaWIN.Inacertainsense,it
almostdoesntmatterwhatyouaremarkingup:lookingforwhatyoushouldunderline,ortakeanoteon,keeps
youactivelyreading.Thatbeingsaid,dontunderlineEVERYTHING,becausethenyouarentdoingyourjob.Try
focusingonunderliningthepointswherenewideasorpeopleareintroduced.Thiswillkeepyouengaged,and
youwillgetfarmoreoutofthepassagethanyouwouldotherwise.


sat.magoosh.com 132

PracticePassage

NowthatyouknowjustabouteverythingthereistoknowaboutSATReading,tryyourhandatthissample
passage.AnswersareattheendwithlinkstovideoandtextexplanationsforthesequestionsonMagooshSAT!

EthanFrome

Hehungback,andshecameoutaloneandpausedwithinafewyardsofhim.E1Shewasalmostthelastto
leavethehall,andshestoodlookinguncertainlyaboutherasifwonderingwhyhedidnotshowhimself.E2
Thenaman's gureapproached,comingsoclosetoherthatundertheirformlesswrappingstheyseemed
mergedinonedimoutline.

"Gentlemanfriendgonebackonyou?Say,Matt,that'stough!No,Iwouldn'tbemeanenoughtotelltheother
girls.Iain'taslow-downasthat."(HowFromehatedhischeapbanter!)"Butlookahere,ain'titluckyIgottheold
man'scutterdowntherewaitingforus?"

Fromeheardthegirl'svoice,gailyincredulous:"Whatonearth'syourfather'scutterdoin'downthere?"

"Why,waitingformetotakearide.Igottheroancolttoo.IkinderknewI'dwanttotakearideto-night,"Eady,in
histriumph,triedtoputasentimentalnoteintohisbraggingvoice.

B1/F1Thegirlseemedtowaver,andFromesawhertwirltheendofherscarfirresolutelyabouther ngers.
B2/F2C1Notfortheworldwouldhehavemadeasigntoher,thoughitseemedtohimthathislifehungonher
nextgesture.C2

"HoldonaminutewhileIunhitchthecolt,"Deniscalledtoher,springingtowardtheshed.

G1Shestoodperfectlystill,lookingafterhim,inanattitudeoftranquilexpectancytorturingtothehidden
watcher;Fromenoticedthatshenolongerturnedherheadfromsidetoside,asthoughpeeringthroughthe
nightforanother gure.G2SheletDenisEadyleadoutthehorse,climbintothecutterand ingbackthe
bearskintomakeroomforherathisside.Then,withaswiftmotionof ight,sheturnedaboutanddartedupthe
slopetowardthefrontofthechurch.

"Good-bye!Hopeyou'llhavealovelyride!"shecalledbacktohimoverhershoulder.


sat.magoosh.com 133

A1Denislaughed,andgavethehorseacutthatbroughthimquicklyabreastofherretreating gure.A2

"Comealong!Getinquick!It'sasslipperyasthunderonthisturn,"hecried,leaningovertoreachoutahandto
her.

Shelaughedbackathim:"Good-night!I'mnotgettingin."

BythistimetheyhadpassedbeyondFrome'searshotandhecouldonlyfollowtheshadowypantomimeoftheir
silhouettesastheycontinuedtomovealongthecrestoftheslopeabovehim.HesawEady,afteramoment,
jumpfromthecutterandgotowardthegirlwiththereinsoveronearm.Theotherhetriedtoslipthroughhers;
butsheeludedhimnimbly,andFrome'sheart,whichhadswungoutoverablackvoid,trembledbacktosafety.
D1Amomentlaterheheardthejingleofdepartingsleighbellsanddiscerneda gureadvancingalonetoward
theemptyexpanseofsnowbeforethechurch.D2

IntheblackshadeoftheVarnumspruceshecaughtupwithherandsheturnedwithaquick"Oh!"

"ThinkI'dforgottenyou,Matt?"heaskedwithsheepishglee.

Sheansweredseriously:"Ithoughtmaybeyoucouldn'tcomebackforme."

"Couldn't?Whatonearthcouldstopme?"

"IknewZeenawasn'tfeelinganytoogoodto-day."

"Oh,she'sinbedlongago."Hepaused,aquestionstrugglinginhim."Thenyoumeanttowalkhomeallalone?"

"Oh,Iain'tafraid!"shelaughed.

Theystoodtogetherinthegloomofthespruces,anemptyworldglimmeringaboutthemwideandgreyunder
thestars.Hebroughthisquestionout.

"IfyouthoughtIhadn'tcome,whydidn'tyouridebackwithDenisEady?"

"Why,wherewereyou?Howdidyouknow?Ineversawyou!"

H1Atthispoint,shedroppedallpretenseandtheirlaughterrantogetherlikespringrillsinathaw.Ethanhadthe
senseofhavingdonesomethingarchandingenious.H2Toprolongthee ecthegropedforadazzlingphrase,
andbroughtout,inagrowlofrapture:"Comealong."


sat.magoosh.com 134


Questions

1.Whichchoicebestsummarizesthepassage?

A. Twocharactersareunabletorevealtheirtruefeelingsforeachother.
B. Arebelliouscharacter ndswaystoavoidthosewhoapproachher.
C. Twocharactersmakeapretenseofnotseeingeachotheruntiloneofthemtakestheinitiative.
D. Acharacterisabletoavoidanunpleasantsituation

2.EthanprimarilyperceivesDenisEadyas

A. aslightnuisance
B. apotentialobstacle
C. aworthyrival
D. anunwittingally

3.ThepassageimpliesthatEthanismostworriedthat

A. MattiewillnotreciprocateEadysfeelingsforher.
B. MattiewilldecidetogowithEady.
C. hewillacttooimpulsively.
D. Mattiewillhavetowalkhomealone.

4.Whichchoiceprovidesthebestevidencetothepreviousquestion?

A. A1-A2Denislaughed...retreating gure.
B. B1-B2Thegirl...her ngers.
C. C1-C2Notfor...nextgesture.
D. D1-D2Amoment...thechurch.

5.Formostofthepassage,thereistensionbetween


sat.magoosh.com 135

A. EthanandDenisEady.
B. MattieandEthan.
C. Ethansdesiretoactandhisreluctancetorevealhimself.
D. Mattiesdesireforameanshomeandherdesiretowalkalone.

6.Duringthecourseofthepassage,Ethanspredominantmoodshiftsfrom

A. apprehensiontorelief
B. disconsolationtoecstasy
C. optimismtopessimism
D. re ectivetoboisterous

7.Inthelastparagraph,Ethansattitudecanbestbedescribedas

A. self-congratulatory
B. oblivious
C. insincere
D. unforthcoming

8.WhydoesMattiemostlikelynotallowDenisEadytowhiskherawayonhisfatherscutter?

A. SheishopingEthanwillapproachher
B. ShethinksDennisLeadyisdishonest
C. Shewantstowalkbyherself
D. Sheisconservativebynature

9.Whichchoiceprovidesthebestevidencefortheanswertothepreviousquestion?

A. E1-E2Shewas...showhimself."
B. F1-F2Thegirl...her ngers.
C. G1-G2Shestood...another gure.
D. H1-H2At...ingenious.

10.MattieanswersEthansquestionregardingwhyshedidnotgowithDenisEadybybeing


sat.magoosh.com 136

A. frank.
B. coy.
C. hostile.
D. evasive.

Answers:Clickonthelinksbelowtoseetextandvideoexplanationsforeachquestion!OrgotoMagoosh
SATandsignupforafreetrialfortheseanswerexplanations,morequestions,andlessonvideos!

1. C
2. B
3. B
4. C
5. C
6. A
7. A
8. A
9. C
10. B

Forvideoandtextexplanationsfortheabovequestions(andhundredsmore),checkoutMagooshSAT!


sat.magoosh.com 137

SATMathTest


sat.magoosh.com 138

IntrotoSATMath
WhattoKnowabouttheChangesinNewSATMath

TheSAThasundergoneprofoundchanges:fouranswerchoiceshavereplaced ve,vocabularyexerciseshave
beenbanished,theessaynearlyso(itsoptionalnow),andthesectionsaremuchlonger.Amidstallthischatter,
however,talkabouthowthemathsectionhaschangedhasbeenrelativelymuted.

Whilesomeofthechangestothemathsectionarenotasnoticeableastheabsenceofwordslikeplatitudinous,
theyareequallyprofound.Soifyouareusedtotheoldtestformat,thenyoumightbeinterestedinthefollowing
changes:

1)TheSAThasbecomelessofalogicgame/IQtestandmoreofadirectmeasureofhowwellyouremember
themathyoulearnedinschool.

GoingovermathquestionsontheoldSATinclass,Idusuallygetacoupleofa-hasfrommystudents(sothats
whatitwasaskingfor!)Thetestwaswrappinguprelativelystraightforwardmathwithtrickywordingand
confusingdiagrams.Assoonasyousawwhatthetestwasaskingfororthatonelittletrickneededtosolve
thequestion,thetestbecameeasy.

WiththenewSATifyoudontrememberaspeci cconcept,youarelikelytogettheproblemwrong.Knowthe
conceptandthingswillfallintoplace.Andinsteadofhavingquestionsthatareconfusinglyworded,nowyoull
havequestionsthatsimplyhavelotsofwordstosiftthrough.

2)Therewillbemorehigher-levelmath

Granted,therewontbeawholelotoftrigonometrybutcomparedtothezerotrigontheoldtest,thecoupleof
trigquestionsyoullseetestdayissomethingyoullneedtopreparefor.

Therewillalsobealotofhigher-levelpolynomialsandyouwillhavetofaceo againstacoupleofquestions
thatcanonlybesolvedbyusingthequadraticformula(yes,youllneedtomemorizethis).

3)Therewillbemore--andlongerwordproblems


sat.magoosh.com 139


WhiletheSATwasnevershortonwordproblems,thetestwritershavereallysteppeditupwiththelatesttest,
includingwordproblemsaslongastwelvelines.Inonesection,therewereelevenproblemsinarowthathadat
least5lines.Essentiallyyoumightbedoingmorereadingthanmath-ing.Butifyoupayattentiontothe
underlyingmathtoaquestionyoullbeabletoeliminatealotofunnecessarywords.

4)Onesectionwillprohibittheuseofacalculator

BackwhenIwasyoung,theSATdidntallowacalculatoronanypartofthetest.TimeschangedandtheSAT
caredmoreaboutyourabilitytounderstandmathematicalconceptsthanyourabilitytodoarithmetic.Well,
apparentlytimeshavechangedbackagainsomewhat.Youwillonlybeallowedtouseacalculatorononeof
thetwomathsections.Inthecalculatormathsection,youmightbeexpectedtodostu like gureouttheexact
gureforasumofmoneycompoundedsemiannuallyforfouryears(thesetupwouldlooksomethinglikethis:
100(1.025)8goodluckdoingthatinyourhead!).

Theno-calculatorsectionwillthankfullyo eruptamerspecimensformentalorpen-and-papermath.Still,it
mightbeagoodideatostartdoingmentalmathdrillsorpracticinglongdivisiononpaper.Iknow,thatmight
soundlikeadragbutthatsthewaythetestischanging.Myinnerteacher,however,issayingthatknowinghow
tocrunchbasicnumbersisanessentialfacetoflife,evenwithasmartphoneconstantlyatyour ngertips.So
donttellyourselfyouarelearningaworthlessskill.Or,ifyoucantconvinceyourselfotherwise,knowthatyoull
stillhavetobeonyournumbergamecometestday.

Onelastthing:theno-calculatorsectionisalwaysthethirdsectionofthetest,whichistheshorterofthetwo
mathsections(itcontains20questionsandhasatimelimitof25minutes).

SATMathConceptandSectionBreakdown

Unliketheprevioustestinwhichtheconceptsvariedovertests,theNewSATisveryspeci caboutexactlythe
typesofquestionsthatyoullseetestday.Whilethisknowledgemightseemacademic(WhyshouldIcare?Dont
Ijusthavetogetthequestionright?),knowinghowoftenacertainconceptpopsupwillhelpyouprioritizeyour
preptime.


sat.magoosh.com 140

Trigonometry,oneoftheconceptsthateverybodyisworriedabout,fallsunderasectioncalledAdditionalTopics
inMath.SincethereareafewotherquestiontypesthatfallunderAdditionalTopics,youlllikelyseeingagrand
totaloftwotrigonometryquestions.SobeforeyouknockyourselfoutoverSOHCAHTOAandtheunitcircle,
backawayfromthe600-pagetrigbookandtakeadeepbreath--theretherearefarbetterwaystospendyour
preptime.

Basicfactsaboutthemathsection
TimeAllotted:80minutes
TotalQuestions:58
Calculatorportion:38questions,55minutes(about1:30minutesperquestion)
No-Calculatorportion:20questions,25minutes(1:15minutesperquestion)

Questiontypes:
Multiple-choice(alwayswithfouroptions):45questions
Student-producedresponse(fancyspeakforwhatstheanswer,buddy?):13questions

Concept #ofquestions Percentofthetest

HeartofAlgebra 19 33%

ProblemSolvingandDataAnalysis 17 29%

PassporttoAdvancedMath 16 28%

AdditionalTopicsinMath 6 10%

Hereisahigh-levelbreakdownofeachconcept:

HeartofAlgebra:thisisyourmeat-and-potatoesalgebrathebasicstu .Noexponentsnexttoyourxs.Thisis
whatwecalllinearequations.4x+1=7.Ofcourse,thetestwontaskyoutosolvebasicequationslikethat.
Instead,itllgiveyoureallylongwordproblemsinwhichthesolutionsamountstosomethinglike3n3=12.
Andassumingthatstherightequation,allyoullhavetodoissolveforn.


sat.magoosh.com 141

ProblemSolvingandDataAnalysis:Thisisbasicallythegraphandtablesection:barcharts,piegraphs,tedious
tableswithabunchof guresforyoutosortthrough.Therewillalsobeafairnumberofwordproblemsthatask
anythingfromratiosandpercentstomedianandmode.

PassporttoAdvancedMath:Thisisthepartmostaredreading.High-orderpolynomials,butoftennothingmore
thantheax2+bx+cvariety,willoftenbeburiedunderamassofverbiage,asina12-linewordproblemthatyou
mustsolvingusingapolynomial.Often,youllhaveto ndcreativewaystobalancetheequationandsolvefor
x.

AdditionalTopics:Thisisthefrustratinglyvaguesection,inwhichalltheremaindersgotthrowin.Inno
particularorder,theyaregeometry,coordinategeometry,andtrigonometry.

Inthefollowingpages,Iwillgoovereachsectionindetail.Soifyouarequestioningwhetheralgebrareallyhas
aheart,youllgetthefullscoopbelow.


sat.magoosh.com 142

HeartofAlgebra

Importantstu rst:27ofthe58questions,ornearlyhalfofthequestionswillbeHeartofAlgebraquestions.
Whendevisingthenewformatofthetest,theCollegeBoardseemstohavecomeupwithmorethana
categorizationofconcepts.Utilitariantags,suchasalgebrafundamentals,havebeeninvestedwithapoetic
air.NowwehaveHeartofAlgebra(Icantbuthelpthinkofaplumpcardioidnexttoanunknownvariable).
Butdontbethrowno forevenabeat:HeartofAlgebrasimplymeanslinearalgebra.

Whatexactlyislinearalgebra,youask?Well,anythingthathasanalgebraicequationinwhichnoneofthe
powersnexttoavariableishigherthan1.
3x + 5 =2
y + 5 < 1
Thismightlookprettyeasy,andindeedtheactualcomputationunderpinningthemathisstraightforward.
However,donotthinktheSATisgoingtogiveyouequationsliketheonesaboveandaskyoutosolveforthe
variable.Rather,andthisiswheretheNewSATistryingtodi erentiateitselffromtheoldtest,themathwillbe
wrappedupinlong,real-lifeworldproblemsthatyoullhavetounwrap,i.e.readseveraltimesoverto gureout
whatisgoingon.

HereisanactualexamplefromtheCollegeBoard,foundintheO cialSATStudyGuide:

In2014,CountyXhad783milesofpavedroads.Starting2015,thecountyhasbeenbuilding8milesofnew
pavedroadseachyear.Atthisrate,ifnisthenumberofyearsafter2014,whichofthefollowingfunctionsf
givesthenumberofmilesofpavedroadtherewillbeinCountyX?(Assumethatnopavedroadsgooutof
service.)

Theanswer,C)f(n)=783+8ndoesntevenaskyoutosolveanequation.Rather,youhavetochoosethe
equationthataccuratelymodelstheinformationinthetext.Andthatsreallytheessenceofthetest:
1)Plentyoftexttosortthrough
2)Understandingoftheconcepthiddeninthetext
3)Reallifescenarioswhereyouoftenhavetomatchavariabletoasituationinthe
wordproblem
4)Littletonocomputationorsolvingforvariables(atleastinwordproblems)


sat.magoosh.com 143


Howdoesknowingthisa ectyourprep?Well,dontthinkthatHeartofAlgebrameansyouhavetodoalgebra
drillsalldaylong.YourtimeisbestspentdoingactualSATwordproblems,orwordproblemsthatcaptureallof
thefourelementslistedabove.So,youllwanttolearntothinkintermsofhowequationscanexplainreal-life
scenarios.Inotherwords,canyoutranslatetheinformationintoamathematicalequation?
Beforewediveintosomeactualpracticequestions,itllbeagoodideatoreviewsomealgebrabasics.

Combiningliketerms
Inordertocombinetwoormoreterms,everytermmusthavethesamevariableandthesamepowernextto
thatvariable.Bycombining,Imeanaddingorsubtractingterms.

Hereisanexampleoftermsthatcannotbecombined:
x2 + x
2y + x
m + n + n3 + p
Hereisanexampleoftermsthatcanbecombined(andIvegoneaheadanddonejustthat!)
Similarvariables:
4a + 5b + 3b + 2a = 6a + 7b
x+2y+3z+4y=x+6y+3z
Similarexponents(willneedtoknowforPassporttoAdvancedMath):
x2 + 4x2 = 5x2
x4 + 2x3 + 3x4 + 5x3 = 4x4 + 7x3

Solvingequations

#1Isolatethevariable
2x 5 = 7
Inthisequation,wewanttomakesurexisbyitself.Todothis,wewanttomakesureallthenumbersareonone
sideoftheequalsideandthevariable,assumingthereisonevariable,isontheothersideoftheequalside.

#2-Whatyoucandotoonesideoftheequation,youhavetodototheotherside.


sat.magoosh.com 144

Wewanttomakesurethattheequationalwaysremainsthesame.Therefore,wecantaddorsubtract
somethingtoonesideoftheequationwithoutdoingthesamethingtotheothersideoftheequation.Sincewe
wanttogetridofthenumberonthelefthandside,oratleastmoveittotheotherside,wehavetoadd5to
bothsides,givingusthefollowing:
2x 5 + 5 = 7 + 5
2x = 12
Nowwedividebothsidesby2sothatwecanisolatethex:
2x/2 = 12/2
x = 6

HerenowareafewquestionsthatcapturetherangeofwordproblemsthatyoucanexpectfromtheHeartof
Algebrasection:linearequations,systemsoflinearequations,andlinearfunctions.

PracticeQuestions
1.SteveandBrianaretakingacross-countryroadtrip.Theyhaveagreedtosplitthecostofgasevenly.The
priceofgasis$4pergallon.Iftheminivan,whentravelingataconstantrateof60mph,isabletotravelxmiles
ononegallon,whichofthefollowingexpressionsrepresentsthedollaramountthateachpaysforatripcovering
150milesinwhichtheminivanistravelingataconstantrateof60mph?
A)2(150/x)
B)4(x/150)
C)8(150/x)
D)1(x/150)

Explanation

Thekeytogettingthisquestionquicklyisto gureouthowmanygallonsitwilltaketheminivantotravel150
miles.Isit(x/150)or(150/x)?Youcandoalittlethoughtexperiment.Imaginethatittravels10milesononegallon.
Howmanygallonswoulditneedtotravel150miles?150/10=15.Therefore,wewant(150/x),oreitheranswer
choiceA)orC).Noticethatthetwosplitthecostofgas,whichis$4pergallon.Therefore,itslikeeachispaying
$2pergallon,oranswerA).


sat.magoosh.com 145

Ifyouarestrugglingtogetthat,agoodideaistopluginanumberforxandbacksolve.Soifweusex=10,we
get150/10=15gallonsat$4,whichequals$60.Thetwosplitthat,meaningthateachpay$30.OnlyA)equals
30.

2.
Inthesystemsofequationsbelow,whatisthevalueofxy?
2x 1 = 4(y + 2)
x+4y=6
A)1/4
B)19/4
C)21/4
D)5

Explanation

Wewanttomakesure,whensolvingforxandyinanequationthathastwovariables,thatxandyareonthe
samesideoftheequation.Then,wemultiplyeitherthetoporthebottombyanumberthatwillallowusto
isolateforeitherxory.Wethensolveforthatvariableandplugthatvaluebackintooneofthetwoequations
to ndthevalueofthesecondvariable.
2x 1 = 4y + 8
Thiscanbesimpli edto
2x 4y = 9
Addingthesecondequationunderneathweget:
2x 4y = 9
x + 4y = 6

Addingthetwoequationsvertically,weget
3x + 15
x=5
Pluggingxbackin
5+4y=6
4y=1
y=1/4


sat.magoosh.com 146

x+y=21/4,whichis(C).

3.Whichofthefollowinglinesisnotperpendiculartothelinewithequation2x+5y=-11?
A)5y=2x+3/2
B)y=-5/2x+6
C)10y5=4x
D)-5y=-2x-4

Explanation

Coordinategeometry,whenitdealswithalinearequationliketheoneabove,iscategorizedasHeartof
Algebra.Inthisquestion,weneedto ndalinethatisnotperpendicularto2x+5y=-11.Perpendicularityis
de nedasthenegativereciprocaloftheslope.Theslopeofthatline,oncewemovethingsintothey=mx+b
form,is
2x+5y=-11
2x=-5y11
x=-5/2y11

Thereforetheslopeis-5/2.Thenegativereciprocal,oralinethatisperpendicular,wouldhaveaslopeof2/5.
Alltheanswerchoiceshaveaslopeof2/5exceptanswerchoiceB),whichhasaslopeof-5/2.Therefore,
answerB).

Conclusion

MostHeartofAlgebraquestionsthatyouseeonthe rstmathsection(thiswillalwaysbesection3andwill
alwayscontain20questions)willbemorelikelytogiveyouanequationthatyousolve.Thesecondmath
section(thiswillalwaysbesection4andwillalwayscontain38questions)willcontainwordproblemslikethe
oneIreferencedfromtheCollegeBoardabove.Thereasonforthis,orsomytheorygoes,isthatsection3is
thenocalculatorsection.Solvingsimplealgebraicequationsrequiresverylittlearithmetic.Sure,youmighthave
tosubtractsevenfrom22.Butifdoingsowithoutacalculatorcausesyoutobreakoutinahotsweat,youmight
wanttofocusmoreonmentalmaththanalgebra.


sat.magoosh.com 147

ThemainpointhereisthatmuchofwhatischallengingaboutHeartofAlgebraisthewordproblems.So
practicedoingtheseasmuchaspossible.Theyarelongandthereforerequirelotsofconcentration,something
thatislikelytobreakdownbythetimeyougettothelastsection,which,yep,istheonethathasallthelong
wordproblems.

PassporttoAdvancedMath
TheSAThasgivenusthiswhimsical(andslightlyintimidating)title.Thegoodnewsisthatitisactuallynot
advancedmathbuttheskillsthatunderpinsomeofthemoreadvancedstu youllbedoinginpre-calculus
calculus.Inthislight,thenamepassportdoesntseemasfanciful,sinceinordertotravelinthelandofadvanced
math,youllneedapassportshowingthatyouhavethefundamentalsdown.

Sowhatexactlyaretheseskillsrequiredtoapproachadvancedmath?Well,thenewSATwantsyoutobeable
todealnotjustwithlinearequations,asyoudointheHeartofAlgebrasection,butpolynomials.However,we
aretalkingaboutnotyourbasicquadratic,nopowerhigherthan2,butpolynomialsinwhichthepowerscanbe
veryhigh,asinthefollowing:
256t16 + 81s27
5x4 + 6x3 2cx2 + x + 1
Thatdoesntmeanquadraticswillbeabsent.Butmostwillnotbeyourgardenvarietyx2-2x+1kindbutwill
requireyoutousethequadraticformula(yes,youllhavetomemorizethat).

Othertopicsincludethegraphsofcomplexpolynomials.Thepointofthetest,though,isnttothrowridiculously
complexequationsatyou;itwantstotestyourabilitytorecognizesimplepatternsinallthatcomplexity.Inother
words,canyounavigatearoundallthenoiseto ndarelativelystraightforwardapproachtosolvingthe
question?

Forexample,lookatthefollowingquestion:

Whichofthefollowingisequivalentto 9a8 4a4 ?


A. a4 (9a4 4a)
B. (3a4 2a4 )(3a4 + 2a4 )
C. (3a4 2a2 )(3a4 + 2a2 )
D. a4 (3a 2)(3a + 2)


sat.magoosh.com 148


Thekeyhereisnoticingthattheform(xy)(x+y),orx2y2canbeappliedtotheoriginalequation,sinceboth
9and4areperfectsquaresandtheexponentsareevenintegers,givingus(C).

Hereareafewmorepracticequestionstogiveyouanideaofwhattoexpect.

1.
x + 4 = x + 6
Whatisthesolutionsetfortheaboveequation?
A){-2}
B){-2,-5}
C){-5}
D)Nosolutions.

Explanation

Tosolveforxwewanttoremovethesquarerootsign.Todoso,wehavetosquarebothsides,givingus:
x2 + 8x + 16 = x + 6

Balancingtheequationgivesus:

x2 + 7x + 10
(x + 5)(x + 2)
x=-5and-2

Butwait!

Thisseemslikeaprettystraightforwardquestion,butheresthetwist:wheneveryouhaveasquarerootsign
overthevariableonesideoftheequation,watchoutforthequantityunderthevariableequalinganegative
whenyouplugthevaluebackin.Thereasonisifyougetanegativenumberunderneaththesquarerootsign,
youdonothaveavalidsolution.


sat.magoosh.com 149

Anotherpossibilityinwhichoneofthexsyousolveforisnotvalidisifonesideoftheequationisasquareroot
signandtheothersideoftheequationtheonethatdoesnothaveasquarerootsignisanegativenumber.
Thereasonisthatthesquarerootofanegativenumberwillneverequalanegativenumber.
Thisiswhatisgoingonhere,sincewhenyoupluginx=-5intotheoriginalequationyouendupgettingthe
following:

-1=-1
Thisisnotvalid,sotherefore-5isnotananswer.Only-2is,givingusA).

2.
Ifg(x)=x+5andf(x)=2g(x)1,whatisf(3)?
Student-ProducedResponse:______

Explanation

FunctionsortheapplicationofacertainrulefallsunderPassporttoAdvancedmath.Thesequestiontypes
cantakeonmanydi erentvarieties.Idrecommendpracticingtheseifyouarenotcomfortablewiththem.
Forthisquestion,whenyouseef(3)thattranslateslooselytothefollowing:whereyouseexontherightsideof
theequalssign,placea3.Inthiscase,thexisinsidetheg.Soyouendupgetting:
f(3)=2g(3)1

Sinceg(x)=x+5,whereyouseex,placea3-->g(3)=3+5=8.

Knowingthatg(3)=8,wegobacktotheoriginalequation:
f(x)=2g(3)1=
f(x)=2(8)1=15,whichistheanswer.

CoordinateGeometryandPassporttoAdvancedMath

Coordinategeometry,ormorecolloquiallygraphing,isanotherareawhichis ledunderPassporttoAdvanced
math,aslongandherestheimportantparttheequationisapolynomial.Mostofthetimes,thiswillmeana
parabola.Sometimes,though,youllgetagraphofsomemonstrouspolynomiallikey5+3y4-2y2+1.


sat.magoosh.com 150

Thegoodsnewsisyoullprobablyonlyhavetodecipherthegraphto gureouthowmanytimesitcrosses
throughthex-axisorsomethingelserelativelystraightforward.

Itsreallytheparabolathatisgoingtoshowupmoreoften.Whatyouneedtoknowisthatparabolasare
symmetrical,meaningthateachsideoccupiesthesameareaonbothsidesofeitherthey-axisorx-axis.
Theequationofaparabolacanbede nedasf(x)=ax2+bx+c,wherea,b,andcareconstants(meaningthey
aresome xednumber).Sinceawilloftenequal1,ithelpsknowinggood-oldfashionedFOIL,asthefollowing
questionshows.

Whatisthesumofx-interceptsoftheequationf(x)=x2-6x+8inthexy-plane?
Student-ProducedResponse:_________________

Explanation:
Reallyallwearedoinghereisold-fashionedFOILsothatcan ndoutthesolutionsforx.Thosesolutionsarethe
sameasanx-intercept,sincewhenyouplugeithervalueforxbackintotheequation,f(x),orthey-coordinate,
willequal0.
x2 6x + 8
(x 4)(x 2)
x=4andx=2
Thesumequals6.

Sometimes,thetestmightaskyouto ndsomethingthatrequiresalittlemoreknowledgeofparabolas.One
usefulformisy=(x-h)2+k,wherea,h,andkareconstantsand(h,k)isthevertexoftheparabola.

Sinceapolynomialusuallyisntinthatform,youlloftenhavetogetittherebycompletingthesquareaswell
seeinthenextproblem:

Whichofthefollowingisanequivalentformoftheequationinthexy-plane,fromwhichthecoordinatesofthe
vertexVcanbeidenti edasconstantsintheequation?

A) f (x) = (x 1)2 25
B) f (x) = (x 6)(x + 2)
C) f (x) = (x + 6)(x 2)


sat.magoosh.com 151

D) f (x) = x(x 2) 24

Explanation

Ithelpstoknowthefollowingequationforaparabola: (x h)2 + k, where(h, k)isthevertex.

Gettingtheequationintotheformabovewillhelpusdeterminethevertex.First,weneedtocompletethe
square,becausex22x24doesnotlenditselftothe(xh)2format.

Noticehowthetwo1scanceleachother,therebyleavinguswiththeoriginalequation.Whydidweevenput
the1sinthe rstplace?Wellx^22x+1becomes(x1)^2.Thisiscalledcompletingthesquare,whichIdidby
dividingthequantitythathasthexinit(inthiscase2x)by2andsquaringit.Whatevernumberresults(inthis
casepositive1),Itakethenegativevalueofitandstickitattheendoftheequation(inthiscasethe-1).Sonow
Ihave:

(x 1)2 25

Therefore,hisequalto1andkisequalto-25.Sothevertex(1,-25).Thisquestiondidhaveuswritethevertex
out,butaskedforthewaywecouldbestidentifyit.Thatstheequationdirectlyabove.

ConclusiontoPassporttoAdvancedMath

PassporttoAdvancedMathonlyincludes16ofthe58questionsspreadoutoverthetwomathsections.

However,ifyourealreadycomfortablewiththeothermath(whichmanyare),youshouldspendmoretimeinthis
area.Thereasonisyouarelikelytogetvery usteredbythisquestiontypestestday,andthiscana ectyour
performanceoneasierquestiontypes.

Togetasenseofallthedi erenttypeofconceptsthatpopupinthePassporttoAdvancedMathsection,check
outtheO cialStudyGuide.Takethepracticeteststoseethe16questionspertest.Ifyoumissaquestion
becauseofconceptualmisunderstanding,youllwanttogobacktopages263-276.


sat.magoosh.com 152

ProblemSolvingandDataAnalysis

Covering29%oftheconceptsonthetest,thissectionisthesecondmostcommononthetest,afterHeartof
Algebra.Manyofthesequestionswillbefamiliartoyou,especiallytheonesfromratioandproportion.WhatIve
donehereistoprovideafewpracticeproblemsinsteadofspendingmuchtimewithconceptreview.Thelatter
part,DataandStatistics,coversconceptsthatarenewtotheSAT.Illspendalittlebitmoretimecoveringthese
concepts.

ThisisapartoftheNewSATthathasde nitelybeen eshedoutsincetheoldSAT.Keepinginlinewithreal


worldscenarios,manyproblemswillaskyoutoinferinformationbasedonastudywithanynumberof
participants.ThisisactuallysomethingthattheSAThasneverdonebefore(wellgettothisquestiontypewhen
wegettoitem#3above)andisawelcomechange,giventhatyoullbelearninglotsaboutcauseande ectand
inferentialstatisticsincollege.

Ratio,Proportion,Units,andPercentage

Butfornow,letstalkaboutthe rstpart:Ratio,Proportion,Units,andPercentage.

Thisquestiontypeshouldntcomeasasurprisesinceithasprobablybeenpartofyourmathcoursesforthelast
veyears(yep,youmostlikelywentoverthisstu ,insomeform,allthewaybackinearlymiddleschool).Illstart
withratios.

Ratios

Agoodwaytothinkofratiosisapplesandoranges.SayIhavetwoorangesandthreeapples,theratioof
orangestoapplesis2:3.Seemsstraightforward.WhatifIhave4orangesand6apples?Ifyouanswered4:6,


sat.magoosh.com 153

thatisnotquitecorrect.Youhavetothinkoftheratiothewayyouwouldafractioninlowestterms.Both4and
6canbedividedby2,givingyou2:3.

Thishighlightsanimportantconceptualidea:ratioisnotabouttotalnumber.Itisaboutthenumberofonething
tothenumberofanotherthing,reducedsothattheratioisexpressedastwoprimenumbers.

Letstryafewpracticequestions.

1.Tomissellingapplesandoranges.Theratioofapplestoorangesinhiscartis3:2.Ifhehas12oranges,how
manyapplesdoeshehave?
(A)2(B)3(C)8(D)18(E)30

Solution:

Again,aratioisbasicallyafractionthathasbeenreducedasmuchaspossible.Inthisproblemtheratio3:2,can
berepresentedas3/2.Onewaytosolvethisproblemistosetupasimpleequation:

3/2=x/12

NoticeIplacedthe12,thenumberoforanges,inthedenominator.Wehavetomakesurethatthenumber12
correspondsto2,theorangesintheratio.Solvingforx,weget18(D).

Anevenquickerwayistonoticethatwehave(x6)theoranges(from2wegoto12)sowejusthaveto(x6)the
applesintheratio:3x6=18.

Nowsletstrythesamequestionbutwithaspin:

2.Tomissellingapplesandoranges.Theratioofapplestoorangesinhiscartis3:2.Ifhehasatotalof30fruits,
howmanyapplesdoeshehave?
(A)2(B)3(C)12(D)18(E)30

Solution:


sat.magoosh.com 154

Thisquestion,whileessentiallythesame,istheonethatgivesstudentsalotmoretrouble.Theproblemis
combiningtwoconcepts:ratioandtotal.Todososimplyaddtheratios.Wehave3:2sothetotalis5.

Onewaytosolvetheproblemistosetupthetable.Tablesaregreatbothfromateachersandbeginners
standpoint.Inthiscase,Igettoshowyouanice,tidywayofsolvingtheproblemandyouhaveaneasywayboth
toconceptualizeandsolvetheproblem.

However,onceyoubecomeusedtotables,intheinterestoftime,learntosolvearatiowithoutone(Illshowyou
howtodosoinasecond!).

Apples Oranges Total

Ratio 3 2 5

M(x)

Actual ? ? 30

Whatdowemultiplythetotalratiobytogettheactualtotal?(x6).
Sointhemiddlerowinthetotalcolumnwecanplacea6.

Apples Oranges Total

Ratio 3 2 5

M(x) 6 6 6

Actual 18 12 30

Noticethe(M)x,whichstandsformultiply(youcandispensewiththeM,Ijustdidntwantanyonethinkingthere
isthisrandomvariablex oatingaround).


sat.magoosh.com 155


Nowwemultiplytheapplesandorangesby6toget18and12,respectively.
RememberthefasterwayImentioned?

Addtheratio
Figureoutthex6
Multiply3x6

Theansweris18(D).Alsoremembernottomixupapplesandoranges!

Onelastthingaboutratios.Letssayyouhavearatioof1:2.Thisisnotthesamethingas.Thebottomnumber
inafractionisalwaysthetotal.Thetotalofaratioisalwaysthepartsofaratioaddedtogether.Inthiscase,1:2is
1+2=3.SoifIhave1appletotwooranges,1/3ofthefruitareapplesand2/3areoranges.

Ifyouhavemorethantworatios,makesuretoaddupalloftheratios.Forinstance,iftheratioofbluemarblesto
redmarblestogreenmarblesis2:5:7,bluemarblesaccountfor5/14ofthetotal(2+5+7=14).

Youthinkyougotthat?Well,heresa nalquestiononratios:

Easy
1.TheratioofshirtstoshortstopairsofshoesinKevinsclosetis5:2:3.IfKevinowns10shirts,howmanypairs
ofshoesdoeshehavetogiveawaysothatheendsuphavingthesamenumberofshortsashedoespairsof
shoes?
A)1
B)2
C)4
D)5

Explanation

Okay,thisquestionisslightlyevil,sinceshortssoundslikeshirtsanditiseasytogetthetwomixedupwhenyou
arereadingfast.Soalwayspayattention,evenoneasierquestions!


sat.magoosh.com 156

SinceweknowthatKevinhas10shirts,andthat10thereforecorrespondstothenumber5intheratio,thatthe
actualnumberofshorts,shirts,etc.heownsisdoublethenumberintheratio.Thus,heownsfourshirtsandsix
pairsofshoes.Sohellhavetogiveawaytwopairsofshoessothathellhavethesamenumberofshoesashe
doesshorts.Answer:(B).

Proportions

Onsomequestions,youllhaveto gureouttheproportionbetweentwodi erentunits.

Easyquestion

Nilestakesaninterstateroadtripoverthecourseoftwodays.Ifhecovers610milesinninehoursthe rstday
and300milesinfourhoursonthesecondday,whatishighaveragespeedperhour?
A)55milesperhour
B)65milesperhour
C)70milesperhour
D)75milesperhour

Explanation

To gureouttheaveragespeedoftheentiretrip,dividethetotaldistancebythetotalnumberofhours.The
handyequationD=rt,whereDistotaldistance,risrate,andtistime,willmakethiseasier.

D=910,r=?,t=9+4=13hours.
910=13r,r=70,answer(C).

Di cultquestion

Acartographerownsasquaremapinwhichoneinchcorrespondsto7/3ofamile.Whatistheareaofthemap
insquareinchesifthemapcoversaterritoryof49squaremiles?

StudentProducedResponse:______________


sat.magoosh.com 157

Explanation

Weknowthat7/3ofmile=oneinch.

Wealsoknowthattheareais49squaremiles,meaningthateachside=7:49=7).To ndhowmanyinches
correspondto7miles,wesetupthefollowingequation:

7=7/3x,x=3

Hereisthelittletwistthatyouwanttowatchoutfor.Thequestionisaskingforsquaremilesininches,sowe
havetotake32,whichequals9.

Anotherpossiblequestiontype,andonethatmostarefamiliarwithandprobablydread,isthepercentquestion.
Toreducesomethingbyacertainpercentage,eitherturnthatpercentintoaratioover100orconvertthe
percentintoadecimalbymovingthepointbacktwospaces.Forexample,40%equalsboth40/100and.40.
Forthefollowingpercents,converteachtoafractionandadecimal:

5%=
26%=
37.5%=
125%=

(Answersattheendofthesection.)

Nowherearetwopracticequestions.

Medium

1.Thereare200,000votersindistrictX,60%ofwhomvotedinthe2008stateelection.In2010stateelection,
thenumberofvotersindistrictXincreasedby20%butifonly55%votedinthiselection,howmanytotalvotes
werecastinthe2010stateelection,assumingthatnovotercancastmorethanonevote?
A)12,000
B)120,000
C)132,000
D)176,000


sat.magoosh.com 158

Di cult
2.Inapopulardepartmentstore,adesignercoatisdiscounted20%o oftheoriginalprice.Afternotsellingfor
threemonths,thecoatisfurthermarkeddownanother20%.Ifthesamecoatsellsonlinefor40%lowerthanthe
originaldepartmentstoreprice,howmuchpercentlesswouldsomebodypayiftheyweretobuythecoat
directlyonlinethaniftheyweretobuythecoatafterithasbeendiscountedtwiceatthedepartmentstore?
A)4%
B)6.25%
C)16%
D)36%

Explanations:
1.The#ofvoterswhovotedin2008electionisequalto200,000x60=120,000
In2010,thenumberofoverallvotersincreasedby20%,so20%of200,000is40,000givingus240,000total
voters.
55%of240,000givesus132,000.Answer(C).

2.Whenyouarenotgivenaspeci cvalueforapercentproblem,use100sinceitiseasiesttoincreaseor
decreaseintermsof%.

1stdiscount:20%o of100=80.
2nddiscount:20%o of80=64.
Online,thecoatsellsfor40%o oftheoriginaldepartmentstoreprice,whichweassumedis100
Onlinediscount:40%of100=60.

Thisisthetrickypart.Wearenotcomparingthepricedi erence(whichwouldbe4dollars)buthowmuch
percentless60(onlineprice)isthan64(departmentstoresaleprice).
Percentdi erence:(6460)/64=1/16=6.25%.AnswerB).

Fractionconversionexercisesanswers
1).05,5/100or1/20(youdontalwayshavetoreduceforquickcalculations)
2).26,26/100or13/50


sat.magoosh.com 159

3).375,375/1000or5/8
4)1.25,5/4

Graphs,Tables,andScatterplots

Imassumingthatmostknowaboutbargraphsandpiecharts.Whileyoumightseeafewofthose,Imgoingto
spendtimeongraphsandtablesthatarentasfamiliar.The rstiscalledthescatterplot.

Scatterplots
UnderstandingscatterplotsisimportantonthenewSATsinceyoulllikelyseesuchaquestion.Thethingisif
youhavenoideawhatascatterplotisyoullmostlikelyanswerthequestion.However,justalittleexposureto
thisseeminglydauntingconceptwillshowyouthatitisnottoodi culttounderstand.

Sowhatsascatterplot?Well,thinkofitthisway.Wheneachdatapoint(eachperson,eachcar,eachcompany,
etc.)givesyouavaluefortwodi erentvariables,thenyoucangrapheachdatapointonascatterplot.Heresan
example.Supposewesurveytenstudentswhocamefromthesamehighschooltothesamecollege.Weask
eachstudentfortheirtotalSATscore(M+CR+W)andtheirGPAinthe rstsemesteroftheirfreshmanyearin
college.Eachstudentappearsbelowasasingledot,thelocationofwhichshowsthatstudentsSATscoreand
rstsemesterGPA.


sat.magoosh.com 160

Asonewouldexpect,theresageneralupwardtrend:studentswithhigherSATscorestendedtoperform
betterintheirfreshmanyearofcollege.Atthesametime,theressomechancevariation:rightinthemiddle,
threestudentsallscoredinthe1700sontheirSATsbut,forwhateverreasons,haddi erentresultsinthe rst
semesteroftheirfreshmanyear.

ABestFitLine

Weseetheresageneralupwardpatterntothisscatterplot.Supposewewantedtomakeapredictionbased
onthatpattern.Forexample,acurrenthighschoolseniorinthishighschool,planningtoattendthissame
college,wouldknowherSATscoreandmightbecuriousaboutherpredictedGPAinherupcomingfreshman
yearofcollege.

Weformalizethispatternbydrawingwhatissometimescalledabest tline.Excelcallsthisatrendline.The
o cialnameinStatisticsistheLeast-SquaresRegressionLine,butyoudontneedtoknowthat.Nordoyou
needtounderstandthemathematicallydetailsofwhythisline,asopposedtoanyotherpossibleline,isinfact
thebest t.

Heresthesamegraphwithabest tline:


Thebest tlineabstractsacommonpatternfromtheindividualdatapoints.Thebest tlinerepresentsthe
expectedrelationship:ifweknowanewstudentsSATscore,then,onaverage,whatwouldwepredictforthat
students rstsemestercollegeGPA?Onestudentappearsalmostexactlyonthebest tline(sometimesadata


sat.magoosh.com 161

pointortwowillbeonthetrendline,andsometimesnonewillbe);inthiscase,wecansaythatstudentsGPAis
moreorlesswhatwewouldexpectfromherSATscore.Thereare vedotsclearlyabovethebest tline:these
vestudentshadhigherGPAsthatwhatwewouldhavepredictedfromtheirSATscores.Fourdotsarebelow
theline:thosefourstudentshad rstsemesterGPAslowerthanwhatwewouldexpect,giventheirSATscore.
Noticethatquestionsoftheformhowmanyindividualshadahigher/lower(y-value)thanwhatwewouldexpect
fromtheir(x-value)?aresimplyaskingyoutocountdotsaboveorbelowthebest tline.

Wealsoneedtomakeadistinctionbetweenpeopleordatausedtogeneratetheline,andthenewdatapoints
predictedbytheline.Inthiscase,weused10peopletogeneratethebest tline.Wehavenopredictionsto
makeaboutthose10people:boththeirSATscoresand rstsemesterGPAsareknown,nowthingsofthepast.
Ifweareaskedforthenow-completed rstsemesterGPAofthepersonwhohada1780SATscore,welookfor
thatdot:thatsthelowdotinthemiddleofthegraph,withavalueof2.7fortheGPA(toomuch rstsemester
partyingforthatperson?)Averydi erentquestionis:supposeanewperson,ahighschoolsenior,hasa1780
SATscoreandwouldliketopredicther rstsemestercollegeGPA.Foraprediction,wearelookingnotatany
individualpointbutattheline:thelinehasay-coordinateofabout3.2there,so,onaverage,wewouldpredict
GPAofabout3.2forthiscurrenthighschoolsenior.

Thepastarethedots,thefutureistheline.

Atableissimplyanarrangementofinformationingridform.Butifyouarentusedtothissetupyoumightwant
topracticeafewsuchquestions.

Time(hours) NumberofbacteriaA NumberofbacteriaB

0 9x104 9x105

1 4.5x106 8.1x106

2 2.25x10n 7.29x10t

3 ? ?


sat.magoosh.com 162

TheNewSATisfondaboutthingsgrowing,whetherbacteria(asinthisproblem)orbisoninanationalforest.A
tableisaperfectwaytorepresentthisinformation.Asyoucanseebelowtherearetwoquestionsfollowingthe
table.TheNewSATwilloftenhaveasetoftwoquestionsthatbothrefertothesamechartortable,aswiththe
nexttwoquestions.

VeryDi cult

1.Whichofthefollowingconclusionscanbedrawnbasedontheinformationabove?
A)t>n
B)t=n
C)Afterthreehours,thenumberofbacteriaAwillbelessthanthatofbacteriaB.
D)ThenumberofbacteriaAwillsurpassthenumberofbacteriaBclosertothetwo-hourmarkthanthe
three-hourmark.

VeryDi cult

2.WhichofthefollowingmodelsthenumberofbacteriaA,N(t),afterthours?
A)N(t)=90,000+ 10t+2
B)N(t)=90,000+50t
C)N(t)=90,000(50t)
D)N(t)=90,000 (50)t

Explanations:

1.WhenanalyzingthenumberofbacteriaofAnoticehowtheexponentincreasesby^2andthenumberthatis
multipliedtothebase10ishalved.Sobasically,x10^2=50.Therefore,n=8andt=7.SowecaneliminateA)
andB).Atthethreehourmark,bacteriaAwillbe10^8multipliedbysomethingwhilebacteriaBwillonlybe10^7
multipliedbysomethingsoC)isout.ThatleavesuswithD).NoticethatthenumberofbacteriaAsurpassesthe
numberofBbefore2hours.Therefore,D)hastobecorrect.

2.Fromtheexplanationfor#1,weknowthatbacteriaAincreasesbyx50aftereachhour.Thatcanbe
representedby50^t.Iftheoriginalis90,000thatgivesusD).


sat.magoosh.com 163

DataandStatistics

Histograms
Thefollowingisanexampleofahistogram.Lets rststarto withapracticeproblemtoseeifyoucaninfer
whatahistogramisallabout.Ifyoustruggle,readtheconceptsbehindthehistogramandthentakeanother
stabattheproblem.

Whichofthefollowingistrueregardingthehistogrambelow?

A)ThemeannumberofhoursofTVwatchedlastweekisgreaterthanthemediannumberofhoursofTV
watchedlastweek.

B)ThemediannumberofhoursofTVwatchedlastweekisgreaterthanthemeannumberofhoursofTV
watchedlastweek.

C)Thenumberofstudentswhowatchedmorethan30hoursoftelevisionisgreaterthanthenumberof
studentswhowatchedlessthan5hoursofTV.

D)ThemeannumberofTVwatchedlastweekisgreaterthan20hours.

Inasurvey,86highschoolstudentswererandomlyselectedandaskedhowmanyhoursoftelevisiontheyhad
watchedinthepreviousweek.Thehistogrambelowdisplaystheiranswers.


sat.magoosh.com 164


Histogramsarenotsimplebarorcolumncharts.Ahistogramshowsthedistributionofasinglequantitative
variable.Here,weaskeachhighschoolstudent,HowmanyhoursofTVdidyouwatchlastweek?,andeach
highschoolstudentgivesusanumericalanswer.Afterinterviewing86students,wehavealistof86numbers.
Thehistogramisawaytodisplayvisuallythedistributionofthose86numbers.

Thehistogramchunksthevaluesintosectionsthatoccupyequalrangesofthevariable,andittellshowmany
numbersonthelistfallintothatparticularchunk.Forexample,theleft-mostcolumnonthischarthasaheightof
13:thismeans,ofthe86studentssurveyed,13ofthemgaveanumericalresponsesomewherefrom1hrto5hrs.
Similarly,eachbartellsushowmanyresponseswereinthatparticularrangeofhoursofTVwatched.

Themedian
Themedianisthemiddleofthelist.Here,thereisanevennumberofentriesonthelist,sothemedianwould
betheaverageofthetwomiddletermstheaverageofthe43rdand44thnumbersonthelist.Wecantell
thatthe rstcolumnaccountsforthe rst13peopleonthelist,andthatthe rsttwocolumnsaccountforthe rst
13+35=48peopleonthelist,sobythetimewegottothelastpersononthelistinthesecondcolumn,we
wouldhavealreadypassedthe43rdand44thentries,whichmeansthemedianwouldbesomewhereinthat
secondcolumn,somewherebetween6-10.

Themean
Tocalculatethemean,wewouldhavetoadduptheexactvaluesofall86entriesonthelist,andthendivide
thatsumby86.Inahistogram,wedonothaveaccesstoexactvalues:weonlyknowtherangesofnumbers
forexample,thereareseventeenentriesbetween11hrsand15hrs,butwedontknowexactlyhowmany
studentssaid11hrs,howmanysaid12hrs,etc.Therefore,itisimpossibletocalculatethemeanfroma
histogram.Noonewillaskyoutodothat.Noonecouldreasonablyexpectyoutodothat,preciselybecauseit
is,infact,impossible.

Medianvs.Mean
Ifitsimpossibletocalculatethemean,thenhowintarnationcantheGREexpectustocomparethemeantothe
median?Well,hereweneedtoknowaslicklittlebitofstatisticalreasoning.Considerthefollowingtwolists:


sat.magoosh.com 165

ListA={1,2,3,4,5}
median=3andmean=3

ListB={1,2,3,4,100}
median=3andmean=21

InchangingfromListAtoListB,wetookthelastpointandsliditoutonthescalefromx=5tox=100.We
madeitanoutlier,thatisapointthatisnoticeablyfarfromtheotherpoints.Noticethatmediandidntchange
atall.Themediandoesntcareaboutoutliers.Themediansimplyisnota ectedbyoutliers.By,contrast,the
meanchangedsubstantially,because,unlikethemedian,themeanissensitivetooutliers.

Now,considerasymmetricaldistributionofnumbersitcouldbeaperfectBellCurve,oritcouldbeanyother
symmetricaldistribution.Inanysymmetricaldistribution,themeanequalsthemedian.Now,consideran
asymmetricaldistribution:iftheoutliersareyankedouttooneside,thenthemedianwillstayput,butthemean
willbeyankedoutinthesamedirectionastheoutliers.Outlierspullthemeanawayfromthemedian.

Therefore,ifyousimplynoticeonwhichsidetheoutlierslie,thenyouknowinwhichdirectionthemeanwas
pulledawayfromthemedian.Thatmakesitveryeasytocomparethetwo.Thecomparisonispurelyvisual,and
involvesabsolutelynocalculationsofanysort.(Yes,sometimesyoucandomathsimplybylooking!)

Practiceproblemexplanation

1)Ifyouthinkyouhavetocalculateboththemedianandthemean,thenthisquestionwouldbeimpossible,
sinceitsimpossibletocalculatethemeanfromahistogram.Ifyouknowthetrickdiscussedabove,thenallwe
havetonoticeisthattheoutliers,thepointsmostdistantfromthecentralhump,areattheupperend.Theyare
onthehighsideofthehoursscale.Themedianprobablyjustsitsinsidethatcentralhump,butthemeanhas
beenpulledawayfromthemedianinthedirectionoftheoutliers,thatis,inthedirectionofthehighsideofthe
scale.Thatmeans,themeanishigheruponthehoursscalethanisthemedian.Thatmeans,themeanis
greaterthanthemedian.Answer=A

Notice,thissolutioninvolveszerocalculations.Itis100%visual.Forinstance,youcaneliminateD)bylookingat
thebarsandnoticingthatthevastmajorityofstudentsspendfarlessthan20hourseachweekwatching
television.


sat.magoosh.com 166


SubjectsandTreatments

Thisisnotano cialtitlebutthenamewearegivingtoquestionsthatdealwithstudiestryingtodetermine
causeande ect.Imguessingthatsoundsprettyvague,sohereisapracticequestion.
Ahighschooltrackcoachhasanewtrainingregimeninwhichrunnersaresupposedtoexercisetwicea
weekridingastationarybikeforonehourinsteadofdoingaone-hourruntwiceaweek.Hertheoryis
that,bybiking,studentswillnotoverlyexerttheirrunningmusclesbutwillstillexercisetheir
cardiovascularsystem.Totestthistheoryoutshehadhervarsityathletes(thefasterrunners)incorporate
thebikingregimenandthejuniorvarsityathletes(theslowerrunners)incorporateusualtraining.After
threeweeks,thetimesofhervarsityathletesona3-milecoursedecreasedbyanaverageof1-minute,
whereasherjuniorvarsityathletesdecreasedtheirtimeonthesame3-milecoursebyapproximately30
seconds.

Whichofthefollowingisanappropriateconclusion?
A)Theexercisebikeregimenledtothereductionofthevarsityrunnerstime.
B)Theexercisebikeregimenwouldhavehelpedthejuniorvarsityteambecomefaster.
C)Noconclusionaboutcauseande ectcanbedrawnbecausetheremightbefundamentaldi erences
betweenthewaythatvarsityathletesrespondtotrainingingeneralandthewaythatjuniorvarsityathletes
respond.
D)Noconclusionaboutcauseande ectcanbedrawnbecausejuniorvarsityathletesmighthavedecreased
theirspeedonthe3-milecoursebymorethan30secondshadtheycompletedthebikingregimen.
Beforeweanswerthisquestion,letstalkaboutrandomization.Theideaofrandomizationistheessencethe
beatingheartofdeterminingcauseande ect.Ithelpsusmorereliablyanswerthequestionofwhethera
certainformoftreatmentcausesapredictableoutcomeinsubjects.

Randomizationcanhappenattwolevels.Firsto ,whenresearchersselectfromthepopulationingeneral,they
havetomakesurethattheyarenotunknowinglyselectingacertaintypeofperson.Say,forinstance,thatIwant
toknowwhatpercentofAmericansuseInstagram.IfIwalkonacollegecampusandaskstudentsthere,Imnot
takingarandomizedsampleofAmericans(thinkhowdi erentmyresponseratewouldbeifIdecidetopollthe
audienceofretiredadults).

Ontheotherhand,ifIhadeverybodylivingintheU.S.droptheirnamesintoagigantichatthesizeoftheGrand
Canyon(therearemanyofthose oatingaround!),andifwehadapersonpickanamefromthehat,afterthehat


sat.magoosh.com 167

hadbeenadequatelyshu ed,wewouldhaveamethodthatisbothrandomizedandinclusiveoftheentire
population--thoughinpracticethismethodisaboutaslikelytopullo asNASAlandingabrontosaurusonMars.

Now,ifweweretodrawanamefromthegigantichatandaskthemabouttheirInstagramuse,our ndings
wouldalignfarbetterwiththegeneralpopulation.Therefore,thismethodwouldallowustomake
generalizationsaboutthepopulationatlarge.

However,whendealingwithcauseande ectinastudy,orwhattheSATcallsatreatment,researchersneedto
ensurethattheyrandomlyselectamongsttheparticipants.Imaginethatwewantedtotestthee ectsonthe
immunesystemofanewca einatedbeverage.Ifresearchersweretobreakour100subjectsintounder-40and
over-40,theresultswouldnotbereliable.Firsto ,youngpeopleareknowntogenerallyhavestrongerimmune
systems.Therefore,oncewehaverandomlyselectedagroupforastudy,weneedtofurtherensurethat,once
inthestudy,researchersrandomlybreakthesubjectsintotwogroups.Inthiscase,thosewhodrinkthe
newfangledbeverageandthosewhomustmakedowithaplacebo,orbeveragethatisnotca einated.

Atthispoint,wearelikelytohaveagroupthatisbothrepresentativeoftheoverallpopulationandwillallow
ustodrawreliableconclusionsaboutcauseande ect.

Anotherscenario,andthiswillhelpusseguetothepracticequestionabove,aretreatments/trialsinwhichthe
subjectsarenotrandomlychosen.Forinstance,inthequestionabouttherunners,clearlytheyarenot
representativeofthepopulationasawhole(Imsuremanypeoplewouldneverdarepeelthemselvesoftheir
couchestosomethingasdaftasrunthreemiles).

Nonetheless,wecanstilldeterminingcauseande ectfromanon-representativepopulation(inthiscase
runners)aslongasthoserunnersarerandomlybrokenintotwogroupsexercisebikevs.usualonehourrun.
Theproblemwiththestudyistherunnercoachdidnotrandomlyassignrunners,butgavetheslowerrunners
onetreatment.Therefore,theobservedresultscannotbeattributedtothebikeregimen;theycouldlikelyresult
fromthefactthatthetwogroupsarefundamentallydi erent.Thinkaboutit:avarsityrunnerisalreadythefaster
runner,onewhoislikelytoimprovefasteratrunningathree-milecoursethanhisorherjuniorvarsityteammate.
Therefore,theanswerisC).


sat.magoosh.com 168

WhileD)mightbetrue,andjuniorvarsitysubjectsmighthavebecomefasterhadtheybeeninthebikegroup,it
doesnthelpusidentifywhatwas awedaboutthetreatmentinthe rstplace:thesubjectswerenotrandomly
assigned.

Conclusion:
Hereisasummaryofthekeypoints:
1)Resultsfromastudycanonlybegeneralizedtothepopulationatlargeifthegroupofsubjectswasrandomly
selectedfromthepopulationatlarge.
2)Oncesubjectshavebeenselected,whetherornottheywererandomlyselected,causeande ectcanonly
bedeterminedifthesubjectswererandomlyassignedtothegroupswithintheexperiment/study/treatment.


sat.magoosh.com 169

AdditionalTopicsinMath

ForallthecolorfultitlestheCollegeBoardadded,itsu eredalackofcreativityfortherestoftheconcepts
coveredintheSATmath.Whatwehaveiseverythingfromcoordinategeometrytoimaginarynumbers
unceremoniouslydumpedintoadditionaltopics.Illrefrainfromgivinganysnazzydescriptionsandinsteadtell
youtheexacttopicscovered.

Geometry

Triangles
The45-45-90Triangle
Letsstartwiththesquare,thatmagicallysymmetricalshape.Assumethesquarehasasideof1.Cutthesquare
inhalfalongadiagonal,andlookatthetrianglethatresults.


WeknowC=90,becauseitwasananglefromthesquare.WeknowAC=BC=1,whichmeansthetriangleis
isosceles,soA=B=45.LetscallhypotenuseAB=x.BythePythagoreanTheorem,
(AC)2+(BC)2=(AB)2
1+1=x2
x2=2
x=2

Thesideshavetheratios1:1:2.Wecanscalethisupsimplybymultiplyingallthreeofthosebyanynumber
welike:a:a:a2


sat.magoosh.com 170

So,thethreenamesforthistriangle(whichareusefultoremember,becausetheysummarizeallitsproperties)
are
1)TheIsoscelesRightTriangle
2)The45-45-90Triangle
3)The1:1:2Triangle

The30-60-90Triangle

Letsstartwithanequilateraltriangle,anothermagicallysymmetricalshape.Ofcourse,byitself,theequilateral
triangleisnotarighttriangle,butwecancutitinhalfandgetarighttriangle.LetsassumeABDisanequilateral
trianglewitheachside=2.WedrawaperpendicularlinefromAdowntoBD,whichintersectsatpointC.
Becauseofthehighlysymmetricalpropertiesoftheequilateraltriangle,thesegmentAC(a)formsarightangle
atthebase,(b)bisectstheangleatA,and(c)bisectsthebaseBD.


So,inthetriangleABC,weknowB=60,becausethatstheoldangleoftheoriginalequilateraltriangle.We
knowC=90,becauseACisperpendiculartothebase.WeknowA=30,becauseACbisectstheoriginal
60angleatAintheequilateraltriangle.Thus,theanglesare30-60-90.WeknowAB=2,becausethatsaside
fromtheoriginalequilateraltriangle.WeknowBC=1,becauseACbisectsthebaseBD.CallAC=x:wecan nd
itfromthePythagoreanTheorem.
x2+12=22
x2=4-1=3
x=3


sat.magoosh.com 171

Thesidesareintheratioof1:3:2.Thiscanbescaledupbymultiplyingbyanynumber,whichgivesthe
generalform:a:a3:2a.

So,thethreenamesforthistriangle(whichareusefultoremember,becausetheysummarizeallitsproperties)
are
1)TheHalf-EquilateralTriangle
2)The30-60-90Triangle
3)The1:3:2Triangle

AnglesofaCircle

Supposeyoustandatthecenterofacircleandturnaroundsothatyoufaceeachandeverypointonthecircle.
Youwouldturnallthewayaround,whichisanangleof360.Inthissense,awholecirclehasanangleof360.
Ifyoudividedacircleequally,youcouldcalculatetheangleofeachslice.Hereareafewdivisionresultsthat
couldhelpyoutoknowontestday(Imjustgivingtheonesthatcomeoutasniceroundnumbers,nottheones
thatresultinuglydecimals):
360/2=180
360/3=120
360/4=90
360/5=72
360/6=60
360/8=45
360/9=40
360/10=36
360/12=30

ArcsandArclength

Supposewelookatasliceofacircle,likeasliceofpizza.


sat.magoosh.com 172


ThecurvedlinefromAtoB,apartofthecircleitself,iscalledanarc.Thiscorrespondstothecrustofthepizza.
Wecantalkaboutthesizeofanarcinoneoftwoways:(a)itsangle,sometimescalledarcangleorarc
measure,and(b),itslength,calledarclength.Theangleofthearc,itsarcmeasure,isjustthesameasthe
angleatthecenterofthecircle.HereAOB=60,sothemeasureofarcABis60.

We ndthearclengthbysettingupaproportionofpart-to-whole.Theangleispartofthewholeangleofa
circle,360.Thearclengthispartofthelengthallthewayaround,i.e.thecircumference.Therefore:

Here,letssaytheradiusisr=12.Then,thecircumferenceisc=24.Sincetheangleis60,theratioontheleft
side,angle/360,becomes1/6.Callthearclengthx.

1/6=x/24
Cross-multiply:
24=6x
4=x=arclength

Inotherwords,sincetheangle60isonesixthofthefullangleofacircle,thearclengthisonesixthofthe
circumference.

Inthenextpost,Iwilldiscussstraightlinesandcircles.Heresapracticequestion.


sat.magoosh.com 173

PracticeQuestion

1)Intheshadedregionabove,KOL=120,andtheareaoftheentirecircleis.Theperimeteroftheshaded
regionis
(A) 12 + 8
(B) 12 + 16
(C) 24 + 8
(D) 24 + 16
(E) 24 + 24

PracticeQuestionExplanation

1)Thearea144=r2,sor=12.ThismeansKO=12andOL=12,sothosetwosidestogetherare24.The
remainingsideisarcKL.Thewholecircumferenceisc=2r=24.Theangleof120is1/3ofacircle,sothe
arclengthis1/3ofthecircumference.Thismeans,arclength=8,andthereforetheentireperimeteris24+8.
Answer=C.

RectangularSolids


sat.magoosh.com 174

Alsoknownasjustaboxorabrick,theseareprettystraightforward.LetssayasolidhasalengthL,awidthW,
andaheightH.Thevolume,ofcourse,isV=LWH.Thesurfaceareaisalittletrickier:thesolidhastwofaces
thatareLxW,twothatareLxH,andtwothatareWxH,foragrandtotalof:
TotalSurfacearea:2LW+2LH+2WH

Forthespecialcaseofacubewithsides,thevolumeisV=s3,andthesurfaceareasimpli esto6s2
Whatsalittlesubtleraboutthese:thethree-dimensionalversionofthePythagoreanTheorem.


LetssaythatAD=L,DF=WandAB+H,,and.LengthslikeAC,DE,orBEarecalledfacediagonals,andto
ndthem,youwouldjustusetheordinaryPythagoreanTheoremwiththesidesoftherelevantrectangle.Ifwe
were,though,toconstructsegmentAE,thatwouldgothroughtheemptycenterofthesolid.Ingeometry,thats
calledaspacediagonal.Youdontneedtoknowthatvocabulary,butyouneedtorecognizethiswhenit
appears,andyouneedtoknowyoucanapplythethree-dimensionalversionofthePythagoreanTheorem.Lets
saythatAEhasalengthofD.Then:D2=L2+W2+H2

Occasionally,amathproblemwillaskyoutousethisformulatoevaluatethelengthofaspacediagonalofa
rectangularsolid.

Cylinders

Cylindersareveryfamiliarfromeverydaylife:acircleatthebottom,acongruentcircledirectlyaboveit,and
smoothcurvingside(thelateralface)joiningonecircletotheother.ThevolumeissimplyV=(areaofbase)xh
=r2xh.


sat.magoosh.com 175

GeometryFormulaCheatSheet
Herearesomegoodformulasandrulestokeepinmind.

Angles

Arightangleismadeupof90degrees.

Astraightlineismadeupof180degrees.

Iftwolinesintersect,thesumoftheresultingfouranglesequals360.

Polygons

Apolygonisany gurewiththreeormoresides(e.g.,triangles,squares,octagons,etc.).

To ndthetotaldegreesofapolygon:180(n-2),wherenisthenumberofsides.

GeometryFormulas:Triangles

Areaequals(basexheight)

Anisoscelesrighttriangle(45-45-90)hassidesinaratioofx:x:x2

Anequilateraltrianglehasthreeequalside.Eachangleisequalto60degrees.

Anygivenangleofatrianglecorrespondstothelengthoftheoppositeside.Thelargerthedegreemeasureof
theangle,thelargerthelengthoftheoppositeside.

Eachsideofcertainrighttrianglesareintegers(e.g.,3:4:5,5:12:13).

Thelengthofthelongestsidecanneverbegreaterthanthesumofthetwoothersides.

Thelengthoftheshortestsidecanneverbelessthanthepositivedi erenceoftheothertwosides.


sat.magoosh.com 176

GeometryFormulas:Circles

Circumferenceequals2rorD(wherer=radiusandD=diameter)
Area=r2.

Afractionofthecircumferenceofacircleiscalledanarc.To ndthedegreemeasureofanarc,lookatthe
centralangle.

Nowthepropertiesofinscribedsquares(Ifxisthesideofthesquarethediameterofthecirclewillequalx2).

GeometryFormulas:Quadrilaterals

Theareaofasquareiss2(s=side).

Thediagonalsofasquarebisectoneanother,formingfour90degreeangles.

Thediagonalsofarhombusbisectoneanother,formingfour90degreeangles.

Twicethelengthplusthetwicethewidthequalstheperimeterofarectangle.

Theareaofaparallelogramcanbefoundmultiplyingbasexheight(thebasealwaysformsarightanglewiththe
height).

GeometryFormulas:Three-DimensionalShapes

Thesurfaceareaofacubeisequalto6s2,wheresisthesideofthecube.

Thevolumeofacubeisequaltos3

Thevolumeofacubeandthesurfaceareaofacubeareequalwhens=6.

Volumeofacylinderisequaltor2h,wherehistheheightandristheradiusofthebase.

CoordinateGeometry


sat.magoosh.com 177

Theslopeofalinecanbefoundsubtractingtheyvaluesofapairofcoordinatesanddividingitbythedi erence
inthexvalues

To ndthey-interceptpluginzeroforx

To ndthex-intercept,pluginzeroforyandsolveforx

Theslopesoftwolineswhichareperpendiculartoeachotherareintheratioofx:-1/x,wherexistheslopeof
oneofthelines(think:negativereciprocal).

CoordinateGeometry

Herearethebasicsatareallyhigh-level.Thisshouldbefamiliartoyoufrommathclass,evenifyouspenthalf
thetimesnoozing.

Quadrants


sat.magoosh.com 178

ThequadrantsbeginwithI,wherebothxandyarepositive,androtatecounterclockwisefromthere.Notice

Formany,coordinategeometryisalreadyadauntingconcept.Whenaquestiondispenseswiththegraphall
together,studentscanfeelevenmoreataloss.Ifyoufallintothisgroup,donotdespair.Hereisahelpful
guideline:

DoNotAlwaysDrawtheGraph

Thisadvicemayseemcounterintuitive.Afterall,theproblemdidntprovideagraph.Wouldntthe rststepbeto
graphtheproblemout?

Manycoordinategeometryconceptssansgrapharetestingyourconceptualthinking.Takethefollowproblem:

Whichofthefollowinglinesdonotcontaincoordinatepointsthatarebothnegative?
1.y=x+2
2.3y=4x-2
3.x+y=2
4.3y-4x=2
5.x-y=2

Solution:
Theslopeformulaisimportantifthequestionisexplicitlyaskingfortheslope.Whatisoftenmoreimportantis
knowingthatalinewithanegativeslopefromlefttorightslopesdownward.Apositiveslope,unsurprisingly,
slopesupward.

Thinkofitthiswaystartatanegativex-coordinate(say-2)ofaline.Ifyouweretoplaceaballontheline
wouldrolldownthelineasitmoveintopositiveterritoryforthex-coordinate?Ifsothelineisnegative,ifnotthe
lineispositive.


sat.magoosh.com 179

Forthisproblem,wearelookingforalinethatdoesnotpassthroughthethirdquadrantthequadrantinwhich
xandyarebothnegative.Graphthepoint(-2,-2).Thatsinthethirdquadrant.

NowheresthebigconceptualpartanylinethatslopesupwardswillalwayspassthroughQuadrantIII.Graphit
ifyouhavetoorsimplyimaginealineofin nitelengthslopingupwards.Anywayyoutrytodosotherewill
alwaysbetheThirdQuadrantwaitingtoclaimapartofyourline.

Now,imagineadownwardslopingline.Isitalsocrossingthroughthethirdquadrant?Well,movetheentireline
totheright.Atacertainpoint,yourlinewillnolongerbeintheThirdQuadrant.Aslongasthatlinecrossthe
y-axisatapositivevalue,itwillnevercrossthroughtheThirdQuadrant.

Nowyouonlyneedto ndtwothings:alinethathasapositivey-interceptandanegativeslope.Andthatis
muchbetterthanhavingtographeveryoneoftheequationsinanswerchoicesAE!

Onlyanswer(C)x+y=2,whichcanbere-writtenasy=-x+2,hasanegativeslope(-1)andpositivey-intercept
(+2).

Trigonometry

The nalmemberinthetriumvirateof-metry,trigonometryisperhapsthemostfeared.Forone,manystudents
mightbelearningitforthe rsttimemonths,orevenweeks,beforetheytaketheSAT.Ifyoufallintothis
category,dontfear:thetrigonometryonthetestisprettybasic/generalconceptstu .Infact,ifyouarejust
learningtrigonometrythatmightworktoyouradvantagebecausetheconceptswillbefreshinyourhead.And
thequestionsintheexammightseemdownrighteasycomparedtowhatyouarelearninginMs.Ficklebottoms
class.

Forthosewhohavenevertakentrigonometry,hereiswhatyoullneedtoknow.

Firstofall:Whatistrigonometry?

Trigonometryisthe eldofmaththatdealswithtrianglesspeci cally,therelationshipsbetweenthethreesides


andthethreeanglesthatmakeupeverytriangle.


sat.magoosh.com 180

Andtypicallythe rstthingyoustudyinatrigclassarerighttriangles:


Soheresarighttriangle.Letssaythatwearelookingattheanglethearrowispointingto.Thesidenexttoitis
theadjacentside,thesideoppositeitistheoppositeside,andthehypotenuseis,ofcourse,thehypotenuse.
Itisimportantthatyouthinkaboutthesidesthiswaybecausethenextthingyoutypicallylearninatrigclassisa
mnemoniccalledSOHCAHTOA,andtheseAsandOsandHsstandforadjacent,opposite,andhypotenuse.

ButwhatdotheS,C,andTstandfor?

Thenextthingsyouneedtomemorizeabouttrigarethesethreetermsandtheirabbreviations:

Sine(sin)
Cosine(cos)
Tangent(tan)

Thesethreetermsareusedtodesignatetheratioofapairofsidesinatriangle.

SohereiswhereSOHCAHTOAcomesin.Thishelpsyourememberwhichratioiswhich:

sin=opposite/hypotenuse


sat.magoosh.com 181

cos=adjacent/hypotenuse
tan=opposite/adjacent

IsuggestanytimeyouseearighttrianglewithtrigtermsontheSATthatyouwritedownSOHCAHTOAnextto
theproblembecauseitsveryeasytoaccidentallyusethewrongratio.

Example1:
WhatisthesinofA?


KnowingSOHCAHTOA,youwouldbeabletoanswerthatitisopposite/hypotenuseor3/5.Easyasthat!

Example2:
WhatisthelengthofXZ?


sat.magoosh.com 182


KnowingSOHCAHTOAmeansthatifwearegivenarighttrianglewithoneknownlengthandoneknownacute
angle(meaningnottherightangle)wecanalways ndtheothertwolengths.

Sointhiscasewecanusesineto ndthelengthofthehypotenuse.

sin(10)=3/XZ
XZ=3/sin(10)

Wecandividesinof10degrees(makesurecalculatorisindegreesmode)by3inourcalculatortogetthe
answer:approximately17.28.OntheSAT,however,youwontbeexpectedtohaveacalculatorthatcandothis
andsotheanswerwouldlikelybegivenas3/sin(10).

Example3
Heresonethatsjustateensybitharder,butwearejustgoingtoapplythesameprinciples.
Thetreebelowcastsashadowthatis24feetlong,andtheangleofelevationfromthetipoftheshadowtothe
topofthetreehasacosineof4/5.Whatistheheightofthetree?


sat.magoosh.com 183


Theproblemtellsusthatthecosineoftheangleofelevationis4/5.RememberSOHCAHTOA,soweare
concernedwiththeadjacentsideoverthehypotenuse.Thefactthatthecosineis4/5meanstheratioofthe
adjacentsidetothehypotenuseis4/5.Sowecansetupaproportion:

=24/x

Cross-multiplyingtosolveforxgivesusx=30.

Butrememberthatthisisthehypotenuseandweneedto ndthelengthoftheverticalsideto ndtheheightof


thetree.WecanusethePythagoreanTheoremto ndthelengthoftheverticalside.

24^2+b^2+30^2
=18

Sotheheightofthetreeis18ft.

Ifyourecognizedthatwehada3-4-5triangleinthebeginning,youcouldactuallytakeashortcutandjust
usetangentoftheangleofelevationto gureouttheheight.


sat.magoosh.com 184

KnowingSOHCAHTOAjustaboutguaranteesyouwillbeabletonabatleastonetrigquestiononthetest.

TheUnitCircle

THISisaunitcircle.Itsacirclewithradiusof1centeredabouttheorigin.

Thereareacornucopiaoffascinatingaspectstotheunitcircle:Isuggestyouconsulttheinterwebsoryourmath
teacherto ndoutmore.Werejustgoingtogothroughtheabsolutebasicshere,butthatsreallyallyoullneed
toanswermostSATquestionsrelatingtotheunitcircle(ImgoingtostandbythatstatementuntiltheSAT
releasesatoughunitcirclequestionsomethingtheyveyettodo).

TheSATwilltestwhetheryouknowwhereangleslargerthan360degreeslie,andtheunitcirclehelpsus
visualizethis.

Thereare360degreesinacircle,butwecanjustkeepswingingthearmoftheanglearoundcounterclockwise
( justlikethehandsoftheclock)togettoananglebiggerthan360.So,forexample,ifyouwanttoknowwhere
anangleof760wouldbe,youwouldcirclearoundthecircletwice(foratotalof720degrees)andwewould
have40leftoverdegrees.Sothatanglewouldlieintheupperrightquadrantoftheunitcircle(QuadrantI).

TheSATwillalsooftenuseradiansontrigquestions,andtheunitcirclehelpsuswrapourheadsaround
this.


sat.magoosh.com 185

Youshouldknowthat:

90degreesonthecircle= /2
180degrees=
270degrees=3 /2
360degrees=2

TheSATwillalsotestwhetheryouknowwherethesine,cosine,andtangentofanglesarepositiveor
negativeontheunitcircle.


Theresagreatmnemonictohelpyourememberwheretrigfunctionsarepositiveornegative:
AllStudentsTakeCalculus

Thishelpsyourememberthat:

InQuadrant1All(sine,tangent,cosine)arepositive
inQuadrant2onlySineispositive(andcosandtanarenegative)
inQuadrant3onlyTangentispositive(andsinandcosarenegative)
inQuadrant4onlyCosineispositive(andsinandtanarenegative)

Ifthemeasureoftheangleallthewayaroundthecircleonceisx,whatisthevalueofx?


sat.magoosh.com 186

ImaginaryNumbers

Ah,imaginarynumberstheimaginaryfriendsofthemathworld.Butreallyspeakingthatsanunfortunatename,
sincethesenumbersdontexistinsomefantasyrealm,butareactuallyjustadi erenttypeofnumberand,for
SATpurposes,averyhelpfulone:imaginarynumbersallowyoutogetaroundthatpesky
you-cant-take-a-square-root-of-a-negative-numberrule.

Forthemostpartwhatyoullbedoingisnotunderstandingthenicetiesoftheconceptbutapplyingalgebraic
rulesofbalancingtheequation.ThekeytounlockinganyimaginarynumberproblemontheSATisthe
following:

i=-1

Fromthis,wecan gureoutthefollowing:

i^2=-1
i^3=-i
i^4=1


sat.magoosh.com 187

TipsandStrategiesforPreparingforNewSATMath

Withtheincreaseinnumberofconceptsonthetest,itismysensethatmanystudentsaregoingtogetbogged
downintheminutiaeofeachkindsofquestion(gee,IdontthinkIreallyunderstandhowtodividecomplex
polynomialsto gureoutwhattheremainderis).Whatisultimatelythemostimportantishowwellyouareable
tofocusforafewhoursandwadethroughallthelanguagetogetattheanswer.Intheend,youllmissmany
questionsyoucouldhaveansweredhadyourmindnotbeensoworndown.

Whyisthis?Well,youwillnothavelikelydoneasmanytimeddrillsondensewordproblems.Thisisaskillinof
itself,andonethatmostwillendupignoringinfavorofstudyingsomeconceptthathasarelativelylowchance
ofshowinguponthetestandshowingupthewayyouthinkitwill.

Atthesametime,doconceptreviewinweakareas.Andthisisespeciallythecaseifyouarestruggleatmath.
Butremember,thetimepracticesessionsaregoingtomakeyouabettertesttakerandthatswhereyoullbe
abletoseealotofyourscoreimprovement.

Anothergoodstrategy,whendoingdrills,istomakesureyoudomixedpracticesets.Thatis,dontonlyfocuson
algebraorpassporttoadvancedmath.Doactualpracticeteststhathavetherandomvarietyofquestiontypes
thatyoullseetestday.Partofdi cultyisoften guringoutthesubjectthatisbeingtested.Byhavingthatgiven
toyoufromthegetgo,itdullsyourabilitytorecognizethisconcept.

Yet,onelastareainwhichpracticetestswillhelpyouisinthegradingofdi culty.Remember,thetestiswritten
insuchawaysothatacertaindi culty-levelquestionisusuallyatacertainspotinthesection.Thisrelationship
tendstobeprettylinearinthesensethatthehigherthenumberwithinaspeci cmathsection(thedi cultywill
resetwhenyougettoStudent-ProducedResponse(SPR)questions),theharderthequestion.Knowingthiscan
helpyoucatchcarelesserrors.Didthelastquestioninthesectionseemtooeasy?Youmighthavemissed
something.Doesoneoftheearlierquestionsseemimpossible?Makesureyoureadthequestionagain,
becausetheremightbesomevitalinformationthatyouareforgetting.

Finally,makesureyoutakethosepracticetestsaboutonceaweek.Thatwayyoucangetexposuretothe
nuancedwaysthetestisconstructed;youcangetinformationonwhetherornotyouareimproving(andusethis


sat.magoosh.com 188

totweakhowyouvebeenstudying);andyoucangetusedtowhatsgoingtohappentestday:lotsofsittingand
lotsoffocusing.


sat.magoosh.com 189

SATStudySchedules


sat.magoosh.com 190

MakingtheMostofYourSATStudySchedule

The rstchallengeofstudyingfortheSATis ndingthetimeandenergytogetstarted.Thesecondchallenge?


Committingtoyourstudyplan.

Ifyouhaventalreadynoticed,highschoolisagameoftimemanagement.Andof ndingawaytocompleteall
yourhomework,studyforyourtests, nishyourprojects,manageallyourextracurricularactivities,sleep,and
maybeevenhavesometimeleftoverforyourself.

Then,youthrowstandardizedtestsintothemix,andyourwell-organizedschedulefallsapartattheseams.
Dontbeoverwhelmed!Therearewaystomakeyourlifeeasier.

FindinganSATstudyplanthatworksforyou,andstickingtothatplan,canhelpyoustayorganizedand
self-motivatedwhilekeepingprocrastinationatbay.Withagoodstudyplan,youllstudyalittlebiteveryday,
preventingtheweekend-longcramsessionsthatmurderyoursleeppatternandsociallife.

Ratherthancreateyourownstudyguide,fromscratch,checkouttheMagooshSATStudySchedulesonline.
TheylistallthematerialsandresourcesyoullneedtostudyfortheSAT(manyofwhichareevenfree),andthen
giveyouday-by-dayassignmentscoveringallSATtopicsandteststrategies.

HowtoUseanSATStudyGuide

Theresnosuchthingasaone-size- ts-allstudyschedule.However,adaptingastudyscheduletoyourneeds
shouldnttaketoomuche ort.Hereswhatyoudo:

1)HonestlyassessyourSATstrengthsandweaknesses.

DoyoualwaysstrugglewithacertainSATtest(thatswhattheycalleachsection),orwithacertainquestion
type?Focusextratimeandenergyhere.Forexample,youcoulddoextrapracticeproblems,readuponthe
subject,orlearnspeci cstrategiesforoutsmartingtheSATsquestionsonthatsubject.Isyourproblemtime
management?Practicewithatimer,andalwaysanswertheeasiestquestions rst.

2)DeterminehowmuchtimeyoucanrealisticallydevotetoyourSATprep.


sat.magoosh.com 191

Ifyoureworking,playingsports,volunteering,andmaintainingyourcourseloadallatthesametime,you
probablycantcommittoanhourofSATprepeachday.Inthiscase,youmightwanttobreaktheOneMonth
SATStudyScheduleintosmallerunits,andtackleitovertwoorthreemonths.Itsokayifyouneedtoreschedule
yourexamtogiveyourselfmoretimetostudy.Trytosetrealisticgoalsforyourselfitsimportantthatyouhave
timetosleep!

3)Alwayscheckyourworkandunderstandwhereyouaremakingmistakes.

JustdoingpracticeproblemswontimproveyourSATscoreverymuch.Youneedto gureoutwhyyoumissthe
problemsthatyougetwrong,andthenre-dothoseproblemsuntilyougetthemright.Theresnouseinmaking
thesamemistakesoverandoverduringyourSATprepitjustmeansthatyoullgetthesametypesof
problemswrongwhentheyshowupontheSAT.

4)Setsmallgoalsandkeeptrackofyourprogress.

Tellyourselfbytheendoftheweek,IwillbeabletodoSOHCAHTOAproblemswithoutconsultingmynotes.
Or,decidetofollowtheOneMonthScheduledaybyday,andrewardyourselfattheendoftheweekwithatrip
toaguiltypleasurefoodjointoranhourofTVtime.Makesureyoureloggingyourprogressinanotebook,or
onyourcomputer,soyoucankeeptrackofyourgoalsandnotewhichtopicsgiveyouthemosttrouble.

5)IncorporateSATprepintotherestofyourlife.

AchievingyourgoalscoreontheSATtakesmorethanjuststudying.Besuretoeatright,exercise,andget
enoughsleep.Ifyourbodyandmindarehealthy,youllbemuchmorecalmandcon dentheadingintothis
challenge.So,takesomemeaningfulstudybreaks!Hangoutwithyourfriends,watchyourfavoriteshowon
Net ix,goonaleisurelyjogallinthenameoftestprep.

WhichStrategyWillWorkBestforMe?

ItsokifyourSATstudyplanisunique!MakingthemostofyourSATstudyschedulerequiresadaptingittoyour
particularneeds.ExperimentwithvariousstrategiesfromtheCollegeBoardso cialbook,andseewhichones
workbestforyou(theresnoonesize tsallapproachtotheSAT).Bepatientifyoudontseeyourscores


sat.magoosh.com 192

shootupimmediately.Thenewstrategiesmayslowyoudownormessyouupat rstbutpracticingthemwill
giveyouextratoolsontestday.

Onceyouvepracticedenoughproblems,sothatyouverefreshedyourknowledgeandtest-takingskills,
scheduleatimeforyourselftositdownandtakeafull-lengthpracticetest.Setatimerforeachsectionandtry
and ndaquietroomtomimictest-takingconditions.Thegoalisnottogeteveryquestioncorrect,buttogeta
feelforwhatthetimingofeachsectionislike.Youmaynot nisheachtestonyour rsttry,butyoullknowwhat
strategiesyouneedtopractice.

HowDoIKnowifMyStudyPlanWorks?

Focusonyourprogress,anddontbeupsetifyour rstcouplepracticetestsarentashigh-scoringasyoudlike.
Preparingforatesttakesorganization,practiceandapositivementalattitude.Thereisnosuchthingasa
goodorabadtest-taker.Somestudentsarejustmorefamiliarwiththecontentareasandstrategiesthan
others.Test-takingisalearnedskill,sodontbediscouraged.Createachecklistofyourstudysessionssoyou
canrewardyourselfasyougetworkdone!

FindingandUsingGreatSATResources

Agreatplacetobeginlearningaboutthetestitselfisatcollegereadiness.collegeboard.org.Ifyoucan,orderan
o cialcopyoftheTheO cialSATStudyGuide(2016Edition).Itincludesfull-lengthpracticetestsaswellas
answersandexplanations.

FreeSATpracticematerialsaboundonline,butmakesurethattheyarehighquality,oryoumaybepracticing
withoutdatedmaterials.Forhelp ndinggreatSATresourcesonline,checkoutourFreeSATPracticeTestspost
onourblog.

Startdoingsomepracticeproblemstogetafeelfortheformatofthetestandtobegintolearnyourpersonal
strengthsandweaknesses.Dontworryifyougetalotofquestionswrongat rst.CreateanSATstudyfolder
andaddaFormulaSheetandaVocabSheet,whereyoucanwriteanyunfamiliarmathformulasand
vocabularyyoucomeacrossinyourstudies.Make ashcardsifthatisaneasywayforyoutolearn!


sat.magoosh.com 193

SATResources


sat.magoosh.com 194

TestDayTips

Afterallthistime,SATtestdayis nallylooming.Herearesometipstomakesureitgoesassmoothlyasyou
hope.

SATTip#1:Packthedaybefore.

Thelastthingyouwanttoworryaboutasyoushakeyourselfawakeontestdayiswhatyouneedtobring.
Heresacompletelist:
admissionsticket(printfromcollegeboard.org)
photoidenti cation(crucial!Youwillnotbeadmittedwithoutit.
severalsharpenedsoftleadNo.2pencils(theoldfashionedwoodenkind;NOTmechanical.)
apermittedcalculator
extrabatteries
awatchtopaceyourself(youcantrelyonthefactthatthetestroomwillhaveaclock.)
eraser
pencilsharpener
healthysnacks(seebelow)
awaterbottle
gum(Didyouknowchewinggumimprovesaccuracyandreactiontimes?Checkoutthisandothertest
preplifehacksonourblog!)
anout twithlayers(Yourtestroommaybetoohotortoocold.Itisrarelyjustright,Goldilocks.)

SATTip#2:Gotobedearly.

Makesureyouaregivingyourselfasolidnightofsleep.Formostteenagers,thisis8-9hours.Figureoutwhen
youneedtogetuptobecompletelyreadyandatthetestcenterstress-freeandworkbackwardsfromthistime
to gureoutwhenyouneedtogotobed.Ifyoucantsleep,though,dontforceit.Getupanddosomethingelse
andtryagaininahalfhour.Dontliethereagonizing.

SATTip#3:Wakeupearlyanddosomephysicalandmentalexercise.


sat.magoosh.com 195

Goforajog;dosomejumpingjacks.Wakingyourbodyandmindupiscrucial.Readsomearticlesfromthe
newspaperandfocuson ndingthemainidea.Tryacouplemathproblems.Avoidtheurgetodoany
last-minuteSATprep,butitisimportantthatyourbrainiswarmedupforthetest.

SATTip#4:Eatahealthy,long-sustainingbreakfast.

Welikegranola,fruit,eggsandveggies.Drinkco eeonlyifyouareusedtoit.Donttryitnowifitisnewyou
mightgetcrazyjitters.

SATTip#5:Bringacheatsheet.

NotTHATkindofcheatsheet.ButwerecommendbringinganSATstrategycheatsheetthatyoucanreview
beforethetestandthentucksafelyawayinabag.Thisshouldincludethemostimportantremindersyouve
learnedfromyourtestprepsuchasDontforgettowatchoutforcommasplices!Havingalast-minutereview
listcanhelpyourememberthatyouare,infact,preparedandyoudo,infact,gotthis.

SATTip#6:Dontloseyourcoolbeforethetest.

Librariesorcafeteriasfullofarrivingtest-takersaregenerallynotgoodplaces.Youcouldcutthenervousenergy
withaknife.Somanyanxiousstudentsfrettingaboutwhetherornottherewillbeacomparisonreadingpassage
orquizzingeachotheronlogarithms.Orevenworse,thekidleaningupagainstthewallwholookslikehecould
careless.OragroupofyourfriendspullingyouintosomeHomecomingDancedramathatdistractsyoufrom
thetaskathand.Ofcourse,eventuallyyouneedtocheckin,butifyougettothetestcenterearly,byallmeans,
pulloutyourheadphones,blastyourfavoritepump-me-upmusicandhangoutoutside.Youvedonetoomuch
preparationtolettheseotherstudentsshakeyourcon dence.

SATTip#6:Keepyourfocusduringthetest.

Asure rewaytonotgetthescoreyouwantistoconstantlypayattentiontothetest-takersaroundyou.How
doesthatgirlinfrontofyouanswermathproblemssofast?Whycantthatkidstoptappinghisfoot?Trytostay
inthezoneandfocusonyourparticularstrategies.Everyoneisdi erent,andwhattheseotherstudentsare
doingisirrelevant.Anditshouldgowithoutsayingthatyoushouldavoidanytemptationtopeeratyour
neighborsanswersheet.Cheatingisnotariskyouwanttorunhere.Andhesprobablywronganyway.


sat.magoosh.com 196

SATTip#7:Eatonyourbreaksevenifyouarenthungry.

Yourbrainneedsfueljustliketherestofyourbody,evenifyournervousstomachdoesntthinkso.Andthelast
thingyouwantistoletaperfectlygoodsnackbreakgotowasteonlytoregretitlaterwhenyourstomachis
howlinghalfwaythroughtheSciencesection.Fruitisgreattestfuel:thenaturalsugarshelpgiveyouenergy.

Makesuretohavesomethingwithproteintoo,likeahandfulofnuts.Manyofourstudentsswearbypeanut
butterandjellysandwichesortrailmixwithchocolatefortheperfectcomboofsugarandsustenance.Dont
forgetthewater!

SATTip#8:Rewardyourself.

Youworkedhardforthis,andnomatterhowyouthinkthetestwent,afterwardstreatyourselftosomethingyou
enjoy.Youneedthementalbreakandrelaxation.Butasimportantasanafter-testrewardis,Ithinkthatwhatis
evenmoreimportantisrewardingyourselfduringtheexam.No,youcantwhipoutacookieinthemiddleofthe
Englishsection,butgiveyourselfmentalpatsonthebackwhenyoucatchyourselfdoingsomethingright.So
manystudentsbeatthemselvesupduringatestforwhattheythinktheyaredoingwrong.Putastoptothis
destructivementalityandinsteadcongratulateyourselfwhenyou ndyourselfdoingsomethingright:stickingto
yourpacingorrecognizingagrammarerroryouvemissedbefore.Itsalongtestandapositiveattitudeis
crucial!

Cheerstoafantastictestday!


sat.magoosh.com 197

BookReviews

Mostofthebigtestprepbookpublishers(ImtalkingCollegeBoard,PrincetonReview,Kaplan,andBarrons)
havereleased,orareonthecuspofreleasing,abookfortheredesignedSAT.ButnotallNewSATprepbooks
arecreatedequal:someyoullwanttouseintheirentirety(thoughonlyafew),othersyoullwanttouseonly
partsof,andothersyoullwanttosteerclearof.

Tohelpyoumaximizeyourstudytime(andsaveyourhardearnedcash)Ivereviewedthetopoptionsforbest
SATbooksonthemarketandbrokenthemdownintothegood,thebad,andthe(notso)ugly.Ifyouseethe
samebookinmultiplesections,dontworryyourenotgoingcrazy.Thatjustmeansthatsomepublishersdoa
goodjobonsomethings(likepracticetestsoraspeci csectionoftheNewSAT)whiledoingasub-parjobon
otherthings.Thisdoesntnecessarilymeanthatyoushouldntbuythebook,butitmightmeanthatyouwont
usethebookinitsentirety.

Then,forevenmore,scrolldownformycompletebookreviewofeachSATbook.

TheGood(theBestSATBooksof2016)

TheO cialSATStudyGuide(2016Edition)
(ThePracticeTests)
TheO cialSATStudyGuideistheholygrailofSATtestprep:abookthatcontainsquestionsbythewritersof
thetest(TheCollegeBoard).Nowherewillyougetabettersenseofwhattoexpecttestday.Speci cally,there
arefourfull-lengthpracticetestswithexplanations(seeTheBadforacommentontheexplanations).

BarronsNewSAT
Barronsoveralldoesadecentjobofrecreatingtestquestionsandagoodjobofdissectingthetestando ering
helpfulexamples.Outofthemainthird-partypublishers(KaplanandThePrincetonReviewarethetwoothers),
thisbookistheoneyoushouldget.ScrolldownforareviewofBarronsStrategiesandPracticefortheNew
PSAT.


sat.magoosh.com 198

TheBad

KaplanNewSAT2016
(TheMathSection)
KaplandoessuchapoorjobofapproximatingtheverbalsectionthatIthinkstudentswouldbebettero not
usinganyKaplanverbalcontent.Thewritingsectionquestions,forinstance,makethetestseemoverlyeasy.To
boot,thepassagesaresupposedtobewrittenbyaprofessional;thepersonwritingthesesentencesisanything
butaprofessional,laboringtostringsentencestogetherinawaythatiscohesive(eloquencebedamned!).
(ReadmyfullreviewofKaplanNewSAT2016below.)

TheO cialSATStudyGuide(2016Edition)
(TheExplanations)
CollegeBoardexplanationsleavesomethingtobedesired.Soifyoudidntquiteunderstandwhyyoumisseda
question,theexplanationsoftenarentveryhelpful.Thestrategy/testoverviewsectionofthebook,which
comesatthebeginning,isntveryuseful.Icanimaginemanystudentsscratchingtheirheadsafterreadingsome
ofthetediousandvagueexplanationsofquestiontypes.

The(notso)Ugly

KaplanNewSATPremier2016
(TheVerbalSection)
ThoughKaplansmathismentionedinthesectionabove,theverbalsectionisadi erentbreedaltogether.
Kaplansgeneralguidedoesagoodjobofbreakingupmathconceptssoyoucangetasenseofthedi erent
areascovered.Additionally,thereisahelpfulpracticequizattheendofeachsection.

PrincetonReview500+QuestionsfortheNewSAT(2016Edition)
ThePrincetonReviewhas,onthewhole,doneanokayjob.Thequestionsarenttotallyaligned,sometimesthey
are ato ,butingeneralwhatyougetisasimpli edversionoftheSAT:goodforbeginners,notsogoodfor
thosewantingtopreparefortherigorsoftheactualtest.HeaddownbelowforafullbookreviewofPrinceton
Reviews500+QuestionsfortheNewSAT.


sat.magoosh.com 199

2016BestSATBookReviews

TheO cialSATStudyGuide2016

ThisislikereviewingtheBible:TheO cialSATStudyGuideissofoundational
toSATsuccessthatitseemssacrosancttosuggestotherwise.So,insteadof
givingmy ve-starstampofapprovalrating,Imgoingtosaysomethingthat
mightseemheretic:thisbookis awed.

Sure,thequestionsinthisbookmightbeindispensable,butisthebook
uniformlyuseful?No.Andhereswhy.

TheQuestions
ThequestionsinthisbookareMt.Sinailevel.Inotherwords,itistheSATgods
partingtheskiesandgivingusatasteofwhattoexpectwhenthetestdebuts
inacoupleofmonths.Allthenuances,allthetraps,alltheidiosyncrasiesare
thereforustobehold;andthemoreyouunderstandthem,thebetterpreparedyoullbetestday.
However,thereareacoupleofbuts,someofwhicharebig

Thepracticequestionscontainedinthisbookareavailableforfreeonlineandsincethequestionsarebyfarthe
bestthingaboutthisbook,youmightrightlystarttowonderifyouactuallyneedtopurchasethisbook.What
followsmightindeedmakeyouthinkyoudontneedthebookatall.

TheExplanations
Hi,Imgoingtoplaypretendtoday.ImgoingtopretendImthepersonorpeoplewhowrotetheexplanations
forthequestions.

Choice(A)isthebestanswerbecausetheinformationinthepassagebestsupports(A).Line11-13,<insertquote
here>showthis.

(B),(C),and(D)donotspeci callyanswerthequestion.


sat.magoosh.com 200

Tobefair,oftentheexplanationtotherightanswerismorethorough.Asforwhythewrongansweriswrong,
forgetaboutit.Thatisaboutasspeci casitgets.Intheend,youarelikelyto ndtheseexplanationsvagueto
thepointofinfuriationandtrulyunderstandingyourmistakes,isoneofthebestwaystoimprove.

TheStrategy
Ilikethewaythatthebookbreaksdownthetestsowecanseewhathaschangedsincetheoldtest.However,
thereissomuchterminologythatIimaginestudentsgettingboggeddownthinkingthattheyhavetoknow rst
o whatnonrestrictiveandparentheticalelementsmean,andthatthismouthfulofadescriptionforsomething
relativelystraightforward,is ledunderconventionsofpunctuation.

Whatwedontgetinthecaseofwriting,isasolidreviewofthegrammarconceptsandastraightforwardwayof
describinghowthoserelatetothetest.Nordowegetstrategiesonhowtoapproachthesequestions.InfactI
feellikethe rstfewhundredpagesismoreforpeoplelikemethatwanttounderstandhowthetestis
constructedandlessforactualstudentswhoneedhelpunderstandingthehowtothematerial,notthewhat.

TheVerdict
Forreview,youaremuchbettero goingwithanyofthemajorpublishersonthemarket.Thoughforquestions
thereisnobettersourcethantheCollegeBoard.ThefactthattheCollegeBoardhasmadethiscontentfree
online,however,makesthisbookunnecessary,ifnotunhelpful.Nowletshopethecloudsdontpartandthe
CollegeBoardstrikesmedownwithlightning.

BarronsStrategiesandPracticefortheNewPSAT/NMSQT

Sure,thisisnotaguidefortheNewSAT,whichBarronsisstillworkingon.But
didyouknowthatthePSATandSATarethesamecontent-wise?Sure,thereare
subtledi erencesinthewaytheeasyanddi cultquestionsarearranged.For
instance,thePSATtendstohavemoreeasyquestionsandfewerhard
questions.But,honestly,mostcantreallytellthedi erence.

SoifyouarelookingforpracticecontentfortheredesignedSATexam
(debutinginMarch2016),BarronsStrategiesandPracticefortheNew
PSAT/NMSQTisagreatplaceto ndit.Therearetwopracticetestsattheend
ofthebookandsomepracticecontent,albeitnotmuch,atthebeginning(thisis
aslimvolume,cominginatamere257pages).


sat.magoosh.com 201

GiventhattherearesofewpracticebooksouttherefortheNewSAT,andthosefewthatareouttheredont
havethebestpracticecontent,BarronsforthePSATisavaluableaddition.Soifyou ndthatyouhaveburned
throughthetestsintheCollegeBoardSATStudyGuide,orifyoudlikeawarmupfortherealtest,theBarrons
isagoodplacetostart.Youllgetusedtotheformatandthequestiontypes.

Thatisnottosaythebookisnotwithout aws.Afterall,nobodycanreallycreatequestionsthatareofthe
standardoftheactualtest.ButBarronsdoesadecentjob;preppingwiththisbookwilllikelyhelpyouraiseyour
score.UsingthisinconjunctionwiththeCollegeBoardbook,though,isbesttoreallygetafeelforthetest,
speci callythewronganswerchoicesthetestwilltrytofoolyouwith.

Also,thisbookisaboutpracticequestionsandadviceonstrategiesandtechniquesisscant.Soattheveryleast
youllneedtosupplementthisbookwithonethatprovidestipsonhowtoapproachthetestandareviewof
fundamentalsandconceptscoveredonthetest.

Verdict:
Agoodplaceforacoupleofpracticetests.
Grade:B+

KaplanNewSAT2016Strategies,PracticeandReviewwith3PracticeTests

KaplanhascreatedaJekylandHydeguide.Ontheonehandthereisan
excellentmathsection.Kaplanreallytakestimetoteachbasicstrategies.
Studentswillunderstandmanyoftheconceptsbeforemovingontothepractice
questionsquestionsthatreinforcemanyoftheprincipleslearnedinthereview
section.IhadtypicallyavoidedusingKaplanforthelastSAT,sincethecontent
wastooeasy;studentswouldoftengainafalsesenseofcon dence.Inthis
book,someofthequestionsareactuallymoredi cultoratleast
involvedthenthequestionso eredinCollegeBoardbook.

Thenthereistheverbal.Tobehonest,itis atoutterrible.Sure,thestrategies
arehackneyedKaplanfellswholerainforeststotellstudentsobvious
approaches(Dontforgettoxyz).ButitsthecreationoftheactualquestionsandpassagesthatKaplanjust


sat.magoosh.com 202

cantseemtobringittogether.Whatthatmeansforthestudentisthattheyaregettingatestthatdoesntreally
preparethemfortheactualSAT.Questionsandpromptsareshoddyimitationsoftherealthing.

Forinstance,theessaysthatarepartoftheWritingsectionaresupposedtobewell-craftedpiecesimparted
withastrong,writerlyvoice.Instead,itfeelslikeKaplanhadsomehaplesshigh-schoolstudentcobbletogether
anessay.Goneisthesenseofcontrolandvoicethatevensemi-professionalwriterscanpullo .Sure,this
soundslikeasnobbishquibbleonmypart,butthetruthisthisshoddinessa ectsthequestions.Manyarejust
atouttooeasybecausethepassageitselfistoobasicanddoesntlenditselftonuancedquestiontypesor
trickytrapanswersthataresuretobeawaitingtestday.

ReadingComprehensionisntquitesobad.Still,thepassagesaretakenfromtextbooksnotreputablejournals.
Whatthattranslatesisalotofdrypassages,writtenataleveldevoidofthemoresophisticatedproseanddense
ideastheCollegeBoardexpectsyoutobeabletonavigate.Whatyoullgetonceagainisasectionthatlooks
liketheNewSATReadingbutistestingacomprehensionlevelclosertothattestedontheTOEFLtest(thatsthe
testforthosewholearnEnglishasaforeignlanguage).

Verdict:
UsetheKaplanbookformathstrategiesandpractice;steerclearoftheverbal.
Math:B+
Verbal:D-

PrincetonReview500+PracticeQuestionsfortheNewSAT

Atthetimeofthisreview,PrincetonReviewhasyettoreleaseageneralguide.A
practicetestguidewillbedueoutshortly(IllreviewthatassoonasIcan).For
now,weareleftwithlotsofquestionsandverylittleguidance/strategies/concept
review.

WhileIcouldeasilyfaultthisbook(500+PracticeQuestionsfortheNewSAT:
CreatedfortheRedesigned2016Exam)forthelackofthesethings,itwouldbe
unfairtodoso,becausethepurposeofthisbookistobeaquestionbank.And
onthatlevelitscoresaresoundingmeh.Thatisnottosayitwontbeofuseto
lower-levelstudents(youcanprobablyseewheremyreviewisgoing).The


sat.magoosh.com 203

PrincetonReviewhascreatedquestionsthatwhileperfectlylegitimatearentquiteascomplexandnuancedas
thosefoundontheactualtest.Andyouknowwhat?Thatsnotnecessarilyabadthingifyouarejuststarting
o .

Formanythatsexactlywhatwillbehappeningonthenewtest.Thequestiontypesandtheformatwillbe
unfamiliar.ThePrincetonReviewisagreatintroduction.Youwontfeelquiteaschallengedandwillbeableto
focusonthenewformat.Onceyoufeelcon dentwiththePrincetonReviewbookyoucanmoveontoCollege
Boardmaterial.

IshouldmakeitclearthatImnotsayingohthisbookisjustaneasyversionofthetest.Imsayingitisaneasier
versionoftherealthinganditisavalidversion.Unlike,Kaplansverbal,forinstance,whichatleastfor
verbalismucheasierthantherealtest,yetaninaccuratekindofeasy,thePrincetonReviewmostlystaystrue
theunderlyingsubtletiesofthequestionsandanswerchoices.Itjustdoesnthavethehard-levelquestionthat
makeup15-20%oftheactualtest.

Ofcourse,youllhavetopickupanotherbooktohelpyouwithstrategiesandtoreviewthefundamentals.
Indeed,youllneedabookthatalsohaspracticetests,sincethisbookisonly lledwithonedrillafteranother.
Butasacompanionguidetoabookofstrategiesandfundamentals,thisbookisagreatplacetostart.

Verdict:
Agreatplacetostartdrilling,especiallyifyouarenewtothetest.Butforpracticetestsandmoredi cult
questionsyoullneedanotherbook.
Grade:B


sat.magoosh.com 204

AdditionalTestPrepResources

MagooshSATYouTubeChannel
WehelpyouacetheSATwithshort,informativevideosthataren'ttotallyboringtowatch.:)SearchMagooshSAT
onYouTubetocheckusout.

ACTPrepResources
WeallknowthattheACTandSATtestareprettydi erentfromoneanother:di erentformats,di erentscoring,
di erentsubjectstested.However,ifyourepreparingforbothexams,youllstarttonoticethatthereisquitea
bitofoverlapwhenitcomestorequiredskills.

DontthinkaboutthetwoexamsastheACTvs.theSAT,butasmoreofanACT/SATvenndiagram.Theysharea
lotofcontent,especiallyatabasiclevel.

So,withthatinmind,feelfreetousesomeofMagooshACTsfreeresourcestobrushuponyourbasicskills:
ACTFlashcards:Toimproveyourgeneralmathandgrammarskills.
SATMathFormulaeBook:Forbrushinguponyourbasicmathskills.


sat.magoosh.com 205

YourCollegeSearch

Notsurewheretobeginyourcollegesearch?Feelingcompletelylostandconfused?

Wellguesswhat?Inmyhumbleopinion,Ithinkthatmeansyouareactuallyonestepaheadofthegame.Despite
allthetalkoutthereaboutacollegesearch,thevastmajorityofcollegeapplicantsnevertrulysearchfora
schoolthemselves.Theymakealistofcollegestheirfriendslike,theirparentslike,theircounselorslike,or
simplyschoolstheyveseenont-shirtsoronTV.Iwasoneofthesestudents.Ineverlookedoutsideofmyown
backyard,andwhenIstartedlearningaboutalltheamazingcollegesanduniversitiesthatareouttherelateron,
boy,didIhavesomeregrets.
So, rst,patyourselfonthebackjustforbeingonthemissiontouncovertherightcollegeforyou.Now,letstalk
abouthowyoucan ndit.

Step1:TakeanInventoryofYourself
Askyourselfthesequestions:WhatdoIlikeanddislikeaboutmycurrentschool?HowdoIlearnbest?DoIlike
tostudyaloneorwithagroup?DoIliketohavepersonalinteractionswithmyteachers?DoIlikesocializingin
largeorsmallgroups?Doesclimatehaveaseriousimpactonme?DoIneedtobeclosetohome?DoIliketobe
involvedinalotofactivities?DoIneedschoolspirit?sports?theatre?Therearesomanyquestionstoaskand
lotsofresourcesforself-assessmentsoutthere.Hereisonethatweparticularlylike(onschoolbu .com).Taking
stockofwhatyouwantandneedwillhelpyouknowwhattolookforasyouresearchschools.

Step2:EstablishYourMust-Haves
Takealookatthefollowinglistofcriteriaanddeterminewhatyourrequirementsareforeachcategory.Are
thereanydeal-breakers?Arethereanyyoudonthaveparticularlystrongfeelingsabout?Becarefulof
eliminatinganythingyouarentsureaboutatthispoint:Ifyouveneverseenasmallliberalartsschoolbefore,
howdoyouknowyoudontwanttogotoone?Ifyoudontknow,thenmakesomecollegevisitsbeforeyourule
anythingout.
Curriculum:Haveyoualreadydecidedonamajortopursue?Doyouwanttomakesureyouhave
options?Doyouwanttohaveacorecurriculumortotalfreedom?Doyouwanttodoublemajor?Doyou
wanttodesignyourownmajor?
Location:Isitimportantforyoutobeclose/farfromhome?Inacertaingeographicalregionorclimate?
Size:Small?Medium?Large?Mega?Dontforgettoconsiderthesizeoftheindividualprogramyouare
lookingat,notjustthewholeschool.


sat.magoosh.com 206

Resources:Doyouneedspeci cresourcesforlearningneedsorpsychological,social,ormedical
concerns?Desireastrongculturalorethnicgroupsupportnetwork?Wantrobustinternshipsorresearch
opportunities?
Activities:Whattypesofactivitiesareyouinterestedinparticipatinginincollege,bothonando
campus?Whattypeofleadership,service,studyabroad,etc.opportunitieswouldyouliketopursue?
CostandFinancialAid:Howmuchcanyou/yourfamilya ordtopayforcollege?Whatlevelof nancial
aiddoyouneed?

Step3:Research,Explore,andVisit
Onceyouvedeterminedwhatyouarelookingforinacollegeexperience,youcanbeginexploringschoolsthat
meetyourcriteria.CollegeNavigatorandCollegeScorecardaregoodtoolstolaunchyourcollegesearchbased
onthecriteriayouveestablished.(Youcanalsocheckoutourlistofthebestfreeonlineresourcesforyour
collegesearchbelow!)

Onceyouvedevelopedanexploratorylist,layoutaplanforvisits.Ifyoudonthavethetimeormeanstovisit
collegesfaraway,pickaselectionofdi erenttypesofschoolswithinadaysdrive.Planoutahandfulof
weekendtripstovisit2to4schoolseachweekend(anymoreandtheywillallstarttoruntogetherinyourmind).

Fortheschoolsyoucantvisit(andevenfortheonesyoucan),doresearchonline,getonmailinglists,visitwith
therepwhentheycometoyourschool,talktocurrentstudentsandalumni,andgotocollegefairs.Learnas
muchasyoucan!

Step4:Re neYourList
Onceyouvedoneyourexploration,re neyourcollegelist.Thelengthofthislistcanvary.Somestudentshave
threeorfourschools,othershave fteen.Whateveryoudo,makesuretocoveryourbases.Youshouldhavea
balanceofgoodbets(schoolsyouhavemorethana75%chanceofgettinginto),targets(schoolsyouhavea
25%to75%chanceofgettinginto),andreaches(schoolsyouhavelessthana25%chanceofgettinginto).

Step5:ExpressYourInterest
Nowthatyouknowwhichschoolsyouareinterestedin,makesuretheyknowittoo!Takeadvantageof
opportunitiesforinterviewsandmeetingswithrepresentativesandalumni;getontheirmailinglist.Evenifthese
contactsdontseemtoleadanywhere,whenitcomestimeforyourapplication,youwillbeabletochecko all
sortsofboxesthatshowyourdemonstratedinterestintheschoolandyouressayswillre ectyoure ort.Goto
eachschoolswebsiteand ndtheadmissionsrepresentativewhowillbereadingyourapplication(sometimes


sat.magoosh.com 207

thisisbasedonthealphabetorgeography,sometimesonothercriteria).Thispersonisgoingtobeyourcontact
throughouttheadmissionsprocess.De nitelydontpesterthem,butdonthesitatetoreachoutifyouhave
importantquestionsorifyouneedtofollowupwithinformationregardingyourapplication.

Remember,thisisYOURcollegesearch!Allsortsofpeoplefromyourparentstoyourfriendstoyour
teachersaregoingtohavetheirownthoughtsaboutit.Listentothem,butdontbeswayedbytheopinionsof
others.Ifyouhaveabettersenseofwhoyouareandwhatyouwantinaschool,youllbemuchlesslikelytofall
intothistrapandfarmorelikelytofallinlovewithyourchosenschoolonceyougetthere.


sat.magoosh.com 208

FreeCollegeSearchResources
FortunatecollegeapplicantsoftheInternetage!Youhavesomanyresourcesatyour ngertipsto ndtheright
schoolforyou!Herearetenofourfavoritefreeresourcesforanonlinecollegesearch.

CollegeScorecard:Auser-friendlyandengagingtooltosearchforschoolsfromtheU.S.governmentCollege
A ordabilityandTransparencyCenterbasedonarangeofcriteria.Youcanalso ndouthowmuchtheaverage
studentpaysforaschool,howmuchtheyborrow,andgraduationandemploymentprospects.

CollegeCon dentialSuperMatch:Asearchtoolthatuses20criteria(attributessuchaslocation,major,
diversity,specialservices,andpartyscene)tohelpyou ndtherightschoolforyou.Itusesafuzzyapproachto
rankingschoolsbasedonyourpreferences,meaningit ndsnotonlytheperfectmatches,butalsotheschools
thatareprettydarnclose.

CollegeNavigator:NotquiteasfunasCollegeCon dentialstool,butisanauthoritativeonehostedbythe
NationalCenterforEducationStatistics.Youcansearchforschoolsbycriteria,comparethemsidebyside,and
pinpointschoollocationsonaninteractivemap.

BigFuturefromtheCollegeBoard:Anotherwell-respectedsearchtool.Ofparticularnoteistheabilityto nd
schoolsbasedontestscores,thosethato eracademiccreditforadvancedhighschoolcourses,andthosethat
meet nancialaidneeds.

U.S.NewsandWorldReportBestColleges:Themostfamousnationalsourceofcollegerankings,highly
anticipatedeachyear.Youwillneedeitherawebsitesubscriptionorapurchaseofthemagazineforfulldetails
ontherankings.

CollegesthatChangeLives:Anon-pro torganizationpromotingasmallgroupofliberalartscolleges
supportingastudent-centeredcollegeexperience.Alsohasnumerousqualityresourcesforageneralcollege
search.

BestColleges:Usescompiledinformationfromvarioussourcestorankcollegesonspeci cfeatures(for
example,onlinecollegesthato erfreelaptops,tuition-freecolleges,andcollegeswithlowestout-of-state
tuition)


sat.magoosh.com 209

eCampusTours:O ers360-degreevirtualtoursof1300campuses.

CollegeWeekLive:Avirtualcollegefair.Livechatwithadmissionsrepresentativesandstudentsat300+
collegesanduniversities.

Niche(formerlyCollegeProwler):aimsprovidestherealdirtoncolleges;studentsreviewaspectsofthe
collegeexperienceattheirrespectiveschools,suchasthesportssceneorcampusfood.Lotsofinformation;
takeindividualstudentopinionswithagrainofsalt,though.

Unigo:SimilartoNiche.Compilesstudentreviewsonvariousschools.


sat.magoosh.com 210

IntrotoFinancialAid

Forhighschoolseniorseverywhere,navigatingtheinsandoutsof nancialaidcanbecompletelyconfusing.So
manydeadlines,somanyforms,somanyweirdacronymsthatwouldmakeinterestingbandnames(FAFSA
NATION,anyone?).

Inallhonesty,everythingcangetabitoverwhelming.Inthissection,welltrytoguideyouthroughthebasicsof
theprocessstep-by-step.

But rstofall

WhydoIneed nancialaid?
Collegeisawalletdemolisher.Costsnotonlyincludetuition,butroomandboard,textbooks,personalandtravel
expenses,andwell,younameit.Itsexpensive!Moreandmorestudentsthesedaysaregraduatingwith
enormousamountsofdebtdebtthatwouldnotexistinsuchconsiderablequantitiesifmorepeoplehad
knownhowtomaximizetheamountof nancialaidtheywereeligiblefor.

Thereisaniftycalculatoron naid.orgforcalculatinghowmuch nancialaidyoucouldqualifyforbasedonyour


EFC(EstimatedFamilyContribution).Evenifyoudontthinkyouwillqualifyformuchaid,youshouldtryanyways.
Therereallyisnoriskinvolved.

Financialaidcancomeintheformofinstitution-based nancialaid,givenoutbytheschool;federalaid;and
separatescholarshipsrunbyprivateorganizations.Theyareallimportant,andtheycanallsaveyoufrom
becomingbankrupt,becominghomeless,andlivinginacardboardboxcryingoveryourframeddiploma.


sat.magoosh.com 211

Imagefromspeedbump.com

Deadlines

TheFAFSA,CSS,scholarshipapplications,Calgrants,taxreturninformationtherearesomanyformstosendin!
Allcollegesusuallyhaveaspeci cdeadlineastowhentheydliketoreceivethese.

Beforeyoustartanyofthisprocess,itsimportanttowritedownallofyourdeadlinesforallofyourschools.
Whenyouremailingmaterialin,deadlinesareusuallyalotmore exiblebutitsstillimportanttogetallof
yourmaterialsmailedinontime.
Foralotofcolleges,theseformsaredueatthebeginningofFebruary.Othercollegeshavedeadlinestoward
MarchorevenAprilandMay.Ifyoucant nddeadlineinformationononeofyourcolleges,calltheAdmissions
O ce!Itwonthurttocheck!

SowhatarealltheseformsIhavetosubmit?


sat.magoosh.com 212


Themostpopularandwell-knownformistheinfamousFAFSA.

ItstandsfortheFreeApplicationforFederalStudentAid,whichbasicallyspeaksforitself.Itscompletelyfree,
andifyouenterinall nancialinfoandsubmityourapplication,youcanreceiveanestimateforhowmuchthe
governmentcangiveyou.Thisapplicationisnicebecauseyoucansubmitthesameonetoallofyourschools.

WithinafewweeksofsubmittingtheFAFSA,apaperStudentAidReportwillbemailedtoyou,detailing
informationfromyourFAFSAandyourExpectedFamilyContribution.Ifyoualsoprovidedanemail,youll
receivealinktoyourresultsafterjustafewdays.

Overall,theFAFSAabitfasterandeasiertocompletethantheothermajor nancialaidform

CSS
TheCollegeScholarshipServiceisnotrequiredbyeveryschoolinfactmanyschoolsonlyrequirethe
FAFSA!Thereareabout200colleges,however,thatdorequiretheCSSPro le.Thesecollegesincludealotof
thetopschools,liketheIvyLeagues.Checkwithyourschooltoseeifitrequiresthisform!Otherwise,youmight
bemissingoutonahugeportionof nancialaid.
UsinginformationprovidedontheCSS,theywillcomputehowmuchinstitutionalaidyouareeligiblefor.Ifyour
EstimatedFamilyContributionislessthanthecollegestuitionfees,youcanqualifyforneed-based nancialaid!
TheCSSPro leisrunbyCollegeBoard,andbecauseCollegeBoardhasntmilkedyourwalletenoughalready,
thereisa$25applicationfeeanda$16feeforeveryadditionalschool.Ifyouareapplyingto10+schools,we
sympathizewithyou.

Separate nancialaiddocuments

1)Institution-speci c
TherearesomeschoolslikePrincetonandtheUniversityofPennsylvaniathathavetheirownseparate nancial
aidformsaswell.ForPrinceton,ifyoualreadysubmittedaCSSform,youcansynctheinformationfromCSSto
Princetonsownapplicationformtomaketheprocessquicker.Theseinstitution-speci cformscanbedi cultto
nd.Makesureyouknowifanyofyourschoolsrequirethis!


sat.magoosh.com 213

2)Calgrants
IfyouareapplyingtoanyschoolinCalifornia,makesureyouhavethisdone!Calgrantso er nancialaid/grants
toanyoneattendingaCaliforniauniversityandaccordingtoitswebsite,youcanreceiveupto$12,192in
nancialaidthatyoudonthavetopayback.Allyouhavetodois llouttheFAFSAandhaveyourcounselor ll
outaGPAveri cationform.ThedeadlineisnormallyearlyMarch.

3)Taxreturns
Thisisaparticularlyannoyingpartofthe nancialaidprocess,butitsnecessarynonetheless.Lotsofschools
requirecopiesofyourtaxreturnsinordertoverifyyourimputedinformation.Thismeanstheentiretaxreturn
packet:allformsandschedulesincluded,signedbybothofyourparents.CollegeBoardhasaniftyservice
calledIDOCthroughwhichyoucansendthesedigitally.Youcanalsomakecopiesandphysicallysenditoutto
yourcolleges,butthisisverylaborintensive.

Other

MeritAid
Thisisanothertypeofstudentaidwhichisawardedbasedo ofyouracademic,athletic,musical,etc.
achievements,ratherthanyourfamilysituation.Unfortunately,mostoftheelitecollegesintheU.S.donoto er
academicmeritaid(andonlyo erneed-basedaid),sothisisconsiderablylesscommon.

Privatescholarships
Sometimes,youcanreceiveaninstitutionally-basedscholarshipjustbysendinginyourapplication.Noextra
formsneeded.Therearetonsofotherscholarshipsoutthere,however,thatrequirealittlebitmoreinitiativeon
yourpart.

Goandlookupscholarshipsthatarebeingo eredlocallyorscholarshipsthatpertaintoyourdemographic,
careergoals,majorchoice,etc.Forbeshasanicelistof10highest-payingscholarshipsforcollege.Butyoure
notlimitedtojustthose!Beproactivetherearetonsofthemoutthere.

Takeaway


sat.magoosh.com 214

Thisarticledoesnotcoverallofyour nancialaidoptionsbyfar,buthopefullyithelpedoutabitwithclearingup
theprocess!Itstricky,butintheenditwillde nitelybeworthit.

Dontletthecostofcollegegetyoudown.Withtheright nancialaid,youcandoanything.


sat.magoosh.com 215

Signuptodayatsat.magoosh.com.:)


sat.magoosh.com 216

You might also like